eureka 2010

144
CONTEÚDO AOS LEITORES 2 XV OLIMPÍADA DE MATEMÁTICA DE MAIO Problemas e Resultado Brasileiro 3 XX OLIMPÍADA DE MATEMÁTICA DO CONE SUL Problemas e Resultado Brasileiro 7 L OLIMPÍADA INTERNACIONAL DE MATEMÁTICA (IMO) Problemas e Resultado Brasileiro 9 XXIV OLIMPÍADA IBEROAMERICANA DE MATEMÁTICA Problemas e Resultado Brasileiro 11 ARTIGOS PAR OU ÍMPAR? EIS A QUESTÃO Samuel Barbosa Feitosa e Einstein do Nascimento Júnior 13 GEOMETRIA DO TRIÂNGULO: FATOS E PROBLEMAS Carlos Yuzo Shine 28 SÉRIE HARMÔNICA DE NÚMEROS PRIMOS Lenimar Nunes de Andrade 45 COMO É QUE FAZ? 50 SOLUÇÕES DE PROBLEMAS PROPOSTOS 52 PROBLEMAS PROPOSTOS 59 AGENDA OLÍMPICA 61 COORDENADORES REGIONAIS 62

Upload: demi-de

Post on 20-Feb-2016

321 views

Category:

Documents


1 download

DESCRIPTION

GEOMETRIA DO TRIÂNGULO: FATOS E PROBLEMAS Carlos Yuzo Shine 28 SÉRIE HARMÔNICA DE NÚMEROS PRIMOS Lenimar Nunes de Andrade 45 XV OLIMPÍADA DE MATEMÁTICA DE MAIO Problemas e Resultado Brasileiro 3 SOLUÇÕES DE PROBLEMAS PROPOSTOS 52 XX OLIMPÍADA DE MATEMÁTICA DO CONE SUL Problemas e Resultado Brasileiro 7 CONTEÚDO XXIV OLIMPÍADA IBEROAMERICANA DE MATEMÁTICA Problemas e Resultado Brasileiro 11 L OLIMPÍADA INTERNACIONAL DE MATEMÁTICA (IMO) Problemas e Resultado Brasileiro 9 ARTIGOS

TRANSCRIPT

Page 1: Eureka 2010

CONTEÚDO

AOS LEITORES

2

XV OLIMPÍADA DE MATEMÁTICA DE MAIO Problemas e Resultado Brasileiro

3

XX OLIMPÍADA DE MATEMÁTICA DO CONE SUL Problemas e Resultado Brasileiro

7 L OLIMPÍADA INTERNACIONAL DE MATEMÁTICA (IMO) Problemas e Resultado Brasileiro

9 XXIV OLIMPÍADA IBEROAMERICANA DE MATEMÁTICA Problemas e Resultado Brasileiro

11 ARTIGOS

PAR OU ÍMPAR? EIS A QUESTÃO Samuel Barbosa Feitosa e Einstein do Nascimento Júnior

13 GEOMETRIA DO TRIÂNGULO: FATOS E PROBLEMAS Carlos Yuzo Shine

28 SÉRIE HARMÔNICA DE NÚMEROS PRIMOS Lenimar Nunes de Andrade

45 COMO É QUE FAZ?

50

SOLUÇÕES DE PROBLEMAS PROPOSTOS

52

PROBLEMAS PROPOSTOS

59

AGENDA OLÍMPICA

61

COORDENADORES REGIONAIS

62

Page 2: Eureka 2010

Sociedade Brasileira de Matemática

EUREKA! N°31, 2010

2

AOS LEITORES

Por mais um ano consecutivo estamos iniciando a realização da Olimpíada Brasileira de Matemática. A Olimpíada Brasileira de Matemática – OBM tem crescido substancialmente nos últimos anos, contando, em 2009, com a adesão ao evento de mais de 3.700 escolas, sendo 2.180 da rede pública e 1.608 da rede privada de ensino, o que implicou em uma participação efetiva de cerca de 180.000 jovens estudantes e seus professores. Além disso, a iniciativa contou com a colaboração de professores universitários em 155 instituições de ensino superior: eles participaram de todas as atividades, inclusive aquelas referentes à OBM Nível Universitário em atividades de coordenação, divulgação, treinamento de alunos, aperfeiçoamento de professores e aplicação das distintas fases da Olimpíada Brasileira de Matemática. Paralelamente, o projeto apóiou a realização de Olimpíadas Regionais de Matemática, contando com a participação de 165.148 estudantes das escolas públicas e privadas em todo o Brasil nas competições estaduais. No que se refere à participação em competições internacionais, os resultados foram excelentes: Em particular, Henrique Ponde conquistou a oitava medalha de Ouro do Brasil na IMO. Além disso, nesta IMO, todos os alunos da equipe brasileria ganharam medalhas. Durante 2009 a CAPES e o CNPq lançaram o Programa de Iniciação Científica – Mestrado (PICME) para medalhistas da OBMEP e OBM, beneficiando 19 estudantes premiados na Olimpíada Brasileira de Matemática – OBM, com o objetivo de aumentar o número de matemáticos no país, e oferecer uma formação matemática mais sólida a jovens profissionais de outras áreas científicas e tecnológicas. Todos estes resultados nacionais e internacionais demonstram que, além de influenciar positivamente o ensino da Matemática nas instituições de ensino fundamental, médio e superior, conseguimos detectar jovens muito talentosos que são estimulados a seguir uma carreira científica, o que é fundamental para o crescimento da Ciência e Tecnologia no país. A Olimpíada Brasileira de Matemática é um projeto conjunto da Sociedade Brasileira de Matemática, do Instituto Nacional de Matemática Pura e Aplicada (IMPA) e conta com o apoio do Conselho Nacional de Desenvolvimento Científico e Tecnológico (CNPq) e do Instituto Nacional de Ciência e Tecnologia de Matemática (INCTMat).

Os editores

Page 3: Eureka 2010

Sociedade Brasileira de Matemática

EUREKA! N°31, 2010

3

XV OLIMPÍADA DE MAIO PRIMEIRO NÍVEL

PROBLEMA 1 A cada número natural de dois algarismos associamos um dígito da seguinte forma: Multiplicam-se seus algarismos. Se o resultado é um dígito, este é o dígito associado. Se o resultado é um número de dois dígitos, multiplicam-se estes dois algarismos, e se o resultado é um dígito, este é o dígito associado. Caso contrario, repetimos a operação. Por exemplo, o dígito associado a 32 é o 6 pois 3 ⋅ 2 = 6; o dígito associado a 93 é o 4 pois 9 ⋅ 3 = 27, 2 ⋅ 7 = 14, 1 ⋅ 4 = 4. Encontre todos os números de dois algarismos aos que se associa o dígito 8. PROBLEMA 2

Encontre números primos p, q, r para os quais 2 3 200p q r+ + = . Diga todas as possibilidades. Obs: Lembre-se que o número 1 não é primo. PROBLEMA 3 Temos 26 cartões e cada um tem escrito um número. Há dois com o número 1, dois com o número 2, dois com o 3, e assim por diante até dois com o 12 e dois com o 13. Deve-se distribuir os 26 cartões em pilhas de maneira que sejam cumpridas as duas condições a seguir:

• Se dois cartões têm o mesmo número estão na mesma pilha. • Nenhuma pilha contém um cartão cujo número é igual à soma dos números

de dois cartões dessa mesma pilha. Determine qual é o número mínimo de pilhas que temos que formar. Dê um exemplo com a distribuição dos cartões para esse número de pilhas e justifique por quê é impossível ter menos pilhas. PROBLEMA 4 Três circunferências são tangentes entre si, tal como mostramos na figura. A região do círculo exterior que não está coberta pelos dois círculos interiores tem área igual a 2π. Determine o comprimento do segmento PQ.

QP

Page 4: Eureka 2010

Sociedade Brasileira de Matemática

EUREKA! N°31, 2010

4

PROBLEMA 5 Pelas linhas de um tabuleiro quadriculado formado por 55 linhas horizontais e 45 linhas verticais caminha uma formiga. Queremos pintar alguns trechos de linhas para que a formiga possa ir de qualquer cruzamento até outro cruzamento qualquer, caminhando exclusivamente pelos trechos pintados. Se a distância entre linhas consecutivas é de 10 cm, qual é a menor quantidade possível de centímetros que deverão ser pintados?

SEGUNDO NÍVEL PROBLEMA 1 Inicialmente no quadro está escrito o número 1. Em cada passo, apaga-se o número do quadro e se escreve outro, que é obtido aplicando alguma das seguintes operações:

• Operação A: Multiplicar o número escrito no quadro por 12

.

• Operação B: Trocar o número escrito no quadrado pela diferença entre 1 e ele.

Por exemplo, se no quadro está escrito o número 38

podemos substituí-lo por

1 3 32 8 16

⋅ = ou por 3 5

18 8

− = .

Encontre uma sequência de passos ao fim dos quais o número do quadro seja

2009

20092

.

PROBLEMA 2 Seja ABCD um quadrilátero convexo tal que o triângulo ABD é equilátero e o

triângulo BCD é isósceles, com µ o90C = . Se E é o ponto médio do lado AD,

determine a medida do ângulo µCED . PROBLEMA 3 Na seguinte soma: 1 + 2 + 3 + 4 + 5 + 6, se suprimirmos os dois primeiros sinais de “+” obtemos a nova soma 123 + 4 + 5 + 6 = 138. Suprimindo três sinais de “+” podemos obter 1 + 23 + 456 = 480. Consideremos agora a soma 1 + 2 + 3 + 4 + 5 + 6 + 7 + 8 + 9 + 10 + 11 + 12 + 13, na qual serão suprimidos alguns sinais de “+”. Quais são os três menores múltiplos de 100 que podemos obter desta forma?

Page 5: Eureka 2010

Sociedade Brasileira de Matemática

EUREKA! N°31, 2010

5

PROBLEMA 4 Cada casa de um tabuleiro 5 × 5 é pintada de vermelho ou de azul, de tal forma que seja cumprida a seguinte condição: “Para quaisquer duas filas e duas colunas, das 4 casas que estão em suas interseções, há 4, 2 ou 0 pintadas de vermelho.” De quantas formas podemos pintar o tabuleiro? PROBLEMA 5 Um jogo de paciência se inicia com 25 cartas em fila. Algumas estão viradas para cima, e outras viradas para baixo. Em cada movimento devemos escolher uma carta que esteja virada para cima, retirá-la e virar as cartas vizinhas à que foi retirada (se houver). Ganha-se o jogo de paciência quando conseguimos, repetindo este movimento, retirar as 25 cartas da mesa. Se inicialmente há n cartas viradas para cima, encontre todos os valores de n para os quais se pode ganhar o jogo. Explique a estratégia vencedora, independentemente da localização inicial das cartas viradas para cima, e justifique por quê é impossível ganhar para os outros valores de n. Duas cartas são vizinhas quando uma está imediatamente ao lado de outra, à direita ou à esquerda. Por exemplo: a carta marcada com A tem duas cartas vizinhas e a marcada com B apenas uma. Depois de retirar uma carta fica um espaço, de modo que a marcada com C tem unicamente uma carta vizinha, e a marcada com D não tem nenhuma.

B A C D

Page 6: Eureka 2010

Sociedade Brasileira de Matemática

EUREKA! N°31, 2010

6

RESULTADO BRASILEIRO

2009: Nível 1 (até 13 anos)

Nome Cidade - Estado Pontos Prêmio Luis Fernando Veronese Trivelatto Cascavel - PR 30 Medalha de Ouro Lucas Carvalho Daher Anápolis - GO 29 Medalha de Prata Guilherme Renato Martins Unzer São Paulo - SP 27 Medalha de Prata Elias Brito Oliveira Brasília - DF 26 Medalha de Bronze Lucas Cardoso Zuccolo São Paulo - SP 25 Medalha de Bronze Gustavo Lima Lopes Barra de São Fco. - ES 24 Medalha de Bronze Rafael Rodrigues Rocha de Melo Fortaleza - CE 24 Medalha de Bronze Igor Albuquerque Araújo Rio de Janeiro - RJ 23 Menção Honrosa Liara Guinsberg São Paulo - SP 23 Certificado Fellipe Sebastiam da Silva Paranhos Pereira Rio de Janeiro - RJ 23 Certificado

2009: Nível 2 (até 15 anos) Nome Cidade - Estado Pontos Prêmio João Lucas Camelo Sá Fortaleza - CE 50 Medalha de Ouro César Ilharco Magalhães Barbacena - MG 42 Medalha de Prata Bruno Silva Mucciaccia Vitória - ES 40 Medalha de Prata Daniel dos Santos Bossle Porto Alegre - RS 40 Medalha de Bronze Gustavo Haddad Francisco e Sampaio Braga S.J. dos Campos - SP 38 Medalha de Bronze Otávio Araújo de Aguiar Fortaleza - CE 38 Medalha de Bronze Gabriel Militão Vinhas Lopes Fortaleza - CE 35 Medalha de Bronze Lara Timbó Araújo Fortaleza - CE 33 Menção Honrosa Artur A. Scussel Fortaleza - CE 32 Menção Honrosa Bruno Ferri de Moraes São Paulo - SP 31 Menção Honrosa

Page 7: Eureka 2010

Sociedade Brasileira de Matemática

EUREKA! N°31, 2010

7

XX OLIMPÍADA DE MATEMÁTICA DO CONE SUL Enunciados e resultado brasileiro

A XX Olimpíada de Matemática do Cone Sul foi realizada na cidade de

Mar del Plata, Argentina entre os dias 16 e 17 de abril de 2009. A equipe foi liderada pelos professores Pablo Rodrigo Ganassim, de São Paulo – SP e Alex Correa Abreu, de Niterói – RJ.

RESULTADOS DA EQUIPE BRASILEIRA BRA1 Deborah Barbosa Alves Medalha de Prata BRA2 Gabriel Militão Vinhas Lopes Medalha de Bronze BRA3 Matheus Barros de Paula Medalha de Prata BRA4 Matheus Secco Torres da Silva Medalha de Bronze PROBLEMA 1 Os quatro círculos da figura determinam 10 regiões limitadas. Nessas regiões são escritos 10 números inteiros positivos distintos cuja soma é 100, um número em cada região. A soma dos números contidos em cada círculo é igual a S (a mesma para os quatro círculos). Determine o maior e o menor valor possível de S.

PROBLEMA 2 Um corchete é composto por três segmentos de comprimento 1, que formam dois ângulos retos como mostra a figura.

É dado um quadrado de lado n dividido em n2 quadradinhos de lado 1 por meio de retas paralelas aos seus lados. Corchetes são colocados sobre esse quadrado de modo que cada segmento de um corchete cubra um lado de algum quadradinho. Dois segmentos de corchete não podem ficar sobrepostos. Determine todos os valores de n para os quais é possível cobrir os lados dos n2 quadradinhos.

Page 8: Eureka 2010

Sociedade Brasileira de Matemática

EUREKA! N°31, 2010

8

PROBLEMA 3 Sejam A, B e C três pontos tais que B é ponto médio do segmento AC e seja P um ponto tal que ∠PBC = 60º. São construídos o triângulo equilátero PCQ tal que B e Q estão em semiplanos diferentes em relação a PC, e o triângulo equilátero APR tal que B e R estão no mesmo semiplano em relação a AP. Seja X o ponto de interseção das retas BQ e PC; seja Y o ponto de interseção das retas BR e AP. Demonstre que XY e AC são paralelos. PROBLEMA 4 Ana e Beto jogam em um tabuleiro de 11 linhas e 9 colunas. Primeiro Ana divide o tabuleiro em 33 zonas. Cada zona é formada por 3 casas adjacentes alinhadas vertical ou horizontalmente, como mostra a figura.

Depois, Beto escreve em cada casa um dos números 0, 1, 2, 3, 4, 5, de modo que a soma dos números de cada zona seja igual a 5. Beto ganha se a soma dos números escritos em cada uma das 9 colunas do tabuleiro é um número primo; caso contrário, Ana ganha. Demonstre que Beto tem uma estratégia vencedora. PROBLEMA 5 Dada uma sequência S de 1001 números reais positivos não necessariamente distintos, e dado um conjunto A de números inteiros positivos distintos, a operação permitida é: eleger um k ∈ A (k = 1001), selecionar k números de S, calcular a média dos k números (média aritmética) e substituir cada um dos k números selecionados por essa média. Se A é um conjunto tal que para cada S pode-se conseguir, mediante uma sucessão de operações permitidas, que os números sejam todos iguais, determine o menor valor possível do maior elemento de A. PROBLEMA 6 Pablo tem uma certa quantidade de retângulos cujas áreas somam 3 e cujos lados são todos menores ou iguais a 1. Demonstre que com esses retângulos é possível cobrir um quadrado de lado 1 de modo que os lados dos retângulos sejam paralelos aos lados do quadrado. Nota: Os retângulos podem estar sobrepostos e podem sair parcialmente do quadrado.

Page 9: Eureka 2010

Sociedade Brasileira de Matemática

EUREKA! N°31, 2010

9

L OLIMPÍADA INTERNACIONAL DE MATEMÁTICA (IMO) Enunciados e resultado Brasileiro

A L Olimpíada Internacional de Matemática (IMO) foi realizada na cidade

de Bremen, Alemanha entre os dias 14 e 21 de julho de 2009. A equipe foi liderada pelos professores Carlos Yuzo Shine, de São Paulo – SP e Ralph Costa Teixeira, de Niterói – RJ.

RESULTADOS DA EQUIPE BRASILEIRA BRA1 Henrique Ponde de Oliveira Pinto Medalha de Ouro BRA2 Renan Henrique Finder Medalha de Prata BRA3 Marcelo Tadeu de Sá Oliveira Sales Medalha de Prata BRA4 Matheus Secco Torres da Silva Medalha de Prata BRA5 Marco Antonio Lopes Pedroso Medalha de Bronze BRA6 Davi Lopes Alves de Medeiros Medalha de Bronze PRIMEIRO DIA

PROBLEMA 1 Seja n um inteiro positivo e sejam ( )1 2ka ,...,a k ≥ inteiros distintos do conjunto

{ }1,...,n tais que n divide ( )1 1i ia a ,+ − para 1 1i ,...,k .= − Demonstre que n não

divide ( )1 1ka a .− PROBLEMA 2 Seja ABC um triângulo cujo circuncentro é O. Sejam P e Q pontos interiores dos lados CA e AB, respectivamente. Sejam K, L, e M os pontos médios dos segmentos BP, CQ e PQ, respectivamente, e Γ a circunferência que passa por K, L, e M. Suponha que a recta PQ é tangente à circunferência Γ . Demonstre que OP = OQ. PROBLEMA 3 Seja 1 2 3s ,s ,s ,... uma sucessão estritamente crescente de inteiros positivos tal que as subsucessões

1 2 3s s ss ,s ,s ,... e 1 2 31 1 1s s ss ,s ,s ,...+ + +

são ambas progressões aritméticas. Demonstre que a sucessão 1 2 3s ,s ,s ,... também é uma progressão aritmética.

Page 10: Eureka 2010

Sociedade Brasileira de Matemática

EUREKA! N°31, 2010

10

SEGUNDO DIA

PROBLEMA 4 Seja ABC um triângulo com AB = AC. As bissectrizes dos ângulos CAB∠ e

ABC∠ intersectam os lados BC e CA em D e E, respectivamente. Seja K o incentro do triângulo ADC. Suponha que µ 45BEK .= ° Determine todos os possíveis valores de µC AB.

PROBLEMA 5 Determine todas as funções f do conjunto dos inteiros positivos no conjunto dos inteiros positivos tais que, para todos os inteiros positivos a e b, existe um triângulo não degenerado cujos lados medem,

a, f (b) e f (b + f(a) – 1). (Um triângulo é não degenerado se os seus vértices não são colineares). PROBLEMA 6 Sejam 1 2 na ,a ,...,a inteiros positivos distintos e M um conjunto de n – 1 inteiros positivos que não contém o número 1 2 ns a a ... a .= + + + Um gafanhoto pretende saltar ao longo da recta real. Ele começa no ponto 0 e dá n saltos para a direita de comprimentos 1 2 na ,a ,...,a ,em alguma ordem. Prove que essa ordem pode ser escolhida de modo que o gafanhoto nunca caia num ponto de M.

Page 11: Eureka 2010

Sociedade Brasileira de Matemática

EUREKA! N°31, 2010

11

XXIV OLIMPÍADA IBEROAMERICANA DE MATEMÁTICA Enunciados e resultado Brasileiro

A XXIV Olimpíada Iberoamericana de Matemática foi realizada na cidade

de Santiago de Queretaro, México no período de 17 a 27 de setembro de 2009. A equipe brasileria foi liderada pelos professores Onofre Campos, de Fortaleza – CE e Luzinalva Miranda de Amorim, de Salvador – BA.

RESULTADOS DA EQUIPE BRASILEIRA

BRA1 Renan Henrique Finder Medalha de Ouro BRA2 Matheus Secco Torres da Silva Medalha de Ouro BRA3 Marco Antonio Lopes Pedroso Medalha de Prata BRA4 Marcelo Tadeu de Sá Oliveira Sales Medalha de Prata PRIMEIRO DIA PROBLEMA 1 Seja n um natural maior que 2. Suponhamos que n ilhas estejam localizadas ao redor de um círculo e que entre cada duas ilhas vizinhas haja duas pontes, como na figura:

x1 x2

xn

xn – 1 xj

x3

Partindo da ilha 1x , de quantas maneiras se podem percorrer as 2n pontes passando por cada ponte exatamente uma vez?

Page 12: Eureka 2010

Sociedade Brasileira de Matemática

EUREKA! N°31, 2010

12

PROBLEMA 2 Para cada inteiro positivo n definimos na n m,= + onde m é o maior inteiro tal que

22 2m nn .≤ Determinar quais inteiros positivos não aparecem na sequência na .

PROBLEMA 3 Sejam 1C e 2C duas circunferências de centros 1O e 2O , com o mesmo raio, que se intersectam em A e B. Seja P um ponto sobre o arco AB de 2C que está dentro de

1C . A reta AP intersecta 1C em C, a reta CB intersecta 2C em D e a bissetriz de CAD∠ intersecta 1C em E e 2C em L. Seja F o simétrico do ponto D em relação

ao ponto médio de PE. Demonstrar que existe um ponto X que satisfaz 30XFL XDC∠ = ∠ = ° e 1 2CX O O .=

SEGUNDO DIA PROBLEMA 4 Seja ABC um triângulo com AB AC.≠ Sejam I o incentro de ABC e P o outro ponto de interseção da bissetriz externa do ângulo A com o circuncírculo de ABC. A reta PI intersecta o circuncírculo de ABC no ponto J. Demonstrar que os circuncírculos dos triângulos JIB e JIC são tangentes às retas IC e IB, respectivamente. PROBLEMA 5 A sequência na está definida por

1 1a ,= 2 1k ka a= + e 2 12

1k

k

a ,a+ = para todo inteiro 1k .≥

Demonstrar que todo número racional positivo aparece exatamente uma vez nesta sequência. PROBLEMA 6 Ao redor de uma circunferência marcam-se 6000 pontos, cada um dos quais se pinta com uma de 10 cores dadas, de modo que entre quaisquer 100 pontos consecutivos sempre figuram as 10 cores. Achar o menor inteiro k com a seguinte propriedade: para toda coloração deste tipo existem k pontos consecutivos nos quais se encontram as 10 cores.

Page 13: Eureka 2010

Sociedade Brasileira de Matemática

EUREKA! N°31, 2010

13

PAR OU ÍMPAR? EIS A QUESTÃO Einstien do Nascimento Jr e Samuel Barbosa Feitosa

Paridade

Quando duas pessoas estão indecisas sobre uma escolha, muitas vezes elas utilizam uma brincadeira chamada Par ou ímpar para se decidirem. Por trás desse simples critério, podem se resolver problemas que parecem ser bastante complicados. Dizemos que um númreo tem paridade par se ele for par, e paridade ímpar, se ele for ímpar. Observar a paridade de um número é algo bem simples mas com aplicações fantásticas em problemas de olimpíadas. Vejamos um exemplo: Paridade como invariante Vamos começar com um problema bastante famoso que já foi utilizado até em entrevistas para grandes empresas de computação. Problema 1. 100 pessoas são postas em uma fila e cada uma delas recebe um chapéu, que pode ser preto ou branco. Cada pessoa só consegue ver os chapéus das pessoas que estão a sua frente. É pedido que cada uma delas tente adivinhar a cor do seu chapéu. Qual o máximo número de acertos que se pode garantir, dado que as pessoas podem combinar uma estratégia antes de recebê-los. Solução: Facilmente consegue-se 50 acertos. Podemos dividir as pessoas em pares: (100,99), (98, 97),...(2, 1) e assim o maior número de cada par falar a cor da pessoa da frente. Que apenas precisa repeti-lo, para garantir 1 acerto por par. De uma forma um pouco mais elaborada, se garante 66 acertos. Separando em trios: (100, 99, 98),...(4, 3, 2). O maior número de cada trio pode falar BRANCO caso os dois da sua frente tenham a mesma cor e PRETO, caso as cores sejam distintas. Assim, após o maior número falar, o número do meio pode acertar sua cor e em seguida, o primeiro do trio pode acertar a dele. Curiosamente esse número pode chegar a 99 acertos utilizando esse poderoso argumento que é a paridade. Notemos que ninguém sabe a cor do último da fila. Então não importa a estratégia de ordem das pessoas, nenhuma informação pode ser obtida para esse chapéu. O que não ocorre com os 99 chapéus restantes. Note ainda que a diferença de conhecimentos entre a pessoa e a pessoa que encontra atrás dela é apenas o seu chapéu. Então, basta seguir a estratégia: As cores serão faladas das pessoas de trás para as da frente. E a última pessoa vai falar BRANCO caso a quantidade de chapéus brancos a sua frente seja par e PRETO, caso contrário. Como a 99ª. pessoa sabe a paridade da quantidade de chapéus brancos estritamente à sua frente, e a paridade da

Page 14: Eureka 2010

Sociedade Brasileira de Matemática

EUREKA! N°31, 2010

14

quantidade de chapéus brancos à sua frente, incluindo ela mesma, que foi informada pela 100ª. pessoa, ela acertará o seu chapéu. A 98ª., computando ambas as informações pode acertar o dela, e assim sucessivamente. Problema 2. Em cada casa de um tabuleiro de 5 x 5 está escrito 1 ou –1. Em cada passo troca-se o número de cada uma das 25 casas pelo resultado da multiplicação dos números de todas as suas casas vizinhas. Inicialmente se tem o tabuleiro da figura. Mostre como fica o tabuleiro ao final de 2004 passos. Observação: Duas casas são vizinhas se tiverem um lado em comum.

1 1 –1 1 1 1 1 1 1 1 1 1 1 1 1 1 1 1 1 1 1 1 1 1 1

Dica: Muitas vezes, quando não se tem ideia de como será a solução de uma questão, pode-se obter várias pistas fazendo alguns casos iniciais do enunciado, esperando observar algum padrão. Meus números da sorte são 5 e 9. Ao achar um padrão repetitivo, basta analisar em que caso cairá o número 2004. Problema 3. Em cada um dos 10 degraus de uma escada existe uma rã. Cada rã pode, de um pulo, colocar-se em outro degrau, mas quando uma rã faz isso, ao mesmo tempo, uma outra rã pulará a mesma quantidade de degraus em sentido contrário: uma sobe e outra desce. Conseguirão as rãs colocar-se todas juntas num mesmo degrau? Solução: Uma maneira muito utilizada para atacar problemas onde é dada uma condição inicial e um conjunto de operações para manipulá-la é tentar procurar o que não muda, independentemente dos movimentos que utilizamos. Note que se uma rã vai de um degrau par para um ímpar (muda de paridade), a outra rã que se movimenta com ela também pulará um número ímpar de degraus, mudando também a paridade. Caso a primeira não mude, a sua parceira de movimento também permanecerá num degrau de mesma paridade. UM INVARIANTE: Paridade da quantidade de rãs em degraus de número par (comprove testando os movimentos possíveis). Como na posição inicial há 5 rãs nos degraus de posição par e na posição final há ou dez ou zero rãs nos degraus de posição par, a posição final NÃO pode ser obtida da posição inicial apenas fazendo essas operações permitidas.

Page 15: Eureka 2010

Sociedade Brasileira de Matemática

EUREKA! N°31, 2010

15

Essa estratégia de invariantes é utilizada principalmente para provar a impossibilidade de ocorrer algum evento. Definiremos uma peça príncipe (que não existe no jogo de xadrez) como uma que só pode andar na horizontal e vertical, uma casa por vez. Um jeito comum de fazer notações em um tabuleiro de xadrez é nomear as colunas da esquerda para a direita de a a h e as linhas de baixo para cima de 1 a 8 tomando o referencial da pessoa que joga com as casas brancas. Problema 4. Sobre um tabuleiro de xadrez, um príncipe começa do quadrado a1 e retorna após fazer alguns movimentos. Mostre que o príncipe fez um número par de movimentos. Solução: veja que em cada movimento, o príncipe muda para uma casa de cor oposta. Como a casa a1 é preta, após um número ímpar de movimentos o príncipe estará numa casa da cor branca. Para ele ter retornado até a casa preta do início, ele deverá ter feito um número par de movimentos. Problema 5. Pode um príncipe começar do quadrado a1 de um tabuleiro de xadrez, ir até o quadrado h8, visitando cada um dos quadrados restantes exatamente uma vez? Solução: A resposta é não. Em cada movimento, o príncipe pula para um quadrado da cor oposta. Como o príncipe tem que fazer 63 movimentos, o último movimento irá deixá-lo em uma casa da cor oposta a cor de a1. Entretanto, a1 e h8 têm a mesma cor. Isto é um absurdo. O último problema nos conduz a um tipo muito importante de demonstração: prova por absurdo. Suponha que lhe perguntaram se é possível somar cinco números ímpares e obter o número 100. Após algumas tentativas você começa a desconfiar que isto não é possúvel. Mas como provar que não é possível? Se realmente fosse possível somar 5 números ímpares e obter 100 o que aconteceria? Como a soma de cinco números ímpares é sempre ímpar obteríamos que 100 é um número ímpar. Mas 100 não é ímpar! Logo não é possível existirem tais 5 números. Para provar que algo não é possível, basta supormos que é possível e chegarmos a um absurdo. Problema 6. Uma linha poligonal fechada é composta por 11 segmentos. Pode uma reta (não contendo um vértice da linha poligonal) intersectar cada um desses segmentos?

Page 16: Eureka 2010

Sociedade Brasileira de Matemática

EUREKA! N°31, 2010

16

Problema 7. Três bolas de gude, A, B, e C estão no chão. Um movimento permitido é passar uma bola entre as outras duas. É possível, após 25 movimentos, que todas as bolas estejam nas suas posições originais? Dica: Que horas são? (Sentidos horário e anti-horário...) Problema 8. Kátia e seus amigos estão em um círculo. Sabemos qua ambos os vizinhos de cada criança são do mesmo sexo. Determine o número de garotas sabendo que existem 5 garotos no círculo. Dica: Comece a analisar por um vizinho da Kátia. Problema 9. (Rússia 1970) O rei Luis estava desconfiado de alguns de seus cortesãos. Ele fez uma lista completa de cada um dos seus cortesãos e disse a cada um deles para espionar um outro cortesão. O primeiro da lista foi espionar o cortesão que estava espionando o segundo da lista, o segundo da lista foi espionar o cortesão que estava espionando o terceiro da lista, e assim sucessivamente; o penúltimo foi espionar o cortesão que estava espionando o último e o último foi espionar o cortesão que estava espionando o primeiro. Prove que o rei Luis tinha um número ímpar de cortesãos. Solução. Seja n o número de cortesão da lista e suponha que n é par. Coloque-os sentados ao redor de uma mesa circular de modo que cada um esteja espionando o seu vizinho da direita.

Y 1

X

2 2n

O cortesão 1 espia o cortesão X que espia o cortesão 2, o cortesão 2 espia o

cortesão Z que espia o cortesão 3, e assim sucessivamente até que o cortesão 2n

espia o cortesão Y que espia o cortesão 12n

.+ Como os números 1, 2, 3,...,n devem

Page 17: Eureka 2010

Sociedade Brasileira de Matemática

EUREKA! N°31, 2010

17

se alternar sobre o círculo, concluímos que o cortesão 12n

+ é igual ao cortesão 1,

ou seja, n = 0. Esse absurdo mostra que n é ímpar. Problema 10. Um cubo 1 1 1× × está posicionado em um plano quadriculado de modo que uma de suas faces coincide com um dos quadradinhos do plano. Em cada movimento podemos “tombar” o cubo por uma de suas arestas, fazendo coincidir uma face, que tinha essa aresta, com um dos quadradinhos do plano. È possível fazer o cubo voltar a sua posição inicial após 2005 movimentos? Dica: Alguém aí joga xadrez? Paridade e Contagens Nesta seção, abordaremos duas ideias muito simples: 1. Se contamos os elementos de um conjunto de duas maneiras diferentes, os valores obtidos devem ter a mesma paridade (Porque são iguais!) 2. Se os elementos de um conjunto podem ser pareados então o conjunto tem uma quantidade par de elementos. Problema 11. Em Brasilândia existem apenas 9 casas muito distantes entre si. È possível que cada casa esteja ligada a exatamente 7 outras casas através de estradas? Solução: Não é possível. Some a quantidade de estradas que saem de cada casa. Bem, facilmente obtemos 9 7× estradas. Como cada estrada liga duas cidades, a contagem que fizemos contou cada estrada duas vezes. Logo o número obtido teria que ser par. Você deve ter ficado com uma pulga atrás da orelha. Será que cada casa ligada a exatamente 7 outras foi realmente crucial? É possível revolvermos o problema anterior com um eneunciado mais geral: Problema 12. Prove que numa festa com n pessoas, o número de pessoas que conhecem um número ímpar de outras pessoas na festa á par. Solução: Numere as pessoas de 1 até n e denote por id o número de amigos da pessoa i. Imagine que existe um fio entre duas pessoas que se conhecem. Se E denota a quantidade de fios, temos

1 2 2nd d ... d E,+ + + =

Page 18: Eureka 2010

Sociedade Brasileira de Matemática

EUREKA! N°31, 2010

18

pois cada fio é contado duas vezes, um para cada ponta. Como o lado direito é par, no lado esquerdo devemos ter uma quantidade par de números ímpares. Problema 13. (Olimpíada de Maio 2000) O conjunto {1, 2, 3, 4} pode ser dividido em dois subconjuntos { }1 4A ,= e { }3 2B ,= sem elementos comuns e tais que a soma dos elementos de A seja igual à soma dos elementos de B. Essa divisão é impossível para o conjunto { }1 2 3 4 5, , , , e também para o conjunto { }1 2 3 4 5 6, , , , , . Determine todos os valores de n para os quais o conjunto dos primeiros n números naturais pode ser dividido em dois subconjuntos sem elementos comuns tais que a soma dos elementos de cada subconjunto seja a mesma. Solução. Como a soma dos elementos de A deve ser igual à soma dos elementos de B, a soma dos números do conjunto { }1 2 3, , ,...,n deve ser o dobro da soma dos elementos de A, ou seja, deve ser um número par. Você já deve saber que

( )11 2 3

2n n

... n .+

+ + + + =

Você não sabia disso? Não fique aí parado! Tente descobrir porque isso é verdade!

Veja que ( )1

2n n +

é par se ( )1n n + é múltiplo de 4. Como estamos interessados no

resto na divisão por 4 de algum número, talvez seja interessante procurar quais os possíveis restos de n na divisão por 4. Podemos escrever n na forma 4n q r= + onde 0 1 2r , ,= ou 3. Mãos à obra!

1. Se 4n q= então ( ) ( )1

2 4 12

n nq q

+= + é par.

2. Se 4 1n q= + então ( ) ( )( )1

2 1 4 12

n nq q

+= + + é ímpar.

3. Se 4 2n q= + então ( ) ( )( )1

2 1 4 12

n nq q

+= + + é ímpar.

4. Se 4 3n q= + então ( ) ( )( )1

2 2 4 32

n nq q

+= + + é par.

Podemos concluir que n deve ser da forma 4q ou 4q + 3. Acabou? Não! Precisamos construir EXEMPLOS para cada uma dessas possibilidades mostrando que realmente esses valores satisfazem as condições do problema. Para n = 4q, considere os conjuntos

Page 19: Eureka 2010

Sociedade Brasileira de Matemática

EUREKA! N°31, 2010

19

( ) ( ) ( ) ( ){ }1 4 5 8 9 12 4 3 4A , , , , , ,..., q , q .= −

( ) ( ) ( ) ( ){ }2 3 6 7 10 11 4 2 4 1B , , , , , ,..., q , q .= − −

Para n = 4q + 3, considere os conjuntos ( ) ( ) ( ) ( ){ } ( ){ }4 7 8 11 12 15 4 4 3 1 2A , , , , , ,..., q, q , .= + ∪

( ) ( ) ( ) ( ){ } ( ){ }5 6 9 10 13 14 4 1 4 2 3B , , , , , ,..., q , q .= + + ∪

Note que os conjuntos foram divididos em parêntesis. Cada parêntese de A possui correspondente em B com a mesma soma, facilitando a construção de um exemplo generalizado. Problema 14. Podemos desenhar uma linha poligonal fechada feita por 9 segmentos de reta, cada um deles intersectando exatamente outro segmento? Solução. Se tal construção é possível, então todos os segmentos podem ser agrupados em pares de segmentos intersectantes. Mas o número de segmentos é ímpar! Absurdo! Os próximos dois problemas tratam de dominós. Um dominó consiste de um tabuleiro 1 x 2 com pontos em cada casinha. A quantidade de pontos varia de 0 até 6. Então, o número total de dominós distintos é 28. Problema 15. Todos os dominós são arranjados em uma cadeia de duas pontas (a quantidade de pontos na extremidade de dois dominós consecutivos é a mesma). Se em uma ponta existe o número 5, qual é o número de outra ponta? Problema 16. Em um conjunto de dominós, descartamos todos aqueles que possuem pelo menos uma casinha vazia. É possível arranjarmos todos os restantes em uma cadeia? Problema 17. (Eslovênia 1992) Prove que para quaisquer inteiros positivos

1 2 na ,a ,...,a o número:

1 2 2 3 1na a a a ... a a− + − + + − é par. Observação: x y− é chamado de valor absoluto da diferença entre x e y e denota o máximo entre x – y e y – x. Na reta real, ele representa a distância entre os números x e y.

Page 20: Eureka 2010

Sociedade Brasileira de Matemática

EUREKA! N°31, 2010

20

Solução: Perceba que x y x y− = ± ± para alguma escolha de sinais. Então a soma total é

1 2 2 3 1na a a a ... a a .± ± ± ± ± ± ± Como cada número ia aparece duas vezes, basta mostrarmos que cada uma das expressões 1 ia a± ± é par para qualquer escolha de sinais. Vejamos os casos: 1. 2i i i i ia a a a a± ± = + + = é par. 2. 0i i i ia a a a± ± = − + = é par. 3. 0i i i ia a a a± ± = + − = é par. 4. 2i i i i ia a a a a± ± = − − = − é par. 1.3 Miscelânia Problema 18. Podemos trocar uma nota de 25 reais usando dez notas que podem assumir os valores 1, 3, 5? Solução. Não. Como a soma de um número par de números ímpares é par, a soma dos valores dessas 10 notas só pode ser um número par. Mas 25 é ímpar. Problema 19. Peter comprou um caderno com 96 folhas, e numerou com os números de 1 até 192. Victor rasgou 25 folhas consecutivas do caderno, e adicionou os 50 números. Victor pode ter obtido o número 1990 como resultado da soma?

Problema 20. Prove que a igualdade 1 1 1 1 1 1

1a b c d e f

+ + + + + = não admite soluções

com todos os números sendo ímpares. Dica: Faça o produto dos denominadores. Problema 21. O produto de 21 inteiros é igual a 1. Mostre que sua soma não pode ser zero. Dica: compare as quantidades de números positivos e negativos. Problema 22. Três gafanhotos estão brincando ao longo da uma linha. Na sua vez, cada gafanhoto pode pular sobre um outro gafanhoto, mas não sobre os outros dois. Eles podem retornar para suas posições iniciais após 1991 movimentos?

Page 21: Eureka 2010

Sociedade Brasileira de Matemática

EUREKA! N°31, 2010

21

Solução. Sejam A, B, C os três gafanhotos. Estaremos interessados apenas na ordem em que os gafanhotos se dispõem ao longo da reta, digamos que inicialmente eles estão na ordem (A, B, C). Podemos fazer os seguintes movimentos:

( ) ( ) ( ) ( )1 2 3 4

A,B,C B,A,C B,C,A C,B,A ...→ → → →

Em cada passo, disponha as letras A, B e C em um círculo (como mostra a figura) e leia a palavra ABC. Percebeu alguma coisa? Antes de efetuarmos nosso primeiro movimetno, a leitura estava no sentido “horário” e logo em seguida passou para o sentido “anti-horário”. Como cada movimento alternar os sentidos, após 1991 movimentos estaremos em um sentido diferente do original. Logo, não é possível retornarmos para a posição original.

A B

C C B A

Observação: Compare com o problema 7. Problema 23. Os números de 1 até 10 são escritos em uma linha. Podemos colocar os sinais + e – entre eles de modo que o resultado da expressão resultante seja 0? Solução: Não é possível. Perceba que quando escolhemos um número para trocarmos de sinal, por exemplo, de + para –, a soma total varia o dobro do número escolhido, ou seja, a paridade da soma não muda. Basta ver agora que 1 2 10 55...+ + + = não tem a mesma paridade que 0. Um INVARIANTE é a paridade da soma. Problema 24. Um gafanhoto pula ao longo de uma linha. No seu primeiro pulo, ele anda 1cm, no segundo 2cm, e assim sucessivamente. Ele pode pular para a esquerda ou para a direita. Mostre que após 1985 pulos, o gafanhoto não pode retornar ao ponto em que começou. Dica: Perceba que você pode associar aos pulos do gafanhoto um número com

Page 22: Eureka 2010

Sociedade Brasileira de Matemática

EUREKA! N°31, 2010

22

sinal (+ se o pulo é para a esquerda e – se é para a direita). Agora use o problema anterior. Problema 25. Os números 1, 2,...,1984, 1985 são escritos em um tabuleiro. A operação permitida é apagar dois números e colocar sua diferença positiva. Após algumas operações, resta apenas um único número no tabuleiro. Pode este número ser 0? Problema 26. Pode um tabuleiro 8 × 8 ser coberto com dominós 1 × 2 de modo que somente os quadrados a1 e h8 não sejam cobertos? Solução. Não é possível. Pinte o tabuleiro de preto e branco da maneira usual. Cada dominó cobre exatamente um quadrado preto e outro branco (Invariante), portanto, a quantidade de quadrados pretos cobertos é igual à quantidade de quadrados brancos cobertos. Como a1 e h8 têm a mesma cor, sobrariam 30 quadrados de uma cor e 32 de outra para serem cobertos. Absurdo! Problema 27. 45 pontos são escolhidos sobre a reta AB, todos fora do segmento de reta AB. Prove que a soma das distâncias desses pontos ao ponto A não pode ser igual à soma das distâncias ao ponto B. Solução. Sejam A e B dispostos, sem perda de generalidade como na figura abaixo. Tomemos um ponto X.

A B X

X pode estar à direita de B ou à esquerda de A. Ou ocorre: AX AB BX+ = ou BX AB AX .+ = Assim, se estivéssemos somando em x as distâncias dos 45 pontos para A e em y para B, estaríamos na verdade, só somando uma diferença de AB em x ou em y. Como 45 é ímpar, não podemos “distribuir” uma igual quantidade de AB´ s para o grupo de A e o de B. Assim, segue que não é possível. Problema 28. Um número de 17 dígitos é somado com o seu reverso (um número com os mesmo dígitos mas escritos na ordem inversa). Mostre que sua soma contém pelo menos um dígito par. Problema 29. Existem 100 soldados em um quartel. Toda noite, três deles ficam de guarda. Após um certo período de tempo, é possível que cada soldado tenha ficado de guarda exatamente uma vez com cada outro soldado?

Page 23: Eureka 2010

Sociedade Brasileira de Matemática

EUREKA! N°31, 2010

23

Solução: Suponha, por absurdo, que seja possível. Tomemos o Soldado Ryan, ele possui 99 companheiros. Suponha que ele em particular tenha conseguido ficar exatamente uma vez de pernoite com cada um dos outros. A cada dia, Ryan formava 2 duplas diferentes, que não poderiam se repetir nos dias posteriores. Caso Ryan tivesse pernoitado x vezes, a quantidade de duplas que ele teria formado seria 2x, que por hipótese, deve ser igual a 99. Chegando à conclusão que 99 é par. Absurdo! Problema 30. 25 garotos e 25 garotas estão sentados ao redor de uma mesa. Prove que é sempre possível encontrar uma pessoa tal que ambos os seus vizinhos são garotas. Solução: Suponha, por absurdo, que não necessariamente haja uma pessoa que possua duas garotas como vizinhas. Denotemos h para garoto e m para garota. Cada pessoa ou possui como vizinho 2h ou h+m. Somando todas as 50 possibilidades, devemos estar contando cada pessoa duas vezes (já que essa é vizinho de duas pessoas). Assim: ( ) ( )2 50 50x h y h m h m+ + = + onde x é o número de pessoas que têm 2 garotos como vizinhos e y é o número de pessoas que têm um garoto e uma garota. Notemos ainda que x + y = 50. Obtemos

( )50xh y m= − assim xh xm.= Mas x garotos só serão iguais a x garotas, se x for nulo. Assim, todas as pessoas têm um garoto e uma garota como vizinhos. Pintemos as 50 posições do círculo apenas de branco e preto. E analisemos apenas as pretas. Todas as pretas terão que ter vizinhos sendo um garoto e uma garota. Logo, as casas brancas serão alternadas: garoto, garota, garoto... Absurdo. Pois com 25 casas brancas, na última e na primeira brancas haverá 2 garotos. Absurdo! Segue o resultado. Problema 31. (Ucrânia 1997) Um tabuleiro é colorido de branco e preto da maneira usual, e cada casa contém um inteiro. Sabemos que a soma dos números em cada coluna e a soma dos números em cada linha é par. Mostre que a soma dos números nas casas pretas é par. Solução. Suponha sem perda de generalidade que o quadrado do canto esquerdo superior é preto. A partir desse quadrado, numere as colunas da esquerda para a direita e as linhas de cima para baixo. Some os números das colunas em posições ímpares e os números das linhas em posições pares. Perceba que cada quadrado preto do tabuleiro é contado apenas uma vez nessa soma enquanto que os quadrados brancos das linhas e colunas mencionadas são contados duas vezes. Logo, esse soma tem a mesma paridade que a soma de todos os números escritos

Page 24: Eureka 2010

Sociedade Brasileira de Matemática

EUREKA! N°31, 2010

24

nos quadrados pretos. Como a soma de quaisquer linhas e colunas é par, a soma dos números nos quadrados pretos é par.

Problema 32. Considere um tabuleiro 1998 × 2002 pintado alternadamente de preto e branco da maneira usual. Em cada casa do tabuleiro, escrevemos 0 ou 1, de modo que a quantidade de 1´s em cada linha e em cada coluna do tabuleiro é ímpar. Prove que a quantidade de 1´s escritos nas casa brancas é par. Dica: Tente imitar a solução anterior. Problema 33. (Austrália 2007) Em cada casa de um tabuleiro 2007 × 2007 escrevemos um número inteiro ímpar. Sejam iZ a soma dos números na i-ésima linha e jS a soma dos números na j-ésima coluna, para 1 2007i, j .≤ ≤ Além disso,

sejam 2 2007iA Z Z ...Z= ⋅ e 1 2 2007B S S ...S .= ⋅ Mostre que A + B não pode ser igual a zero. Problema 34. (China 1986) É possível arranjar os números 1, 1, 2, 2, 3, 3,...,1986, 1986 em fila de modo que entre quaisquer dois i´s hajam (i – 1) números? Solução: Vamos tentar fazer alguns casos pequenos. É fácil ver que não conseguimos fazer o que o enunciado pede com os números 1, 1, 2, 2 mas com os números 1, 1, 2, 2, 3, 3, 4, 4 temos um exemplo:

1º. 2º. 3º. 4º. 5º. 6º. 7º. 8º. a3 a4 a2 b3 b2 b4 a1 b1 3 4 2 3 2 4 1 1

Contados da squerda para a direita, denotemos por ia e ib as posições do primeiro e segundo número i, respectivamente. No nosso exemplo, 2 3a = e 2 5b .= Como existem i – 1 números entre dois números i´s, devemos ter i ib a i.− = Se é possível escrever os números 1, 1, 2, 2, ..., n, n em linha como no enunciado,

Page 25: Eureka 2010

Sociedade Brasileira de Matemática

EUREKA! N°31, 2010

25

obtemos: ( ) ( ) ( )1 2 1 2 1 2 2 2 1n na a ...a b b ...b ... n n n+ + + + + = + + + = +

( ) ( ) ( ) ( )1 1 2 2

11 2

2+

− + − + − = + + + =n n

n nb a b a ... b a ... n .

Somando as duas linhas,

( ) ( )1 2

5 32

2n

n nb b ...b

++ + =

Como o lado esquerdo é sempre par, a fração ( )5 3

2n n +

deve ser um inteiro par.

Isso já restringe os possíveis valores de n. Para n = 1986,

( )5 39863469

2n n +

=

é ímpar e conseqüentemente não é possível dispormos esses números em linha. Uma pergunta natural que você deve tentar responder é: para quais n tal distribuição é possível? Problema 35. É possível arranjar os números de 1 até 9 em uma sequência, de modo que exista uma quantidade ímpar de números entre 1 e 2, entre 2 e 3,..., e entre 8 e 9? Problema 36. (Rússia 1984) O número de todos os inteiros positivos de 64 dígitos sem zeros em sua representação e que são divisíveis por 101 é par ou ímpar? Solução: Precisamos bolar alguma maneira de agrupar os números em pares. Seja

64

11 110vezes

A ...= 123 repetições do número 1.

Como 1111 é múltiplo de 101 é fácil ver que A é múltiplo de 101. Para todo número de 64 dígitos 1 2 63 64a a a ...a a ,= sem zeros em sua representação decimal,

considere o seu conjugado ( )( ) ( )1 2 63 64 1 2 6410 10 10= = − − −b b b ...b b a a ... a . Nenhum

dígito de a é igual a zero, portanto, cada número 10 ia− pertence ao conjunto

{ }1 2 9, ,.., .Da equação a + b = A obtemos que a é divisível por 101 se e somente se b é divisível por 101 (lembre-se que A é múltiplo de 101). Como o único número que é igual ao seu conjugado é o número

64

55 55 vezes

...123 (que é múltiplo de 101) e os

demais números que satisfazem o enunciado podem ser pareados, concluímos que a

Page 26: Eureka 2010

Sociedade Brasileira de Matemática

EUREKA! N°31, 2010

26

quantidade procurada é ímpar. Problema 37. (Putnam 1997) Seja nB a quantidade de n – uplas ordenadas de

inteiros positivos ( )1 2 na ,a ,...,a tais que

1 2

1 1 11

n

...a a a

+ + + =

10B é par ou ímpar? Solução: Uma ideia natural é tentar agrupar as soluções em pares. Qualquer solução com 1 2a a≠ pode ser pareada com a outra solução obtida pela troca de posição entre 1a e 2a . Logo, 10B tem a mesma paridade que o número de soluções com 1 2a a .= Das soluções com 1 2a a ,= podemos parear aquelas que tem 3 4a a≠

da mesma maneira. Repetindo esse argumento com ( ) ( )5 6 7 8a ,a , a ,a e ( )9 10a ,a ,

concluímos que a paridade de 10B é a mesma do número de soluções com

5 6 7 8a a ,a a= = e 9 10a a ,= ou seja, das soluções de:

1 3 5 7 9

2 2 2 2 21.

a a a a a+ + + + =

Como anteriormente, podemos nos restrigir à quantidade de soluções com 1 3a a= e

5 7a a= , que é igual ao número de soluções da equação:

1 5 9

4 4 21.

a a a+ + =

Mais uma vez, podemos nos restringir à quantidade de soluções com 1 5a a= , que é igual ao número de soluções da equação:

1 9

8 21.

a a+ =

Agora ficou fácil! Basta contar explicitamente o número de soluções da equação anterior. Como fazer isso? Bem, ela pode ser fatorada como:

( )( )1 98 2 16a a− − =

que admite 5 soluções correspondendo às fatorações de 16 como 42 2i i−× para 0 1 2 3 4i , , , , .= Então 10B é ímpar.

Problema 38: Prove que numa festa com 2n pessoas existem duas com um número par de amigos em comum.

Page 27: Eureka 2010

Sociedade Brasileira de Matemática

EUREKA! N°31, 2010

27

Solução: Suponha que quaisquer duas pessoas tenham um número ímpar de amigos em comum e seja A um dos participantes da festa. Seja { }1 2 kM F ,F ,...,F= o conjunto dos amigos de A. Considere uma nova festa restrita apenas ao conjunto M. Como cada iF tem um número ímpar de amigos em comum com A, na nova festa, cada iF possui um número ímpar de amigos. Pelo problema 12, k deve ser par. O mesmo argumento vale para qualquer pessoa na festa e conseqüentemente todos têm um número par de amigos. Peça para cada um dos amigos de A fazerem uma lista de seus amigos diferentes de A. A soma da quantidade de nomes listados é par, pois é uma soma de uma quantidade par (igual a k) de números ímpares (cada iF possui um número ímpar de amigos diferentes de A). Agora comparemos o número de aparições de cada uma das 2 1n − pessoas diferentes de A nessas listas. Se cada uma delas aparecer em um número ímpar de listas, a soma total de todos os nomes em todas as listas seria ímpar. (Lembre-se que a soma de uma quantidade ímpar de números ímpares é ímpar!). Mas isso é uma contradição. Logo, existe uma pessoa diferente de A que aparece em um número par de listas, e portanto tem um número par de amigos em comum com A. Problema 39. Alex desenhou uma coleção de K retas no plano em posição geral (quaisquer duas retas se intersectam em um ponto e quaisquer três definem um triângulo não degenerado). Para quais valores de K é sempre possível (não importa como as retas são desenhadas) colocar um elemento do conjunto { }1 2 1, ,...,K − em cada ponto de interseção das retas de modo que em toda reta não existam números iguais. Problema 40. (Rússia) Em cada planeta de um sistema solar existe um astrônomo observando o planeta mais próximo. As distâncias entre os planetas são distintas duas a duas. Demonstre que se a quantidade de planetas é ímpar, então existe pelo menos um planeta que não é observado. Dica: Procure as cadeias de planetas que um olha para o outro que olha para o outro com mais de 2 planetas. REFERÊNCIAS [1] D. Fomin, S. Genkin e I. Itenberg, Mathematical Circles, MAS (1996). [2] C. Augusto, S. Feitosa, B. Holanda e Y. Lima, treinamento Cone Sul 2007, Fortaleza, Realce (2007). [3] P. J. Taylor, Tournament of the Towns 1980 to 1984, Australian Mathematical Trust (1993). [4] D. Fomin e A. Kirichenko, Leningrand Mathematical Olympiadas 1987-1991, MathPro Press (1994). [5] E. Wagner, Paridade, Eureka! No. 2, pp. 32-38, (1998).

Page 28: Eureka 2010

Sociedade Brasileira de Matemática

EUREKA! N°31, 2010

28

GEOMETRIA DO TRIÂNGULO: FATOS E PROBLEMAS Carlos Yuzo Shine

1. O Teorema de Miquel Começamos com o teorema em si, que é um dos vários pequenos milagres dos chamados quadriláteros completos (veja um pouco mais desses “milagres” nos exercícios!), que são os quadriláteros conhecidos unidos com as retas que contêm os lados. Isto é, um quadrilátero completo é a união de quatro retas em vez de quatro segmentos. Teorema de Miquel. Sejam a, b, c, d quatro retas coplanares, de modo que não há duas paralelas nem três concorrentes. Os circuncírculos dos quatro triângulos determinados pelas quatro retas passam por um mesmo ponto, denominado ponto de Miquel das quatro retas.

Demonstração: Seja M a intersecção dos circuncírculos de CEF e BDF na figura acima. Então

180 180MEA MEC MFC BFM BDM ADM ,∠ = ∠ = ° − ∠ = ∠ = ∠ = ° − ∠ de modo que 180MEA ADM∠ + ∠ = ° e, portanto, MDAE é inscritível. Isso quer dizer que M pertence ao circuncírculo de ADE. Analogamente, prova-se que M pertence ao

Page 29: Eureka 2010

Sociedade Brasileira de Matemática

EUREKA! N°31, 2010

29

circuncírculo de ABC. Vamos resolver, a título de exemplo, o problema 6 da olimpíada norteamericana de 2006. Exemplo 1.1. (USAMO 2006, Problema 6) Seja ABCD um quadrilátero e E e F os pontos sobre os lados AD e BC, respectivamente, tal que AE / ED BF / FC.= A semirreta FE corta as semirretas BA e CD em S e T, respectivamente. Prove que os circuncírculos dos triângulos SAE, SBF, TCF e TDE passam por um mesmo ponto. Resolução: Ao fazer a figura, você provavelmente vai notar uma certa semelhança com a figura anterior.

Queremos provar que os pontos de Miquel de ADTS e BCTS coincidem! Isso não é difícil, na verdade: seja M a intersecção dos circuncírculos de SAE e SBF. Mostraremos que os circuncírculos de TED e TFC também passam por M. Um arrastão e uma semelhança dão conta do recado: primeiro, note que

AME ASE BSF BMF e MEA MSA MSB MFB.∠ = ∠ = ∠ = ∠ ∠ = ∠ = ∠ = ∠ Então os triângulos AME e BMF são semelhantes, e da igualdade AE / ED BF / FC= os triângulos MAD e MBC são semelhantes também. Uma rápida verificação mostra que MAB e MDC também são semelhantes: de fato (pois como MAD e MBC são

Page 30: Eureka 2010

Sociedade Brasileira de Matemática

EUREKA! N°31, 2010

30

semelhantes então DMA CMB∠ = ∠ ) e AM DMBM CM

= (novamente da semelhança).

Você pode imaginar que o triângulo MAD “gira” em torno de M e, após um “acerto de escala”, é transformado no triângulo MBC. Isso é uma transformação geométrica conhecida como roto-homotetia de centro M. Assim, A é levado em B e D é elevado em C. Note que a semelhança obtida anteriormente envolve o centro de roto-homotetia M, os pontos e suas imagens na transformação. Isso na verdade sempre acontece (é uma das semelhanças automáticas). Agora podemos terminar o problema: da semelhança entre MAB e MDC, os ângulos externos MDT∠ e MAS∠ são congruentes. Como M pertence ao circuncírculo de SAE, MAS MES MET ,∠ = ∠ = ∠ ou seja, MDT MET ,∠ = ∠ o que significa que MEDT é cíclico e, portanto, M pertence ao circuncírculo de TED. Utilizando outra semelhança automática, entre MEF e MDC (pois E é levado em F!), prova-se que M pertence também ao circuncírculo de TFC. Note que se U é a interseção de AB e CD, então pelo teorema de Miquel M também pertence ao circuncírculo de STU. Então na vaerdade cinco círculos passam pelo ponto M!

A seguinte versão do teorema de Miquel também é útil: Teorema de Miquel para triângulos. Seja ABC em triângulo e D, E, F pontos sobre as retas BC, CA, AB, respectivamente. Então os circuncírculos de AEF, BFD e CDE têm um ponto em comum. Esse ponto também é chamado de ponto de Miquel.

Demonstração: Seja M a segunda interseção dos circuncírculos de AEF e BFD. Então

180CDM BFM AEM CEM .∠ = ∠ = ∠ = ° − ∠

Page 31: Eureka 2010

Sociedade Brasileira de Matemática

EUREKA! N°31, 2010

31

Exercícios: 01. Demonstre o teorema de Miquel para quadriláteros utilizando o teorema de Miquel para triângulos. 02. Seja ABCDE um pentágono convexo e F, G, H, I, J as interseções dos prolongamentos de EA, AB, AB, BC, CD, DE e DE, EA respectivamente. Prove que as segundas interseções dos circuncírculos de ABF, BCG, BCG, CDH, DEI, DEI, EAJ, e EAJ, ABF pertencem a uma mesma circunferência. 03. Considere um quadrilátero completo. Seja M o seu ponto de Miquel. Prove que: (a) os circuncentros dos quatro triângulos determinados pelo quadrilátero e M estão sobre uma mesma circunferência. (b) as projeções ortogonais de M sobre as quatro retas do quadrilátero pertencem a uma mesma reta r; além disso, M é o único ponto do plano com essa propriedade. (c) os ortocentros dos quatro triângulos pertencem a uma mesma reta s. (d) as retas r e s são paralelas, e a distância de M e r é metade da distância de M a s. 04. Seja ABCD um quadrilátero convexo e X e Y as interseções dos lados opostos AD e BC e AB e CD, respectivamente. Prove que os pontos médios de AC, BD e XY são colineares. Observação: a reta que passa pelos três pontos é a reta de Gauss do quadrilátero completo. 2. Conjugados isogonais A ideia de conjugado é fazer uma associação entre objetos. Objetos conjugados supostamente têm propriedades semelhantes. Isso é bastante comum em equações: se um número é raiz, então o conjugado também é raiz. Em geometria, também existe a ideia de conjugado. De fato, dado um triângulo, cada ponto tem um conjugado isogonal e um conjugado isotômico. Aqui, trataremos somente de conjugados isogonais. Definição 2.1. Dado um triângulo ABC, o conjugado isogonal em relação a ABC de um ponto T do plano de ABC é obtido refletindo as retas TA, TB e TC em relação às bissetrizes internas de ABC que passam por A, B, e C, respectivamente. As retas resultados são concorrentes no isogonal 1T − de T. A seguir, as linhas pontilhadas são as bissetrizes, e as cevianas cinzas são as reflexões das cevianas pretas.

Page 32: Eureka 2010

Sociedade Brasileira de Matemática

EUREKA! N°31, 2010

32

O fato de que as retas isogonais são concorrentes é extremamente importante, tanto que será enunciado novamente. Teorema fundamental dos conjugados isogonais. Dados um triângulo e três retas que passam pelos respectivos vértices e concorrem em um ponto P, as retas isogonais a elas, obtidas através da reflexão em relação à bissetriz interna correspondente, são concorrentes no conjugado isogonal 1P− de P. Demonstração Por que as cevianas cinzas são concorrentes? Isso decorre de duas aplicações do teorema de Ceva trigonométrico: primeiro com as cevianas concorrentes em T e depois, com as cevianas concorrentes em 1T ,− que formam os mesmos ângulos que as outras cevianas, porém no sentido contrário. Na verdade, pode ocorrer de as três cevianas serem paralelas. Isso ocorre se, e somente se, T está sobre o circuncírculo de ABC; nesse caso, pensamos projetivamente, ou seja, o conjugado isogonal é um ponto do infinito. 2.1 para que servem isogonais? O que é mais útil em conjugados isogonais é simplesmente que as cevianas são reflexões umas das outras em relação às bissetrizes, e isso costuma levar a algumas igualdades entre ângulos um pouco mais difíceis de obter ou mesmo de se imaginar com contas.

Page 33: Eureka 2010

Sociedade Brasileira de Matemática

EUREKA! N°31, 2010

33

Exemplos 2.1 No triângulo ABC, P e Q são pontos no interior de ABC tais que

3CBP PBQ QBA ABC /∠ = ∠ = ∠ = ∠ e 3BCP PCQ QCA ACB / .∠ = ∠ = ∠ = ∠ Sejam D e E as projeções ortogonais de P sobre AB e AC, respectivamente. Prove que AQ é perpendicular a DE. Resolução Seja PAD.θ = ∠ Então 90APD∠ = ° − θ e, como ADP∠ e AEP∠ são retos, o quadrilátero ADPE é inscritível. Logo 90AED APD .∠ = ∠ = ° − θ

Olhando a figura, note que basta provarmos que QAC .∠ = θ Aí é que entram os conjugados isogonais. Como PBC QBA∠ = ∠ e BCP QCA,∠ = ∠ os pares de retas BP;BQ e CP;CQ são simétricos entre si em relação às bissetrizes de ABC∠ e ACB,∠ respectivamente. Ou seja, P e Q são conjugados isogonais e, portanto, PAB∠ e

QAC∠ também são iguais. Logo QAC∠ = θ e o ângulo entre as retas AQ e DE é

( )180 90 90 .° − θ − − θ = ° Note que para provar o resultado na conta, bastaria repetir a demonstração do teorema fundamental dos conjugados isogonais. Mas o mais interessante é que, sabendo da existência dos conjugados isogonais, é natural pensar nessa solução. Em contraste, fazer a conta sem pensar em conjugados isogonais não parece ser tão natural. Então dá para pensar que os conjugados isogonais nos economizaram não só fazer a conta, mas mostraram onde fazer as contas relevantes.

Page 34: Eureka 2010

Sociedade Brasileira de Matemática

EUREKA! N°31, 2010

34

2.2. Conjugados isogonais dos pontos notáveis Você já deve estar familiarizado com os pontos notáveis do triângulo: o baricentro (encontro das medianas), o incentro (encontro das bissetrizes internas), o ortocentro (encontro das alturas) e o circuncentro (encontro das mediatrizes). Quais são os conjugados isogonais desses pontos? Vamos aproveitar e conhecer mais um ponto notável (mas não tão conhecido). Vamos fazer isso em ordem de dificuldade. Incentro As reflexões coincidem com as próprias bissetrizes. Logo o conjugado isogonal do incentro, que é o encontro das bissetrizes internas, é ele mesmo. O mesmo vale para os ex-incentros (encontros de duas bissetrizes externas e uma bissetriz interna e centros dos ex-incírculos, que são tangentes externamente aos lados ou seus prolongamentos). Pense sobre o assunto! Ortocentro e circuncentro A figura a seguir deve convencê-lo de que o ortocentro e o circuncentro são conjugados isogonais.

Baricentro Os isogonais das medianas são as simedianas (SImétrico + MEDIANA). O ponto de encontro das simedianas é o ponto de Lemoine, também conhecido como ponto simediano. O ponto de Lemoine é costumeiramente denotado por K. Primeiro, vamos aprender a traçá-las de modo mais prático.

Page 35: Eureka 2010

Sociedade Brasileira de Matemática

EUREKA! N°31, 2010

35

Lema: Seja D a interseção das retas tangentes ao circuncírculo do triângulo ABC por B e C. Então a reta AD contém a simediana que passa por A. Demonstração

Construa o paralelogramo ABEC. Então AD contém a mediana AM. Afirmamos que D e E são conjugados isogonais. De fato, BCE B∠ = ∠ e o ângulo entre AC e CD, pela tangência, é igual a B.∠ Assim, as retas CD e CE são conjugadas isogonais. Analogamente, BD e BE também são, e o resultado segue do teorema fundamental dos conjugados isogonais. Exercícios 05. Sejam P e Q pontos no interior do ângulo BAC∠ tais que BP CP,BQ CQ= = e 180ABP AQC .∠ + ∠ = ° Prove que BAP CAQ.∠ = ∠ 06. As retas obtidas através das reflexões da diagonal BD do quadrilátero ABCD em relação às bissetrizes de B∠ e D∠ passam pelo ponto médio de AC. Prove que as reflexões da diagonal AC do quadrilátero ABCD em relação às bissetrizes de

Page 36: Eureka 2010

Sociedade Brasileira de Matemática

EUREKA! N°31, 2010

36

A∠ e C∠ passam pelo ponto médio de BD. 07. (Prova de Seleção EUA, 2008) Seja ABC um triângulo e G o seu baricentro. O ponto P varia sobre o segmento BC. Os pontos Q e R pertencem aos lados AC e AB respectivamente, e são tais que PQ é paralelo a AB e PR é paralelo a AC. Prove que, ao variar P sobre BC, o circuncírculo de AQR passa por um ponto fixado X tal que BAG CAX .∠ = ∠ 08. (IMO 2004, Problema 5) Num quadrilátero convexo ABCD a diagonal BD não é bissetriz do ângulo ABC∠ nem do ângulo CDA.∠ Um ponto P no interior de ABCD satisfaz

PBC DBA∠ = ∠ e PDC BDA.∠ = ∠ Prove que os vértices do quadrilátero ABCD pertencem a uma mesma circunferência se, e somente se, AP = CP. 3. Triângulo Pedal Definição 3.1. Seja P um ponto no plano do triângulo ABC e D, E e F as projeções de P sobre as retas BC, CA e AB. O triângulo DEF é o triângulo pedal de P em relação ao triângulo ABC. O que triângulos pedais têm de especial? Primeiro, aparecem muitos ângulos retos, o que propicia o aparecimento de quadriláteros inscritíveis. Segundo, eles normalmente minimizam áreas. Teorema do mínimo. Dados dois triângulos T e ABC, considere todos os triângulos DEF semelhantes a T, todos na mesma ordem, com D sobre o lado BC, E sobre o lado CA e F sobre o lado AB. Dentre todos esses triângulos, o de menor área é o triângulo pedal de algum ponto P. Demonstração Não provaremos aqui a existência de um triângulo de área mínima (caso você esteja curioso, estude topologia e depois volte!). Seja DEF o triângulo de área mínima. Seja M o ponto de Miquel de ABC e DEF, e sejam P, Q e R as projeções de M sobre os lados.

Page 37: Eureka 2010

Sociedade Brasileira de Matemática

EUREKA! N°31, 2010

37

Note que o quadrilátero CPMQ é inscritível (pois MPC∠ e MQC∠ são retos), de modo que 180DME PMQ C.∠ = ∠ = ° − ∠ Portanto, PMD QME :∠ = ∠ imagine o ângulo DME∠ girando em torno de M para coincidir com PMQ;MD∠ vira MP e ME vira MQ. Analogamente, RMF QME.∠ = ∠ Portanto os triângulos PMD, QME e RMF são semelhantes e induzem uma roto-homotetia (você se lembra o que é isso?) que leva DEF a PQR. A razão de

homotetia é 1MP

,MD

≤ de modo que a área de PQR é menor ou igual à área de DEF.

Como DEF tem área mínima, os triângulos devem ser congruentes e deste modo MP = MD, ou seja, P = D. Analogamente, Q = E e R = F, de modo que DEF é o triângulo pedal de P. Exemplo 3.1. (Prova de seleção EUA, 2008) Sejam P, Q, R pontos sobre os lados BC, CA, AB de um triângulo acutângulo ABC tais que PQR é equilátero e tem área mínima entre todos tais triângulos equiláteros. Prove que a reta perpendicular a QR que passa por A, a reta perpendicular a RP que passa por B e a reta perpendicular a PQ que passa por C têm um ponto comum. Resolução Pelo teorema do mínimo, PQR é triângulo pedal de algum ponto T.

Page 38: Eureka 2010

Sociedade Brasileira de Matemática

EUREKA! N°31, 2010

38

Como os ângulos TQA∠ e TRA∠ são ambos retos, o quadrilátero AQTR é inscritível, e o seu circuncentro é o ponto médio A de AT . Assim, a reta perpendicular a QR e que passa por A, que contém a altura relativa a QR, é isogonal a AT, que contém o circuncentro, em relação ao triângulo AQR. Como os ângulos

BAC∠ e QAR∠ são iguais, a perpendicular e AT são isogonais em relação ao triângulo ABC também. O análogo para as perpendiculares a PR por B e a PQ por C. Como AT, BT e CT são concorrentes em T, seus isogonais são concorrentes no conjugado isogonal de T. A título de curiosidade, o ponto T é o primeiro ponto isodinâmico. Os dois pontos isodinâmicos (adivinhe o nome do outro ponto!) são os pontos de interseção dos círculos de Apolônio de A, B e C (que passam pelos vértices, o pé da bissetriz interna e têm centro sobre o lado oposto). Os seus conjugados isogonais são os pontos de Fermat. O primeiro ponto de Fermat é o ponto cuja soma das distâncias aos vértices é mínima (supondo que os ângulos internos do triângulo são todos menores do que 120° ). Veja [5] para aprender isso e muito, muito mais. 3.1. Voltando às simedianas Uma aplicação interessante da ideia de triângulo pedal está relacionada às simedianas. Uma outra maneira de construir as simedianas é a seguinte: Lema. Construa quadrados c cABB A , a aBCC B e b bCAA C externamente sobre os lados do triângulo ABC. Prolongue c cA B , a aB C e b bC A para obter o triângulo A B C´. Então as retas AA ,BB e CC´ concorrem no ponto simediano K de ABC.

Page 39: Eureka 2010

Sociedade Brasileira de Matemática

EUREKA! N°31, 2010

39

Demonstração Por simplicidade, sejam BC a,= CA b= e AB c= e L o encontro de AA ,BB ,CC´. Queremos provar que L = K. Primeiro, como os pares de retas AB; A B ,BC;B C´ e CA;C´ A são paralelos, os triângulos ABC e A B C´ são semelhantes. Seja k a razão de semelhança. Sejam

a bk ,k e ck as distâncias de L a BC, CA e AB, respectivamente. Das semelhanças entre LAB; LA’B, LBC; LB’C’ e LCA; LC’A’, todas de razão k,

1a b c a b c

a b c

k k k k k k kk

k a k b k c a b c k= = = ⇔ = = =

+ + + −

Isto quer dizer que as distâncias de L a cada um dos lados é proporcional aos seus comprimentos. Além disso, considerando uma semelhança prova-se que um ponto X pertence a, digamos, AL se, e somente, as distâncias de X aos lados AB e AC são proporcionais a seus comprimentos. Basta provar que a simediana por A tem a mesma propriedade. Para isso, considere a construção anterior, sendo D o mesmo ponto definido anteriormente.

Page 40: Eureka 2010

Sociedade Brasileira de Matemática

EUREKA! N°31, 2010

40

Sendo x e y as distâncias de D a AB e AC, respectivamente, considerando que o ângulo entre AB e BD é FBD ACB C∠ = ∠ = ∠ e o ângulo entre AC e CD é

DCE ABC B∠ = ∠ = ∠ (não se preocupe com triângulos obtusângulos; nesse caso, troque o ângulo obtuso por seu suplementar), nos triângulos retângulos BDF e CDE,x BDsen C= ∠ e y DCsen B.= ∠ Observando ainda que, sendo DB e DC

tangentes, DB = DC, temos x sen C AB

.y sen B AC

∠= =

∠ Logo D pertence a AL e,

consequentemente, K também. Da mesma forma provamos que K pertence a BL e CL, de modo que L = K. Assim como no teorema das bissetrizes, as simedianas dividem os lados opostos em razões interessantes.

Lema. Seja ABC um triângulo e NA uma simediana. Então 2BN AB

.CN AC

=

Demonstração

Page 41: Eureka 2010

Sociedade Brasileira de Matemática

EUREKA! N°31, 2010

41

Já provamos anteriormente que as distâncias do ponto simediano K aos lados são proporcionais a seus comprimentos. Então existe t real tal que ka = ta, kb = tb e kc =

tc. Assim, as áreas de KAB, KAC e KBC são 2 2

2 2 2 2tc c tc tb b tb

,⋅ ⋅

= = e

2

2 2ta a ta

,⋅

= respectivamente. Logo

22

2

BN área ABN área KBN área ABN área KBN área KAB c ABCN área ACN área KCN área ACN área KCN área KAC b AC

− = = = = = = − Lema. Sejam a bd ,d e cd as distâncias de um ponto P aos lados BC, CA, e AB do

triângulo ABC. Se AP corta BC em N , então c

b

c dBN.

CN b d⋅

=⋅

Demonstração Fica a cargo do leitor. 3.2 A desigualdade de Erdös-Mordell Um dos principais teoremas sobre triângulos pedais é a desigualdade de Erdös-Mordell: Desigualdade de Erdös-Mordell. Seja P um ponto no plano do triângulo ABC e

a b cd ,d ,d as distâncias de P às retas BC, CA, AB respectivamente. Então

( )2 a b cPA PB PC d d d+ + ≥ + + Demonstração Seja PA = d. “Multiplique” a figura original por d e construa triângulos semelhantes aos triângulos obtidos por PA e as projeções de P sobre AB e AC:

Page 42: Eureka 2010

Sociedade Brasileira de Matemática

EUREKA! N°31, 2010

42

Note que 90GDE∠ = ° − β e 90HDF ,∠ = ° − α de modo que G, D e H são colineares. Além disso, EGD∠ e FHD∠ são ambos retos, de modo que as retas EG e FH são paralelas. A distância entre essas duas retas é b cGH AB d AC d ,= ⋅ + ⋅ que é menor ou igual a EF d BC.= ⋅ Lembrando que d = PA, temos

b c b c

AB ACAB d AC d PA BC PA d d .

BC BC⋅ + ⋅ ≤ ⋅ ⇔ ≥ ⋅ + ⋅ Analogamente,

a c

AB BCPB d d

AC AC≥ ⋅ + ⋅

ba dABBC

dABAC

PC ⋅+⋅≥

Somando as três desigualdades e lembrando que 1

2tt

+ ≥ para todo t real positivo,

( )2a b c a b c

AB AC AB BC AC BCPA PB PC d d d d d d

AC AB BC AB BC AC + + ≥ + + + + + ≥ + +

Exemplo 3.2. (IMO 1991, Problema 4) Sejam ABC um triângulo e M um ponto interior. Mostre que pelo menos um dos ângulos MAB, MBC∠ ∠ e MCA∠ é menor ou igual a 30 .° Resolução: Sejam P, Q e R as projeções de M sobre BC, CA, e AB, respectivamente.

Pela desigualdade de Erdös-Mordell, ( )2MA MB MC MP MQ MR .+ + ≥ + + Se

Page 43: Eureka 2010

Sociedade Brasileira de Matemática

EUREKA! N°31, 2010

43

todas as razões MR MP MQ, ,

MA MB MC são maiores do que

12

,então

2 2MA MR,MB MP< < e 2MC MQ,< e ( )2MA MB MC MP MQ MR ,+ + < + +

contradição. Então uma das razões, digamos, MRMA

, é menor ou igual a 12

. Todavia,

MRsen MAB,

MA= ∠ de modo que 30MAB .∠ ≤ °

Exercícios 09. Dado um triângulo com perímetro L, seja P o perímetro de um triângulo pedal. Prove que 2L P.≥ Quando ocorre a igualdade? 10. Seja P um ponto interior ao triângulo ABC e T o seu triângulo pedal. Prove que

a área de T é igual a 2 2

24R OP

R−

vezes a área de ABC.

11. Seja G o baricentro do triângulo ABC e D, E, F as projeções ortogonais de G sobre os lados BC, CA e AB, respectivamente. Prove que

4 127 4

área DEFárea ABC

< ≤

12. Seja P um ponto qualquer no plano do triângulo ABC. As projeções de P sobre BC, CA, AB são D, E e F respectivamente. (a) Prove que as perpendiculares a EF, FD, DE por A, B, C respectivamente têm um ponto P´em comum. (b) Sejam Q e Q´ as segundas interseções de AP e AP´ com o circuncírculo de ABC, respectivamente. Prove que as retas QQ´ e BC são paralelas. 13. Os pontos X, Y e Z estão sobre BC, CA, e AB, respectivamente, e são tais que XYZ e ABC são semelhantes, nessa ordem. Prove que o circuncentro de XYZ é equidistante aos ortocentros de ABC e XYZ. 14. (Ibero 2008, Problema 5) Seja ABC um triângulo e X, Y, Z pontos interiores dos lados BC, AC, AB, respectivamente. Sejam A´, B´, C´ os circuncentros dos triângulos AZY, BXZ, CYX, respectivamente. Demonstre que

Page 44: Eureka 2010

Sociedade Brasileira de Matemática

EUREKA! N°31, 2010

44

( ) ( )4

ABCA B C´ ≥

e que a igualdade ocorre se, e somente se, as retas AA ,BB ,CC´ têm um ponto em comum. Observação: Para um triângulo qualquer RST, denotamos a sua área por (RST). 15. (OPM, 2001)

(a) Na figura acima, considere pontos 1B e 1C sobre as semirretas ABuuur

e AC,uuur

respectivamente. (i) Mostre que a soma das áreas dos paralelogramos com lados 1AB e AM e com lados 1AC e AM é igual à área do paralelogramo tal que um de seus lados é 1 1B C e o outro é paralelo e igual a AM. (ii) Tomando 1AB AC= e 1AC AB,= conclua que AB v AC w BC x⋅ + ⋅ ≤ ⋅

(b) Prove a Desigualdade de Erdös-Mordell: ( )2 u v w x y z+ + ≤ + + Referências Bibliográficas [1] Uma ótima fonte de problemas é o Mathlinks: http://www.mathlinks.ro/ (em inglês). [2] Para quem gosta de Geometria, o Forum Geometricorum é um prato cheio! Tudo sobre quadriláteros completos foi retirado do artigo Steiner´s Theorems on the Complete Quadrilateral, de Jean – Pierre Ehrmann, Volume 4 (2004), pp 35-52. [3] Para quem quer saber mais sobre o teorema de Erdös-Mordell, na Eureka! 18. [4] O livro Modern Geometry of the Triangle, de William Gallatly, contém muita informação interessante, incluindo a maior parte dos fatos sobre simedianas e o ponto simediano. [5] Mais conjugados isogonais? Isso e muito mais no livro Geometry of Conics (o “livro do bode” – Veja a Capa!), de A. V. Akoplyan e A. A. Zaslavsky.

Page 45: Eureka 2010

Sociedade Brasileira de Matemática

EUREKA! N°31, 2010

45

SÉRIE HARMÔNICA DE NÚMEROS PRIMOS Lenimar Nunes de Andrade UFPB – João Pessoa, PB

1. Série harmônica

Há séculos que se sabe que a soma dos recíprocos dos números inteiros positivos

1 1 1 11

2 3 4nH ...n

= + + + + +

pode ultrapassar o valor de qualquer constante positiva pré-estabelecida, bastando, para isso, somar determinada quantidade de parcelas, considerando o valor de n suficientemente grande. Quando consideramos uma infinidade de parcelas desse tipo, temos uma série infinita conhecida pelo nome de série harmônica, e, como a soma nH vai aumentando à medida que n aumenta e ultrapassa qualquer valor pré-estabelecido, temos que se trata de uma série divergente. Existem pelo menos 20 demonstrações diferentes desse fato (veja, por exemplo, a referência [3]) e algumas demonstrações simples podem ser encontradas em [1] ou [2].* Sabe-se que o crescimento das somas parciais nH da série harmônica é bastante lento. Se somarmos 1000 termos da série, obtemos 7, 4855 como resultado. Se somarmos 1000000 de termos, obtemos 14,3927. Para a soma ultrapassar 100, estima-se que seja necessário somar-se aproximadamente 431 5 10, × parcelas – um número de parcelas tão grande que nem os computadores mais modernos de hoje em dia, trabalhando ininterruptamente ao longo de vários milênios, conseguiriam efetuar todos os cálculos. Se for escolhido um determinado algarismo, e retirados da série harmônica todos os termos que contenham esse algarismo, então surpreendentemente, obtem-se uma série infinita na qual a soma dos n primeiros termos é sempre inferior a 80, não

* Nota do editor: Uma demonstração particularmente simples deste fato é a seguinte:

12

1 1 1 1 1 1 1 1 11

2 3 4 5 6 7 8 2 1 2−

= + + + + + + + + + + + ≥ + k k kH ... ...

1 1 1 1 1 1 1 1 1 1 1 1 1 11 1

2 4 4 8 8 8 8 2 2 2 2 2 2 2 ≥ + + + + + + + + + + + + = + + + + + k k k... ... ...

12k

,= + que pode ultrapassar o valor de qualquer constante positiva pré-estabelecida.

Page 46: Eureka 2010

Sociedade Brasileira de Matemática

EUREKA! N°31, 2010

46

importando qual seja o valor de n**. Obtemos, portanto, o que chamamos de série convergente.

2. Série dos recíprocos de números primos A sequência de números primos 2, 3, 5, 7, 11, 13,... é infinita. Apesar de estarem bem próximos uns dos outros, para valores pequenos, à medida que aumentamos o valor de n, torna-se difícil encontrar números primos maiores do que n, e eles vão ficando cada vez mais distantes uns dos outros em média. Se retirarmos da série hermônica todos os termos cujos denominadores não sejam primos, obtemos ainda assim uma série infinita:

1 1 1 1 1 1 12 3 5 7 11 13 17

...+ + + + + + +

vamos denominar a série assim obtida de série harmônica de números primos. O principal objetivo deste artigo é mostrar que essa série harmônica de números primos é divergente, ou seja, a soma dos seus n primeiros termos pode ultrapassar qualquer valor pré-estabelecido. Esse fato foi observado pela primeira vez por Leonhard Euler (1707–1783). O crescimento do valor das somas dos n primeiros termos da série harmônica de números primos é exatamente lento. Muito mais lento do que o da série harmônica. Com os recursos computacionais atuais, é impossível realizar a tarefa de somar uma certa quantidade de termos dessa série e obtermos o resultado igual ou superior a 5,0. 3. Demonstração da divergência Inicialmente, vamos mostrar que dado um n inteiro positivo temos que

2

2

11 1

k k kn n n

+ < + +

para todo inteiro positivo k tal que k n.≤ Em particular, fazendo k = n, obtemos que

2

2

11 1 3

n n n.

n n n + < + + =

(1)

Para isso, vamos usar o método da indução matemática. Para k = 1, a desigualdade

reduz-se a 2

1 1 11 1

n n n+ < + + o que é verdadeiro. Suponho a desigualdade válida

para k, vamos verificar que vale para k + 1 também:

** Veja o problema proposto no. 141, na página 60.

Page 47: Eureka 2010

Sociedade Brasileira de Matemática

EUREKA! N°31, 2010

47

1 2 2

2 2

1 1 1 1 11 1 1 1 1 1

k k k k k kn n n n n n n n n

+ + = + + < + + + = + + +

( ) ( ) ( )2 222

2 3 2 3 2

0

1 1 11 11 1

k k n k kk k k k,

n n n n n n n<

+ − + ++ ++ + = + + + < + +

1442443

onde utilizamos que ( )2 1k n k< + porque k n.≤ Desse modo, a desigualdade fica demonstrada. Escolhido n, um inteiro positivo qualquer, consideremos r o inteiro tal que

12 2r rn ,+≤ < ou seja, r é o expoente da maior potência de 2 que não ultrapassa n. Sejam 1 2 32 3 5 sp , p , p ,..., p= = = os primos positivos menores ou iguais a n. Se m for um inteiro tal que 1 m n,≤ ≤ então o Teorema Fundamental da Aritmética nos garante que m pode ser escrito de modo único como um produto de potências dos primos kp com expoentes inteiros não negativos 1 2 st ,t ,...,t :

31 21 2 3

st tt tsm p p p ...p=

Note que nenhum dos expoentes kt pode ser maior do que r , pois, se assim fosse,

teríamos 12 2k kt t rkm p n,+≥ ≥ ≥ > o que seria um absurdo. Assim, para todo k,

temos 0 kt r.≤ ≤

Temos, então, a seguinte desigualdade:

1 1 1 1 1 1 1 1 1 11 1 1

2 3 4 2 4 2 3 9 3r r... ... ...n

+ + + + + < + + + + × + + + + ×

2

1 1 1 1 1 11 1

5 25 5r rs s s

... ... ...p p p

+ + + + × × + + + +

(2)

Sua demonstração consiste na observação de que cada parcela 1 m que aparece do lado esquerdo da desigualdade pode ser escrita de modo único na forma

1 21 2

1 1 1 1stt t

s

...m p p p

= × × ×

e que cada fração da forma 1

ktkp

ocorre uma única vez como uma das parcelas do

fator 2

1 1 11 r

k k k

...p p p

+ + + +

que aparece no segundo membro da desigualdade.

Page 48: Eureka 2010

Sociedade Brasileira de Matemática

EUREKA! N°31, 2010

48

Supondo 2q ,≥ temos 2 2q q ,− ≥ ou seja, ( )2 1q q− ≥ que equivale a 1 2

1q q≤

− e

calculando a seguinte soma de uma progressão geométrica de razão 1 q , obtemos:

( )1

2

11

1 11 1 1 1 11 1

1 1 1 1 11 1

r

r

qqq...q q q q q q

q q

+−− +

+ + + + = < = = = +− − −− −

Usando agora a desigualdade (1) obtida no início desta seção, obtemos 1

11 3

1

q

q

+ < − que é equivalente a

111

1 31

q .q

−+ <−

Aplicando-se logaritmos, obtemos: 1

11log 1 log 3

1q ,

q−

+ < −

ou seja, 1 log 3 2 log 3

log 11 1q q q

+ < ≤ − −

de onde finalmente obtemos

2

1 1 1 1 2log3log 1 log 1

1r...q q q q q

+ + + + < + < −

(3)

Aplicando-se logaritmos aos dois membros da desigualdade (2), e usando-se propriedade ( )log log log ab a b,= + obtemos:

1 1 1 1 1 1 1log 1 log 1

2 3 4 2 4 2r... ...n

+ + + + + < + + + + +

2

1 1 1 1 1 1log 1 log 1

3 9 3r rs s s

... ... ...p p p

+ + + + + + + + + +

(4)

Usando-se várias vezes o resultado (3) na desigualdade (4), obtemos: 1 1 1 1 2 log 3 2 log 3 2 log 3 2 log 3

log 12 3 4 2 3 5 s

... ...n p

+ + + + + < + + + +

1 1 1 12 log 3

2 3 5 s

... .p

= + + + +

Page 49: Eureka 2010

Sociedade Brasileira de Matemática

EUREKA! N°31, 2010

49

Se existisse uma constante L tal que 1 1 12 3 s

... Lp

+ + + < para todo inteiro positivo

s, então teríamos ( )21 1 1 1log 1 2 log 3 log 3

2 3 4L... L

n + + + + + < × =

(5)

o que implicaria 21 1 1 11 3

2 3 4+ + + + + < L... ,

n para todo n natural, (6)

o que seria um absurdo, pois a série harmônica não é limitada e, para algum n, a

soma 1 1 1

12 3

...n

+ + + +

ultrapassaria a constante 23 L.

4. Algumas somas parciais Sabe-se que quando maior o valor de n, mais próximo de ( )( ) 1ln ln n B+ será a

soma de todos os recíprocos de primo inferiores a n. A constante 1B é conhecida como constante de Mertens e tem valor igual a 0,2614972128.... Com a ajuda de um computador, se somarmos os recíprocos dos números primos inferiores a 1000, obtemos 2,1990 como resultado. Observe que esse valor é próximo de ( )( ) 1ln ln 1000 2 1941B , .+ ≈

Somando-se todos os recíprocos de númreos primos inferiores a 710 (dez milhões), obtemos uma soma total igual a 3,041449. Calculando-se

( )( )71ln ln 10 B ,+ obtemos 3,041440 que é muito próximo da soma obtida.

A aproximação ( )( ) 1ln ln n B S+ ≈ para a soma dos recíprocos de primos inferiores

a n pode ser escrita na forma 1S Ben e .

Por exemplo, para chegar a 5,0, a soma necessitaria de 5 4 73851 494 2 10

B ,e en e e ,−

= = ≈ × parcelas, o que é um número realmente assustador: nem o computador mais rápido de hoje em dia, trabalhando incessantemente por milênios a fio, conseguiria somar tal quantidade de termos. Referências Bibliográficas [1] G. Ávila, “As séries infinitas”, RPM 30, 1996. [2] G. Garbi, “A surprendente série harmônica”, RPM 42, 2000. [3] S.J. Kifowit, T. A. Stamps, “The harmonic series diverges again and again”, The AMATYC Review, Vol. 27, No. 2, 2006. [4] D. O. Shkiyarsky, N. N. Chentsov, I. M. Yaglom, “Selected problemas and theorems in elementary Mathematics – Arithmetic and Algebra”, Mir Publishers, Moscow, 1979.

Page 50: Eureka 2010

Sociedade Brasileira de Matemática

EUREKA! N°31, 2010

50

COMO É QUE FAZ? PROBLEMA PROPOSTO POR WILSON CARLOS DA SILVA RAMOS (BELÉM – PA)

1) Resolva o sistema ( )

( )2 2

2

3 854 4

3

1 132

3

xy x yx y

xx y

+ + + = + + = +

Solução: Da segunda equação, obtemos 1

.13

23

y xx

= −−

Substituindo esse valor de

y na primeira equação, obtemos 2

24 13 854 3 2 .

13 3 323

xy x

x

+ + − = −

Fazendo

132 ,

3u x= − temos

1 13,

2 3x u = −

e

1 1 13,

2 6u

y xu u

= − = + −

donde2

226 2 13 852 3 ,

3 3 3u u

u u − + + − + =

ou seja, 22

26 68 26 44 0.

3 9 3u u

u u− − − + =

Fazendo 1

,w uu

= + temos 2 22

12,u w

u+ = − donde 2 26 140

4 0,3 9

w w− − = e logo

103

w = ou 7

.6

w = − Assim, 1 10

3u

u+ = ou

1 7.

6u

u−

+ = A primeira dessas

equações tem soluções 3u = e 1

,3

u = que nos dão as duas soluções

reais1 13 2 1 1

,2 3 3 3

x u y xu

= − = = − = −

e 1 13 1

2, 1 .2 3

x u y xu

= − = = − =

A

segunda não tem soluções reais, mas tem as soluções complexas conjugadas 7 95

,12 12

iu = − ± que nos dão as soluções complexas correspondentes

( ) 59 95 73 95, ,

24 24i i

x y + − −

=

e ( ) 59 95 73 95, , .

24 24i i

x y − − +

=

Page 51: Eureka 2010

Sociedade Brasileira de Matemática

EUREKA! N°31, 2010

51

PROBLEMA PROPOSTO POR MARCÍLIO MIRANDA DE CARVALHO (TERESINA – PI) (teste de seleção da Romênia para IMO de 1978) 2) Para cada n natural, resolva a equação: sen sen 2 ...sen cos cos 2 ...cos 1x x nx x x nx⋅ + ⋅ = SOLUÇÃO DE RENAN HENRIQUE FINDER (JOINVILLE – SC) Se 2,n ≥ usando a desigualdade triangular e o fato de que { }max sen , cos 1≤α α ,

temos1 sen sen 2 ...sen cos cos2 ...cos sen sen 2 ...sen cos cos2 ...cos= + ≤ +x x nx x x nx x x nx x x nx

sen sen 2 cos cos 2x x x x≤ ⋅ + ⋅

Pela desigualdade de Cauchy-Schwarz, 2 2 2 21 sen cos sen 2 cos 2 1.≤ + + =x x x x Para que ocorra a igualdade, devemos ter sen cos 2 cos sen 2 ,x x x x= logo

( )sen sen 2 0,x x x= − = e portanto , 0.x m m senx= ∈ ⇒ =¢π Então cos cos 2 ...cos 1.x x nx = Se m é par, isso sempre ocorre. Se m é ímpar,

cos 1,x = − cos 2 1,x = cos3 1,...,x = − logo ( ) 2cos ...cos 1 .n

x nx = − Portanto,

• Se 2n ≥ e 2n

é par (ou seja, se n é da forma 4j – 1 ou 4j), as soluções são

, .x m m= ∈¢π

• Se 2n ≥ e 2n

é ímpar (ou seja, se n é da forma 4j + 1 ou 4j + 2), as

soluções são 2 , .x m m= ∈¢π

Se n = 1 o problema equivale a 2 2 2

sen cos sen sen2 2 2 4 4

x x x = + ⇔ = +

π π

2x m⇔ = π ou 2 , .2

x m m= + ∈¢ππ

Page 52: Eureka 2010

Sociedade Brasileira de Matemática

EUREKA! N°31, 2010

52

SOLUÇÕES DE PROBLEMAS PROPOSTOS ! Publicamos aqui algumas das respostas enviadas por nossos leitores.

123. Determine todas as funções : * *f →¥ ¥ tais que

2 2 3 2 22 ( ) ( ) ( ) ( ) ( ) ,f m n f m f n f m f n+ = + para quaisquer , *m n ∈ ¥ distintos. Obs: * {1,2,3,...}=¥ é o conjunto dos inteiros positivos. SOLUÇÃO DE ÍTALO DOWELL LIRA MELO (TERESINA – PI) Primeiro note que se f é uma função constante então f é solução. Agora suponhamos que exista uma função não constante que seja solução. Assim existem naturais a e b com ( ) ( ).f a f b<

Daí temos que ( ) ( ) ( ) ( ) ( ) ( ) ( ) ( )3 3 3 2 2 32 2 .f a f a f a f a f b f a f b f b= + < + <

Como ( ) ( ) ( ) ( ) ( )3 2 22 22f a b f a f b f a f b+ = + , segue que ( ) ( ) ( )33 32 22 2 2 .f a f a b f b< + <

Se dividirmos por 2 encontramos que ( ) ( ) ( )33 32 2f a f a b f b< + < ⇒

( ) ( )2 2( ) .f a f a b f b⇒ < + < Isto nos diz que entre quaisquer dois valores distintos de f podemos encontrar um outro valor de f mas isto não pode ocorer sempre uma vez que f assume valores em

*.¥ Esta contradição mostra que tal função não existe. Assim as funções constantes são as únicas soluções. 124. Considere a seqüência 1( )n na ≥ definida por 1 2 3 4 1a a a a= = = = e

21 3 2

4, 5.n n n

nn

a a aa n

a− − −

+= ∀ ≥

Prove que na é um inteiro positivo, para todo inteiro positivo n. SOLUÇÃO DE ZOROASTRO AZAMBUJA NETO (RIO DE JANEIRO – RJ) Vamos provar por indução que, para todo 5,n ≥ valem as seguintes afirmações:

• 24 1 3 2 , ,5k k k ka a a a k k n− − − −+ ∀ ∈ ≤ ≤¥ (e logo ,ka k n∈ ∀ ≤¥ ).

• 3 23 2 1 4 , ,5 .k k k ka a a a k k n− − − −+ ∀ ∈ ≤ ≤¥

• 2 32 1 4 3 , ,5 .k k k ka a a a k k n− − − −+ ∀ ∈ ≤ ≤¥

• ( )1, 1, ,1 1 .k kmdc a a k k k n− = ∀ ∈ ≤ − < ≤¥

• ( )2 , 1, ,1 2 .k kmdc a a k k k n− = ∀ ∈ ≤ − < ≤¥

Page 53: Eureka 2010

Sociedade Brasileira de Matemática

EUREKA! N°31, 2010

53

• ( )3 , 1, ,1 3 .k kmdc a a k k k n− = ∀ ∈ ≤ − < ≤¥

Note que, como 1 2 3 4 1,a a a a= = = = temos 5 2a = e 6 3a = , e portanto os itens acima se verificam para todo 6n ≤ . Vamos agora verificá-los para k= n + 1: queremos mostrar que

( )2 221 3 2 2 1 42 21 3 2

3 2 1 2 14 4

.n n n n n nn n nn n n n n n

n n

a a a a a aa a aa a a a a a

a a− − − − − −− − −

− − − − −− −

+ + ++ = + =

Como o lado direito é inteiro e ( )3 4, 1,n nmdc a a− − = isso equivale a mostrar que

( )2 23 1 3 2 2 1 4 ,n n n n n n na a a a a a a− − − − − − −+ + o que segue de 3 2

3 2 1 4 .n n n na a a a− − − −+

Também queremos mostrar que

( )22 3 2 221 4 1 3 2 33 2 3 1 3 2

2 1 3 1 3 24 4

.n n n n n nn n nn n n n n n

n n

a a a a a aa a aa a a a a a

a a− − − − − −− − −

− − − − −− −

+ + ++ = + =

Como o lado direito é inteiro e ( )2 4, 1,n nmdc a a− − = isso equivale a mostrar que

( )23 2 22 1 4 1 3 2 3 ,n n n n n n na a a a a a a− − − − − − −+ + o que equivale a

( )3 2 2 3 2 2 32 1 4 1 3 1 1 4 3 ,− − − − − − − − −+ = +n n n n n n n n na a a a a a a a a e isso segue de 2 3

2 1 4 3 .n n n na a a a− − − −+

E também queremos mostrar que

( )2 2 321 3 2 3 2 42 3 2 31 3 2

1 3 2 3 24 4

.n n n n n nn n nn n n n n n

n n

a a a a a aa a aa a a a a a

a a− − − − − −− − −

− − − − −− −

+ + ++ = + =

Como o lado direito é inteiro e ( )1 4, 1,n nmdc a a− − = isso equivale a mostrar que

( )2 2 31 1 3 2 3 2 4 ,n n n n n n na a a a a a a− − − − − − −+ + o que equivale a

( )2 2 3 2 21 2 3 2 4 2 3 2 4 ,n n n n n n n n na a a a a a a a a− − − − − − − − −+ = + que segue de 2

1 3 2 4 ,n n n na a a a− − − −+

que por sua vez segue da igualdade 21 5 3 2 4 ,n n n n na a a a a− − − − −= + que vem da definição

de 1.na −

Finalmente, de 21 3 2 1,n n n n na a a a a+ − − −= + segue que ( ) ( )2

1 1, , 1,n n n nmdc a a mdc a a+ − =

( ) ( )1 1 1 2, , 1n n n n nmdc a a mdc a a a+ − − − = e ( ) ( )21 2 2 1, , 1.n n n nmdc a a mdc a a+ − − − =

125. Considere dois naturais 2 m ≥ e 2,n ≥ e as seqüências

0 1 2( , , ,..., ), {0,1}.mn ia a a a a ∈ As seqüências de tipo m satisfazem as condições:

Page 54: Eureka 2010

Sociedade Brasileira de Matemática

EUREKA! N°31, 2010

54

• 0,k k ma a + = para todo k; • Se 1 1k ka a + = então m divide k

As seqüências de tipo n são definidas analogamente. Prove que existem tantas seqüências do tipo m quanto do tipo n. SOLUÇÃO DE JOSÉ DE ALMEIDA PANTERA (RIO DE JANEIRO – RJ) Considere a matriz ( )1

1, i mij

j nm n B b ≤ ≤

≤ ≤× = dada por ( 1) ,1 ,1 .− += ≤ ≤ ≤ ≤ij m i jb a i m j n

Temos que ( )0 ,..., mna a é uma sequência de tipo m se e somente se B é uma matriz m n× cujas entradas pertencem a {0, 1} sem dois termos vizinhos iguais a 1 numa mesma linha ou numa mesma coluna, e tal que 1 01 0.mb a= ⇒ =

Considere agora a função f que leva uma matriz B m n× na matriz ( ) f B n m×

dada por ( )( ) ( ) ( )1 , 1,.m j n ii j

f B B + − + −= Temos que f é uma bijeção entre as matrizes

m n× cujas entradas pertencem a {0, 1} sem dois termos vizinhos iguais a 1 numa mesma linha ou numa mesma coluna e as matrizes n m× cujas entradas pertencem a {0, 1} sem dois termos vizinhos iguais a 1 numa mesma linha ou numa mesma coluna, tal que ( )( ) 1,1,

.nmf B B= Isso mostra que o número de seqüências do tipo

m é igual ao número de seqüências do tipo n. 126. As circunferências ,0 5,i iΓ ≤ ≤ são tangentes a uma circunferência Γ nos pontos iA . Além disso, iΓ é tangente a 1i+Γ para 0 5i≤ ≤ e 5Γ é tangente a 0.Γ Prove que 0 3 1 4 2 5, ,A A A A A A são concorrentes. SOLUÇÃO DE MATHEUS SECCO TORRES DA SILVA (RIO DE JANEIRO – RJ)

Sejam iO os centros de iΓ e ir os raios de iΓ . Além disso, seja r o raio de Γ , que tem centro O. Vamos supor inicialmente que as circunferências iΓ são exteriores a Γ . Usando Ceva trigonométrico no 2 4 0 ,A A A∆ devemos provar que

( )( )

( )( )

( )( )

1 4 2 3 0 4 5 2 0

1 4 0 3 0 2 5 2 4

1sen A A A sen A A A sen A A Asen A A A sen A A A sen A A A

< < <⋅ ⋅ = ⇔

< < <

3 4 5 01 2

2 3 4 5 0 1

2 22 1

2 2 2

A OA A OAAOAsen sensen

A OA A OA A OAsen sen sen

< << ⋅ ⋅ = < < <

Page 55: Eureka 2010

Sociedade Brasileira de Matemática

EUREKA! N°31, 2010

55

A0

A1

A2

A5

A4

A3

Denotamos 1 ,0 5,i i iAOA i+< = ≤ ≤α índices módulo 6. Mas no 1,i iOO O +∆ temos:

( ) ( ) ( ) ( )( )2 2 21 1 12 cosi i i i i i ir r r r r r r r r r+ + ++ = + + + − + + ⋅ ⇒α

( )( )1

1

2cos 1 i i

ii i

r rr r r r

+

+

= − ⇒+ +

α

( )( )2 1

1

21 2 1

2i i i

i i

r rsen

r r r r+

+

− = − ⇒+ +

α

( )( )2 1

1

,2

i i i

i i

r rsen

r r r r+

+

=+ +

α donde

2 22 3 51

2 2 22 4 6

2 22 1

2 2 2

sen sensen

sen sen sen

⋅ ⋅ = ⇒

α αα

α α α

3 51

2 4 6

2 22 1,

2 2 2

sen sensen

sen sen sen

⋅ ⋅ =

α αα

α α αpois 0, 5,

2isen i > ∀ ≤

α

donde obtemos o desejado. Isso conclui a prova!. Obs.: Supusemos que as circunferências iΓ estavam no exterior de 1Γ , mas se

Page 56: Eureka 2010

Sociedade Brasileira de Matemática

EUREKA! N°31, 2010

56

fossem interiores, obteríamos ( )( )

2 1

1,

2i i i

i i

r rsen

r r r r+

+=

− −α o que também nos daria o

desejado, com um argumento análogo.

129. Um coelho está numa rua infinita dividida em quadrados numerados pelos inteiros, e começa no quadrado 0. Se num dado momento ele está no quadrado k,

ele escolhe, com probabilidade 12

, pular para o quadrado k + 2 ou, também com

probabilidade 12

, pular para o quadrado k – 1. Ele continua esse processo

indefinidamante. Dado m ∈¢ , determine a probabilidade de, em algum momento, o coelho pisar no quadrado m.

SOLUÇÃO DE ASDRUBAL PAFÚNCIO SANTOS (BOTUCATU – SP) Denotemos por ma a probabilidade de, em algum momento, o coelho pisar no quadrado m. Seja 0n ≠ um inteiro. Após o primeiro passo do coelho, ele pode estar no quadrado

–1, com probabilidade 1

,2

ou no quadrado 2, com probabilidade1

,2

ficando a

distâncias respectivamente n + 1 e n – 2 do quadrado n. Assim, para todo

( )2 1

1 10, * .

2 2n n nn a a a− +≠ = +

Como 2 5

0

12

n

nk

nk

=

∑ tende a 0 exponencialmente rápido, com probabilidade total

temos que, para um certo 0 ,n ∈¥ e para todo 0 ,n n≥ pelo menos 40% dos n primeiros passos do coelho são para frente, o que faz com que, após n passos, ele

esteja num quadrado de número maior ou igual a 2 3

2 ,5 5 5n n n

⋅ − = para todo

0 .n n≥ Em particular, a probabilidade de o coelho pisar no quadrado m tende a 0 quando m tende a −∞ .

De (*), temos 3 22 , 0,m m ma a a m+ += − ∀ ≥ donde existem constantes A, B, C com

1 5 1 5,

2 2

m m

ma A B C + −

= + +

0,m∀ ≥ pois o polinômio característico da

Page 57: Eureka 2010

Sociedade Brasileira de Matemática

EUREKA! N°31, 2010

57

recorrência acima é ( )3 2 1 5 1 52 1 1 .

2 2x x x x x

+ −− + = − − −

Como ( )ma é limitada, devemos ter B = 0. Como 0 1,a = devemos ter A + C = 1.

De (*), também temos 3 12 , 1,m m ma a a m− − − − += − ∀ ≥ ou seja, fazendo ,k kb a−= temos 3 12 , 0,k k kb b b k+ += − ∀ ≥ donde, como

( )3 1 5 1 52 1 1 ,

2 2x x x x x

− + − −− + = − − −

existem ° ° °, ,A B C com

° ° °1 5 1 5,

2 2

k k

kb A B C − + − −

= + +

0.k∀ ≥ Como kb não só é limitado como

tende a 0 quando k tende a +∞ , devemos ter ° ° 0,A C= = e como 0 0 1,b a= =

devemos ter ° 1.B =

Assim, 1 5

1 , 02

m

ma C C m −

= − + ∀ ≥

e 1 5 1 5

, 0.2 2

m m

m ma b m−

− + += = = ∀ ≤

Fazendo n = 1 em (*), obtemos 1 1 2

1 1,

2 2a a a−= + donde

1 5 1 1 5 1 3 51 1 ,

2 2 2 2 2C C C C

− − + −− + = + − +

e portanto

7 3 5.

2C

−=

Assim, temos

3 5 5 7 3 5 1 5, 0

2 2 2.

1 5, 0

2

m

m m

m

a

m

− − − + ∀ ≥ = +

∀ ≤

130. Suponha que , ,a b c ∈¡ e a equação ( )2 ( ) 0x a b c x ab ac bc− + + + + + = não tem raízes reais. Prove que a, b e c têm todos o mesmo sinal e existe um triângulo de lados ,a b e .c

Page 58: Eureka 2010

Sociedade Brasileira de Matemática

EUREKA! N°31, 2010

58

SOLUÇÃO DE DANIEL EITI NISHIDA KAWAI (TAUBATÉ – SP) Se a equação ( ) ( )2 0x a b c x ab ac bc− + + ⋅ + + + = não tem raízes reais, temos

( ) ( )20 4 1 0∆ < ⇔ + + − ⋅ ⋅ + + < ⇔a b c ab ac bc 2 2 2 2 2 2 4 4 4 0a b c ab ac bc ab ac bc⇔ + + + + + − − − < ⇔ 2 2 2 2 2 2 0a b c ab ac bc⇔ + + − − − < ⇔

( )22 2 2 2 2 2 4 0 4 0⇔ + + + − − − < ⇔ + − − < ⇔a b c ab ac bc ab a b c ab

( ) ( )2 24 0 4 4 0 0a b c ab a b c ab ab ab a⇔ + − < ⇔ ≤ + − < ⇒ > ⇒ > ⇒ e b têm o mesmo sinal. De maneira análoga, b e c têm o mesmo sinal. Assim, a, b e c têm todos o mesmo sinal, e logo ,ab a b ac a c= = e bc b c= .

Assim, 2 2 2 2 2 2 0,a b c ab ac bc+ + − − − < donde 2 2 2 2 2 2 4a b c ab ac bc ab⇒ + + + − − < ⇒

2 2 2 2 2 2 4a b c a b a c b c a b⇒ + + + − − < ⇒

( ) ( )222 2a b c a b a b a b c⇒ + − < ⋅ ⇒ − ⋅ < + − ⇒

( ) ( ) ( )2 2 2

2 2⇒ < + ⋅ + ⇒ < + ⋅ + ⇒c a a b b c a a b b

( ) ( )2 2

.c a b c a b⇒ < + ⇒ < +

De maneira análoga, b a c< + e .a b c< + Assim, existe um triângulo de lados

,a b e .c

Agradecemos o envio de soluções e a colaboração de: Carlos Alberto da Silva Victor (Nilópolis – RJ) Prob. 123, 130 Rodrigo dos Anjos Azevedo (Três Rios – RJ) Prob. 130 Vinicius dos Nascimento S. Mano (Petrópolis – RJ) Prob. 130 Marcelo Robeiro de Souza (Rio de Janeiro – RJ) Prob. 130 Jheimyson Rego Barnabé (Imperatriz – MA) Prob. 130 Flávio Antonio Alves (Amparo – SP) Prob. 130 Ítalo Dowell Lira Melo (Teresina – PI) Prob. 130 Curro Fernández López (Lugo, Espanha) Prob. 42 Miguel Amengual Covas (Mallorca, Espanha) Prob. 110 Bruno Salgueiro Fanego (Galicia, Espanha) Prob. 116, 117, 118 Continuamos aguardando soluções para os problemas 131 e 132.

Page 59: Eureka 2010

Sociedade Brasileira de Matemática

EUREKA! N°31, 2010

59

PROBLEMAS PROPOSTOS * Convidamos o leitor a enviar soluções dos problemas propostos e sugestões de novos problemas para próximos números. 133) Considere um n–ágono regular inscrito em um círculo unitário, fixe um vértice i e denote por dj a distância entre este vértice i e o vértice j. Prove que

( )1

2 2

0

5n

j nj ij

d F−

≠=

− =∏ onde 1 10, 1F F= = e 1 2,n n nF F F− −= se 2.n ≥

134) Considere a operação ⋅ entre dois vetores do 3¡ definida por: ( ) ( ) ( ), , , , , ,x y z u v w xu yw zv xw zu yv xv yu zw⋅ = + + + + + +

Prove que, para todo 1,k ≥ se ( ) ( ), , 0,0,0 então 0. kx y z x y z= = = =

Obs.: Para qualquer ( ) ( ) ( )13, , , , , , ,∈ =¡x y z x y z x y z e, para todo 1,>k

( ) ( ) ( ) 1, , , , , , .−= ⋅k kx y z x y z x y z

135) Considere um hemisfério cuja base é um círculo ( )1 .C Um círculo ( )2C do

hemisfério é paralelo a ( )1 ,C de forma que existem n círculos do hemisfério,

congruentes, tangentes entre si, a ( )1C e a ( )2 .C Mostre que a razão K(n) entre os

raios de ( )2C e ( )1C é igual a: ( )2

2

cos.

π=

+n

K nsen n

136) Sejam R, 1 2,r r e 3r os raios dos círculos de centro ,O 1 2,O O e 3 ,O

respectivamente, conforme a figura abaixo. Prove que: 1 2 1 3 2 3R r r r r r r= + + .

O1

O2 O3

R

O R2

137) Sendo A um conjunto de quinze pontos de 2¡ tal que a distância de cada ponto à origem é positiva e menor do que 1 e que quaisquer dois deles nunca sejam

Page 60: Eureka 2010

Sociedade Brasileira de Matemática

EUREKA! N°31, 2010

60

colineares com a origem. Mostre que existe um triângulo com dois vértices em A e

um na origem cuja área é menor que 1

.4

138) Calcule o máximo divisor comum entre todos os números da forma ,⋅ ⋅x y z

onde ( ), ,x y z percorre todas as soluções inteiras da equação 2 2 2x y z+ = com 0.⋅ ⋅ ≠x y z

139) Determine todos os inteiros positivos x, y, z satisfazendo 3 3 2x y z− = , onde y é primo, z não é divisível por 3 e z não é divisível por y. 140) Mostre que 2903 803 464 261n n n n− − + é divisível por 1897, para todo .n∈¥ 141) Dado { }0,1,2,3,4,5,6,7,8,9 ,∈a seja ≠ ∅X um conjunto finito de inteiros positivos, tal que nenhum dos seus elementos possui o algarismo a em sua

representação decimal. Prove que 1

80.∈

<∑n X n

Problema 133 e 134 proposto por Evandro Makiyama de Melo (São Paulo – SP) (foram propostos originalmente na IX e na II Olimpíada Iberoamericana de Matemática Universitária, respectivamente); 135 e 136 propostos por Ramilson Medeiros Pitombeira (Rio de Janeiro – RJ); 137 proposto por Ítalo Dowell Lira Melo (Teresina – PI); 138 proposto por Luiz Felipe Silva; 139 proposto por Adriano Carneiro (Caucaia – CE); 140 proposto por Wilson Carlos da Silva Ramos (Belém – PA).

Page 61: Eureka 2010

Sociedade Brasileira de Matemática

EUREKA! N°31, 2010

61

AGENDA OLÍMPICA

XXXII OLIMPÍADA BRASILEIRA DE MATEMÁTICA

NÍVEIS 1, 2 e 3 Primeira Fase – Sábado, 12 de junho de 2010

Segunda Fase – Sábado, 18 de setembro de 2010 Terceira Fase – Sábado, 16 de outubro de 2010 (níveis 1, 2 e 3)

Domingo, 17 de outubro de 2010 (níveis 2 e 3 - segundo dia de prova).

NÍVEL UNIVERSITÁRIO Primeira Fase – Sábado, 18 de setembro de 2010

Segunda Fase – Sábado, 16 e Domingo, 17 de outubro de 2010

ASIAN PACIFIC MATH OLYMPIAD (APMO) 06 de março de 2010

XVI OLIMPÍADA DE MAIO

08 de maio de 2010

XXI OLIMPÍADA DE MATEMÁTICA DO CONE SUL 13 a 19 de junho de 2010

Águas de São Pedro, SP – Brasil

LI OLIMPÍADA INTERNACIONAL DE MATEMÁTICA 02 a 14 de julho de 2010

Astana, Cazaquistão

XVII OLIMPÍADA INTERNACIONAL DE MATEMÁTICA UNIVERSITÁRIA 24 a 30 de julho de 2010

Blagoevgrad, Bulgária

XXIV OLIMPÍADA IBEROAMERICANA DE MATEMÁTICA 17 a 27 de setembro de 2010

Paraguai

II COMPETIÇÃO IBEROAMERICANA INTERUNIVERSITÁRIA DE MATEMÁTICA 3 a 9 de outubro de 2010

Rio de Janeiro, Brasil

XIII OLIMPÍADA IBEROAMERICANA DE MATEMÁTICA UNIVERSITÁRIA

Page 62: Eureka 2010

Sociedade Brasileira de Matemática

EUREKA! N°31, 2010

62

COORDENADORES REGIONAIS

Alberto Hassen Raad (UFJF) Juiz de Fora – MG Américo López Gálvez (USP) Ribeirão Preto – SP Andreia Goldani FACOS Osório – RS Antonio Carlos Nogueira (UFU) Uberlândia – MG Benedito Tadeu Vasconcelos Freire (UFRN) Natal – RN Carmen Vieira Mathias (UNIFRA) Santa María – RS Claus Haetinger (UNIVATES) Lajeado – RS Cláudio de Lima Vidal (UNESP) S.J. do Rio Preto – SP Denice Fontana Nisxota Menegais (UNIPAMPA) Bagé – RS Disney Douglas Lima de Oliveira (UFAM) Manaus – AM Edson Roberto Abe (Colégio Objetivo de Campinas) Campinas – SP Edney Aparecido Santulo Jr. (UEM) Maringá – PR Élio Mega (Grupo Educacional Etapa) São Paulo – SP Eudes Antonio da Costa (Univ. Federal do Tocantins) Arraias – TO Fábio Brochero Martínez (UFMG) Belo Horizonte – MG Florêncio Ferreira Guimarães Filho (UFES) Vitória – ES Francinildo Nobre Ferreira (UFSJ) São João del Rei – MG Genildo Alves Marinho (Centro Educacional Leonardo Da Vinci) Taguatingua – DF Graziela de Souza Sombrio (UNOCHAPECÓ) Chapecó – SC Gilson Tumelero (UTFPR) Pato Branco – PR Ivanilde Fernandes Saad (UC. Dom Bosco) Campo Grande – MS João Benício de Melo Neto (UFPI) Teresina – PI João Francisco Melo Libonati (Grupo Educacional Ideal) Belém – PA Jose de Arimatéia Fernandes (UFPB) Campina Grande – PB José Luiz Rosas Pinho (UFSC) Florianópolis – SC José Vieira Alves (UFPB) Campina Grande – PB José William Costa (Instituto Pueri Domus) Santo André – SP Krerley Oliveira (UFAL) Maceió – AL Licio Hernandes Bezerra (UFSC) Florianópolis – SC Luciano G. Monteiro de Castro (Sistema Elite de Ensino) Rio de Janeiro – RJ Luzinalva Miranda de Amorim (UFBA) Salvador – BA Marcelo Rufino de Oliveira (Grupo Educacional Ideal) Belém – PA Marcelo Mendes (Colégio Farias Brito, Pré-vestibular) Fortaleza – CE Newman Simões (Cursinho CLQ Objetivo) Piracicaba – SP Nivaldo Costa Muniz (UFMA) São Luis – MA Nivaldo de Góes Grulha Jr. (USP – São Carlos) São Carlos – SP Osnel Broche Cristo (UFLA) Lavras – MG Uberlândio Batista Severo (UFPB)) João Pessoa – PB Raul Cintra de Negreiros Ribeiro (Colégio Anglo) Atibaia – SP Ronaldo Alves Garcia (UFGO) Goiânia – GO Rogério da Silva Ignácio (Col. Aplic. da UFPE) Recife – PE Reginaldo de Lima Pereira (Escola Técnica Federal de Roraima) Boa Vista – RR Reinaldo Gen Ichiro Arakaki (UNIFESP) SJ dos Campos – SP Ricardo Amorim (Centro Educacional Logos) Nova Iguaçu – RJ Sérgio Cláudio Ramos (IM-UFRGS) Porto Alegre – RS Seme Gebara Neto (UFMG) Belo Horizonte – MG Tadeu Ferreira Gomes (UEBA) Juazeiro – BA Tomás Menéndez Rodrigues (U. Federal de Rondônia) Porto Velho – RO Valdenberg Araújo da Silva (U. Federal de Sergipe) São Cristovão – SE Vânia Cristina Silva Rodrigues (U. Metodista de SP) S.B. do Campo – SP Wagner Pereira Lopes (CEFET – GO) Jataí – GO

Page 63: Eureka 2010

CONTEÚDO

XXXI OLIMPÍADA BRASILEIRA DE MATEMÁTICA Problemas e soluções da Primeira Fase

2

XXXI OLIMPÍADA BRASILEIRA DE MATEMÁTICA Problemas e soluções da Segunda Fase

14 XXXI OLIMPÍADA BRASILEIRA DE MATEMÁTICA Problemas e soluções da Terceira Fase

34 XXXI OLIMPÍADA BRASILEIRA DE MATEMÁTICA Problemas e soluções da Primeira Fase Nível Universitário

59 XXXI OLIMPÍADA BRASILEIRA DE MATEMÁTICA Problemas e soluções da Segunda Fase Nível Universitário

65 XXXI OLIMPÍADA BRASILEIRA DE MATEMÁTICA Premiados

75 AGENDA OLÍMPICA

81

COORDENADORES REGIONAIS

82

Page 64: Eureka 2010

Sociedade Brasileira de Matemática

EUREKA! N°32, 2010

2

XXXI OLIMPÍADA BRASILEIRA DE MATEMÁTICA Problemas e soluções da Primeira Fase

PROBLEMAS – NÍVEL 1

1. Se 18

de um número é 1 ,5

quanto vale 58

desse número?

A) 18

B) 15

C) 1 D) 85

E) 2

2. Na figura, C é um ponto do segmento BD tal que ACDE é um retângulo e ABCE é um paralelogramo de área 22 cm2. Qual é a área de ABDE, em cm2? A) 28 B) 33 C) 36 D) 42 E) 44

A

B E

C

D 3. Numa festa, o número de pessoas que dançam é igual a 25% do número de pessoas que não dançam. Qual é a porcentagem do total de pessoas na festa que não dançam? A) 50% B) 60% C) 75% D) 80% E) 84% 4. De quantas maneiras dois casais podem sentar-se em quatro cadeiras em fila se marido e mulher devem sentar-se em cadeiras vizinhas? A) 2 B) 4 C) 8 D) 12 E) 24 5. Eliana tem 27 cubos iguais em tamanho, mas 4 são brancos e os demais, pretos. Com esses 27 cubos, ela monta um cubo maior. No máximo, quantas faces inteiramente pretas ela poderá obter? A) 1 B) 2 C) 3 D) 4 E) 5 6. A figura abaixo é o mapa de um bairro: os pontos A, B, C e D são as casas e os segmentos são as ruas. De quantas casas é possível fazer um caminho que passa exatamente uma vez por cada uma das ruas? É permitido passar mais de uma vez por uma mesma casa.

Page 65: Eureka 2010

Sociedade Brasileira de Matemática

EUREKA! N°32, 2010

3

A

B

C

D

A) 0 B) 1 C) 2 D) 3 E) 4 7. Se a = 240, b = 320 e c = 710, então: A) c < b < a B) a < c < b C) b < a < c D) b < c < a E) c < a < b 8. Esmeralda lançou um dado dez vezes e obteve 57 como soma de todos os pontos obtidos nesses lançamentos. No mínimo, quantas vezes saíram 6 pontos? A) 5 B) 6 C) 7 D) 8 E) 9 9. Usando palitos de fósforos, podemos construir um hexágono regular, formado por seis triângulos equiláteros unitários, como mostra a figura. Juntando mais palitos a esse hexágono, queremos obter outro hexágono regular com o quádruplo da área, também formado por triângulos equiláteros unitários. Quantos palitos deverão ser acrescentados? A) 12 B) 24 C) 30 D) 36 E) 48

10. Cinco cartas iguais têm um lado branco e um lado preto. Elas se encontram em fila com a face branca para cima. Um movimento consiste em escolher um único par de cartas vizinhas e virá-las. No mínimo, quantos movimentos são necessários para que as cartas fiquem como na figura ao lado?

A) 2 B) 3 C) 4 D) 5 E) Não é possível obter a configuração acima.

Page 66: Eureka 2010

Sociedade Brasileira de Matemática

EUREKA! N°32, 2010

4

11. Uma barra de chocolate é dividida entre Nelly, Penha e Sônia. Sabendo que

Nelly ganha 52 da barra, Penha ganha

41 e Sônia ganha 70 gramas, o peso da

barra, em gramas, é: A) 160 B) 200 C) 240 D) 280 E) 400 12. Numa fila para compra de ingressos para um jogo da seleção brasileira, havia 49 pessoas: 25 corintianos, 14 flamenguistas e 10 gremistas. Sabendo que cada pessoa da fila torce para um único time, dois torcedores do mesmo time não estão em posições consecutivas, podemos concluir que: A) tal fila não existe. B) algum dos torcedores das extremidades da fila é gremista. C) algum dos torcedores das extremidades da fila é flamenguista. D) algum flamenguista é vizinho de um gremista. E) algum gremista é vizinho de dois corintianos. 13. Na figura, P é um ponto da reta CD. A região cinza é comum ao retângulo ABCD e ao triângulo ADP. Se AB = 5 cm, AD = 8 cm e a área da região cinza

é 34

da área do retângulo, quanto vale a distância

PC? A) 1 cm B) 2 cm C) 3 cm D) 4 cm E) 5 cm

C D

B

Q

A

P

14. Numa pesquisa sobre o grau de escolaridade, obtiveram-se os resultados expressos no gráfico abaixo: Que fração do total de entrevistados representa o total de pessoas que terminaram pelo menos o Ensino Fundamental?

Page 67: Eureka 2010

Sociedade Brasileira de Matemática

EUREKA! N°32, 2010

5

A) 117

B) 133 C)

165 D)

1311

E) 16

17

15. Um número natural A de três algarismos detona um número natural B de três algarismos se cada algarismo de A é maior do que o algarismo correspondente de B. Por exemplo, 876 detona 345; porém, 651 não detona 542 pois 1 < 2. Quantos números de três algarismos detonam 314? A) 120 B) 240 C) 360 D) 480 E) 600 16. O relógio de parede indica inicialmente meio-dia. Os ponteiros das horas e dos minutos irão formar um ângulo de 90 graus pela primeira vez: A) entre 12h e 12h10min. B) entre 12h10min e 12h15min. C) entre 12h15min e 12h20min. D) entre 12h20min e 12h25min. E) após as 12h25min.

12

6

3 9

17. Eduardo escreveu todos os números de 1 a 2009 numa folha de papel. Com os amigos, combinou o seguinte: cada um deles poderia apagar quantos números quisesse e escrever, no fim da lista, o algarismo das unidades da soma dos números apagados. Por exemplo, se alguém apagasse os números 28, 3, 6, deveria escrever no fim da lista o número 7, pois 28 + 3 + 6 = 37. Após algum tempo, sobraram somente dois números. Se um deles era 2000, qual dos números a seguir poderia ser o outro? A) 0 B) 1 C) 3 D) 5 E) 6 18. Uma folha de caderno de Carlos é um retângulo com dois lados (bordas) amarelos de 24 cm e dois lados (bordas) vermelhos de 36 cm. Carlos pinta cada ponto do retângulo na mesma cor do lado mais próximo desse ponto. Qual é a área da região pintada de amarelo? A) 144 cm2 B) 288 cm2 C) 364 cm2 D) 442 cm2 E) 524 cm2 19. O professor Piraldo aplicou uma prova de 6 questões para 18 estudantes. Cada questão vale 0 ou 1 ponto; não há pontuações parciais. Após a prova, Piraldo elaborou uma tabela como a seguinte para organizar as notas, em que cada linha representa um estudante e cada coluna representa uma questão.

Page 68: Eureka 2010

Sociedade Brasileira de Matemática

EUREKA! N°32, 2010

6

Questões→ Estudantes

1 2 3 4 5 6

Arnaldo 0 1 1 1 1 0 Bernaldo 1 1 1 0 0 1 Cernaldo 0 1 1 1 1 0

Piraldo constatou que cada estudante acertou exatamente 4 questões e que cada questão teve a mesma quantidade m de acertos. Qual é o valor de m? A) 8 B) 9 C) 10 D) 12 E) 14 20. Alguns cubos foram empilhados formando um bloco. As figuras ao lado representam a vista da esquerda e da frente desse bloco. Olhando o bloco de cima, qual das figuras a seguir não pode ser vista?

vista da esquerda vista da frente

A) B) C) D) E)

esqu

erda

esqu

erda

esqu

erda

esqu

erda

esqu

erda

frente frente frente frente frente

PROBLEMAS – NÍVEL 2 1. Veja o Problema No. 1 do Nível 1. 2. Veja o Problema No. 9 do Nível 1. 3. Veja o problema No. 4 do Nível 1.

4. Se 1 4,5

=+x

o valor de 6

1+x

é:

A) 51

B) 41

C) 32

D) 54

E) 1

5. Veja o Problema No. 6 do Nível 1. 6. Os inteiros positivos m e n satisfazem 15m = 20n. Então é possível afirmar, com certeza, que mn é múltiplo de:

Page 69: Eureka 2010

Sociedade Brasileira de Matemática

EUREKA! N°32, 2010

7

A) 5 B) 10 C) 12 D) 15 E) 20 7. Veja o problema No. 15 do Nível 1. 8. Veja o Problema No. 11 do Nível 1. 9. Veja o Problema No. 8 do Nível 1. 10. Na figura abaixo, 18=α e AB = AC = AD = AE. O valor do ângulo β é:

B

E C D

A

β

α α α

A) 18o B) 36o C) 15o D) 20o E) 30o 11. Veja o Problema No. 10 do Nível 1. 12. Na figura abaixo, ABCDE é um pentágono regular, CDFG é um quadrado e DFH é um triângulo equilátero. O valor do ângulo β é:

H F

G D

E

A B

β

C

A) 30o B) 36o C) 39o D) 45o E) 60o

Page 70: Eureka 2010

Sociedade Brasileira de Matemática

EUREKA! N°32, 2010

8

13. Veja o problema No. 12 do Nível 1. 14. Veja o Problema No. 13 do Nível 1. 15. A famosa Conjectura de Goldbach diz que todo número inteiro par maior que 2 pode ser escrito como a soma de dois números primos. Por exemplo, 18 pode ser representado por 5 + 13 ou, ainda, por 7 + 11. Considerando todas as possíveis representações de 126, qual a maior diferença entre os dois primos que a formam? A) 112 B) 100 C) 92 D) 88 E) 80 16. Na figura ao lado, E é o ponto médio de AB, F é o ponto médio de AC e BR = RS = SC. Se a área do triângulo ABC é 252, qual é a área do pentágono AERSF? A) 168 B) 189 C) 200 D) 210 E) 220

A

B

E F

R S C

17. Quantos pares ordenados (x, y) de números reais satisfazem a equação

( ) ( )2 22 2 0?− + − − =x y x y

A) 0 B) 1 C) 2 D) 3 E) infinitos 18. Veja o Problema No. 19 do Nível 1. 19. Entre os inteiros positivos 4018,+n 21,2,...,2009 ,=n quantos são quadrados perfeitos? A) 1945 B) 1946 C) 1947 D) 1948 E) 1949 20. Para cada número natural n, seja nS a soma dos dez primeiros múltiplos positivos de n. Por exemplo, 2S = 2 + 4 + 6 + 8 + 10 + 12 + 14 + 16 + 18 + 20. Quanto é 10321 SSSS ++++ ? A) 2925 B) 3025 C) 3125 D) 3225 E) 3325

Page 71: Eureka 2010

Sociedade Brasileira de Matemática

EUREKA! N°32, 2010

9

21. Em uma folha quadriculada em que cada quadrado tem lado 2cm, são desenhados dois círculos como na figura ao lado. A distância mínima entre os dois círculos mede: A) 3cm B) 10 cm

C) ( )10 3+ cm

D) ( )10 2− cm

E) ( )10 3− cm

22. Quantos números naturais de 1 a 100, inclusive, podem ser escritos na forma de potência ba , com , ∈a b e , 1?>a b A) 10 B) 12 C) 14 D) 16 E) 18 23. Veja o Problema No. 18 do Nível 1. 24. Os inteiros 0 < x < y < z < w < t são tais que w = z(x + y) e t = w(y + z). Sendo w = 9, então t é igual a A) 45 B) 54 C) 63 D) 72 E) 81 25. Veja o Problema No. 20 do Nível 1. PROBLEMAS – NÍVEL 3 1. Veja o problema No. 15 do Nível 1. 2. Veja o problema No. 6 do Nível 2. 3. Se x2 = x + 3 então x3 é igual a: A) x2 + 3 B) x + 4 C) 2x + 2 D) 4x + 3 E) x2 – 2 4. Na figura, o quadrado A’B’C’D’ foi obtido a partir de uma rotação no sentido horário do quadrado ABCD de 25 graus em torno do ponto médio de AB. Qual é o ângulo agudo, em graus, entre as retas AC e B’D’?

Page 72: Eureka 2010

Sociedade Brasileira de Matemática

EUREKA! N°32, 2010

10

AB

A’

B’

C’

D’ CD

A) 5 B) 25 C) 45 D) 65 E) 85 5. Um dos cinco números a seguir é divisor da soma dos outros quatro. Qual é esse número? A) 20 B) 24 C) 28 D) 38 E) 42 6. Sempre que Agilulfo volta para casa depois da escola com uma advertência, se sua mãe está em casa, ela o coloca de castigo. Sabendo-se que ontem à tarde Agilulfo não foi colocado de castigo, qual das seguintes afirmações é certamente verdadeira? A) Agilulfo recebeu advertência ontem. B) Agilulfo não recebeu advertência ontem. C) Ontem à tarde a sua mãe estava em casa. D) Ontem à tarde a sua mãe não estava em casa. E) Nenhuma das afirmações acima é certamente verdadeira. 7. Qual é o menor valor de n > 1 para o qual é possível colocar n peças sobre um tabuleiro nn× de modo que não haja duas peças sobre a mesma linha, mesma coluna ou mesma diagonal? As figuras a seguir mostram pares de peças na mesma linha, na mesma coluna e na mesma diagonal em diversos tabuleiros.

• • • • • •

A) 3 B) 4 C) 5 D) 6 E) 7

Page 73: Eureka 2010

Sociedade Brasileira de Matemática

EUREKA! N°32, 2010

11

8. Na figura a seguir, ABCD é um quadrado de lado 4, K pertence ao lado AD, L pertence ao lado AB, M pertence ao lado BC e KLM é um triângulo retângulo isósceles, sendo L o ângulo reto. Então a área do quadrilátero CDKM é igual a A) 6 B) 8 C) 10 D) 12 E) 14

A B

C D

K

L

M

9. Veja o Problema No. 6 do Nível 1. 10. Veja o Problema No. 16 do Nível 1. 11. Considere o número inteiro positivo n tal que o número de divisores positivos do dobro de n é igual ao dobro do número de divisores positivos de n. Podemos concluir que n é A) um número primo B) um número par C) um número ímpar D) um quadrado perfeito E) potência inteira de 2 12. Esmeralda tem cinco livros sobre heráldica em uma estante. No final de semana, ela limpou a estante e, ao recolocar os livros, colocou dois deles no lugar onde estavam antes e os demais em lugares diferentes de onde estavam. De quantas maneiras ela pode ter feito isso? A) 20 B) 25 C) 30 D) 34 E) 45 13. Veja o Problema No. 19 do Nível 1. 14. Seja : →f uma função tal que f(0) = 0, f(1) = 1, f(2) = 2 e f(x + 12) = f(x + 21) = f(x) para todo .∈x Então f(2009) é: A) 0 B) 1 C) 2 D) 3 E) 2009

Page 74: Eureka 2010

Sociedade Brasileira de Matemática

EUREKA! N°32, 2010

12

15. Na figura, CD = BC, 72=∠BAD , AB é o diâmetro e O o centro do semicírculo. Determine a medida do ângulo .DEC∠ A) 36o

B) 42o

C) 54o

D) 63o

E) 18o

A B

C

D

E

O

16. Sabe-se que 2x2 – 12xy + ky2 ≥ 0 para todos x, y reais. O menor valor real de k é A) 9 B) 16 C) 18 D) 27 E) 36 17. Veja o problema No. 15 do Nível 2. 18. Um subconjunto de {1,2,3,…,20} é superpar quando quaisquer dois de seus elementos têm produto par. A maior quantidade de elementos de um subconjunto superpar é: A) 3 B) 4 C) 6 D) 7 E) 11 19. Veja o problema No. 20 do Nível 2. 20. Os círculos C1 e C2, de raios 3 e 4, respectivamente, são tangentes externamente em T. As tangentes externas comuns tocam C1 em P e Q e C2 em R e S. A tangente interna comum em T corta as tangentes externas nos pontos M e N, como mostra a figura. A razão entre as áreas dos quadriláteros MNPQ e MNRS é:

C1

C2

P

Q M

N R

S

T

A) 71 B)

169 C)

43 D)

23 E)

1513

Page 75: Eureka 2010

Sociedade Brasileira de Matemática

EUREKA! N°32, 2010

13

21. Dois carros deixam simultaneamente as cidades A e B indo de uma cidade em direção à outra, com velocidades constantes, e em sentidos opostos. As duas cidades são ligadas por uma estrada reta. Quando o carro mais rápido chega ao ponto médio M de AB, a distância entre os dois carros é de 96 km. Quando o carro mais lento chega ao ponto M, os carros estão a 160 km um do outro. Qual a distância, em km, entre as duas cidades? A) 320 B) 420 C) 480 D) 520 E) 560

22. Seja 8

88=N , em que aparecem 2009 números 8. Agilulfo ficou de castigo: ele deve escrever a soma dos dígitos de N, obtendo um número M; em seguida, deve calcular a soma dos dígitos de M; e deve repetir o procedimento até obter um número de um único dígito. Vamos ajudar Agilulfo: esse dígito é A) 1 B) 2 C) 3 D) 7 E) 8 23. Veja o Problema No. 20 do Nível 1. 24. Veja o Problema No. 18 do Nível 1. 25. Os lados de um triângulo formam uma progressão aritmética de razão t. Então a distância entre o incentro e o baricentro deste triângulo é:

A) t B) 2t

C)

3t

D)

23t

E) faltam dados

GABARITO NÍVEL 1 – (6º. ou 7º. Anos)

1) C 6) C 11) B 16) C 2) B 7) A 12) E 17) D 3) D 8) C 13) E 18) B 4) C 9) C 14) E 19) D 5) D 10) B 15) B 20) C

NÍVEL 2 – (8º. ou 9º. Anos)

1) C 6) C 11) B 16) A 21) E 2) C 7) B 12) C 17) C 22) B 3) C 8) B 13) E 18) D 23) B 4) D 9) C 14) E 19) B 24) A 5) C 10) A 15) B 20) B 25) C

NÍVEL 3 – (Ensino Médio)

1) B 6) E 11) C 16) C 21) C 2) C 7) B 12) A 17) B 22) A 3) D 8) B 13) D 18) E 23) C 4) D 9) C 14) C 19) B 24) B 5) D 10) E 15) C 20) E 25) C

Page 76: Eureka 2010

Sociedade Brasileira de Matemática

EUREKA! N°32, 2010

14

XXXI OLIMPÍADA BRASILEIRA DE MATEMÁTICA Problemas e soluções da Segunda Fase

PROBLEMAS – NÍVEL 1 – PARTE A (Cada problema vale 5 pontos)

01. A figura ao lado mostra castelos de cartas de 1, 2 e 3 andares. Para montar esses castelos, foram usadas 2, 7 e 15 cartas, respectivamente. Quantas cartas serão necessárias para montar um castelo de 5 andares?

02. Numa classe do 6º ano, de cada 11 estudantes, 4 são meninas. Se há 15 meninos a mais que meninas, quantos alunos há na classe? 03. Num curso com duração de cinco dias, a frequência dos alunos foi registrada na tabela abaixo: Cada aluno faltou exatamente dois dias. No dia de menor frequência, de quantos por cento foi o total de faltas? 04. Mariazinha deseja cobrir o tampo de uma mesa retangular de 88 cm por 95 cm colando quadrados de cartolina de lado 10 cm, a partir de um canto, como mostrado na figura. Ela cola os quadrados sem buracos nem superposições, até chegar às bordas opostas. Aí, em vez de cortar as folhas para não ultrapassar as bordas, ela as sobrepõe, formando regiões retangulares com duas folhas de espessura (região cinza) e uma

pequena região retangular com quatro folhas de espessura (região preta). Qual é a área da região coberta por quatro folhas?

Dia de aula 1º dia 2º dia 3º dia 4º dia 5º dia Quantidade de alunos presentes 271 296 325 380 168

Page 77: Eureka 2010

Sociedade Brasileira de Matemática

EUREKA! N°32, 2010

15

05. O número 200920092009... 2009 tem 2008 algarismos. Qual é a menor quantidade de algarismos que devem ser apagados, de modo que a soma dos algarismos que restarem seja 2008? 06. Dizemos que dois ou mais números, com a mesma quantidade de algarismos, são membros da mesma família, quando todos possuem pelo menos um algarismo comum. Por exemplo, os números 72, 32, 25 e 22 pertencem à mesma família, pois todos possuem o algarismo 2, enquanto que os números 123, 245 e 568 não pertencem à mesma família, pois não há um algarismo que apareça nesses três números. Qual é a maior quantidade de membros de uma família, cujos elementos têm três algarismos? PROBLEMAS – NÍVEL 1 – PARTE B (Cada problema vale 10 pontos) PROBLEMA 1 Carlinhos tem folhas iguais na forma de triângulos retângulos de lados 6 cm, 8 cm e 10 cm. Em cada triângulo, o ângulo assinalado opõe-se ao menor lado. Fazendo coincidir lados iguais desses triângulos sobre uma mesa, sem superpor as folhas, ele desenha o contorno de cada figura obtida (linha grossa), como nos exemplos ao lado. O perímetro de uma figura é o comprimento do seu contorno.

a) Qual é a diferença entre os perímetros das figuras 1 e 2 do exemplo? b) Com figuras de três triângulos, qual é o maior perímetro que pode ser obtido? PROBLEMA 2 Esmeralda ia multiplicar um número A de três algarismos por outro número B de dois algarismos, mas na hora de multiplicar inverteu a ordem dos dígitos de B e obteve um resultado 2034 unidades maior. a) Qual era o número A, se os dígitos de B eram consecutivos? b) Qual seria o número A, se os dígitos de B não fossem consecutivos?

Page 78: Eureka 2010

Sociedade Brasileira de Matemática

EUREKA! N°32, 2010

16

PROBLEMA 3 Um campeonato de xadrez de 7 rodadas, com 4 jogos por rodada, tem 8 participantes, cujas pontuações por jogo são as usuais: um ponto por vitória, meio ponto por empate e nenhum ponto por derrota. Cada par de jogadores se enfrenta exatamente uma vez. a) Ao término da terceira rodada, é possível que um grupo de jogadores esteja em primeiro lugar e o restante dos jogadores esteja em segundo lugar? Explique por meio de um exemplo.

b) Ao término da terceira rodada, é possível que todos os jogadores tenham pontuações diferentes? Explique.

PROBLEMAS – NÍVEL 2 – PARTE A (Cada problema vale 5 pontos) 01. Esmeralda tem uma garrafa com 9 litros de uma mistura que tem 50% de álcool e 50% de água. Ela quer colocar água na garrafa de tal forma que apenas 30% da mistura seja de álcool. Quantos litros de água ela irá colocar? 02. Se a, b, c e d são, em alguma ordem, 1, 2, 3 e 4. Qual é o maior valor possível de

ab + bc + cd + da? 03. Dizemos que dois ou mais números, com a mesma quantidade de algarismos, são membros da mesma família, quando todos possuem pelo menos um algarismo em comum. Por exemplo, os números 32, 25 e 22 pertencem à mesma família, enquanto que 123, 245 e 568 não pertencem à mesma família, pois 123 e 568 não pertencem à mesma família. Qual é a maior quantidade de membros de uma família, cujos elementos têm três algarismos? 04. Determine a quantidade de inteiros de dois algarismos que são divisíveis pelos seus algarismos.

05. Na figura abaixo, ABCD e EFGH são quadrados de lado 48 cm. Sabendo que A é o ponto médio de EF e G é o ponto médio de DC, determine a área destacada em cm2.

Page 79: Eureka 2010

Sociedade Brasileira de Matemática

EUREKA! N°32, 2010

17

A B

CD

E

F

G

H

K

L

PROBLEMAS – NÍVEL 2 – PARTE B (Cada problema vale 10 pontos) PROBLEMA 1 Sejam m e n dois inteiros positivos primos entre si. O Teorema Chinês dos Restos afirma que, dados inteiros i e j com 0 ≤ i < m e 0 ≤ j < n, existe exatamente um inteiro a, com 0 ≤ a < m⋅n, tal que o resto da divisão de a por m é igual a i e o resto da divisão de a por n é igual a j. Por exemplo, para m = 3 e n = 7, temos que 19 é o único número que deixa restos 1 e 5 quando dividido por 3 e 7, respectivamente. Assim, na tabela a seguir, cada número de 0 a 20 aparecerá exatamente uma vez.

Restos por 7

Restos por 3

0 1 2 3 4 5 6

0

1 19

2

Qual a soma dos números das casas destacadas? PROBLEMA 2 Observe:

(x – r)(x – s) = x2 – (r + s)x + rs

Assim, substituindo x por r e por s, obtemos

0))((0))((

0)(0)(

12

12

2

2

=⋅++−=⋅++−

⇒=++−=++−

++

++

nnn

nnn

srsssrsbrrsrsrra

rsssrsrsrsrr

Page 80: Eureka 2010

Sociedade Brasileira de Matemática

EUREKA! N°32, 2010

18

Somando as duas equações e sendo nnn sbraS ⋅+⋅= , verifica-se que

nnn rsSSsrS −+= ++ 12 )(

Dados 11 =+= bsarS , 2222 =+= bsarS , 533

3 =+= bsarS e

6444 =+= bsarS , determine 55

5 bsarS += .

PROBLEMA 3 Seja N é o ponto do lado AC do triângulo ABC tal que NCAN 2= e M o ponto do lado AB tal que MN é perpendicular a AB . Sabendo que AC = 12 cm e que o baricentro G do triângulo ABC pertence ao segmento MN, determine o comprimento do segmento BG. OBS: Baricentro é o ponto de interseção das medianas do triângulo.

PROBLEMA 4 Um campeonato de xadrez de 7 rodadas, com 4 jogos por rodada, tem 8 participantes, cujas pontuações por jogo são as usuais: um ponto por vitória, meio ponto por empate e nenhum ponto por derrota. Cada par de jogadores se enfrenta exatamente uma vez.

a) Ao término da terceira rodada, é possível que todos os jogadores tenham pontuações distintas?

b) Se no final do campeonato todos os jogadores têm pontuações distintas qual o menor número possível de pontos obtidos pelo primeiro colocado? PROBLEMAS – NÍVEL 3 – PARTE A (Cada problema vale 5 pontos) 01. Veja o problema No. 1 do Nível 2. 02. No triângulo retângulo ABC, ∠A = 90º, AB = 5cm e BC = 9cm. Se I é o incentro de ABC, determine o comprimento do segmento CI. 03. Seja c a maior constante real para a qual

x2 + 3y2 ≥ c⋅(x2 + xy + 4y2). para todos x, y reais. Determine o inteiro mais próximo de 2009⋅c.

Page 81: Eureka 2010

Sociedade Brasileira de Matemática

EUREKA! N°32, 2010

19

04. No programa de auditório Toto Bola, o apresentador Ciço Magallanes dispõe de duas caixas idênticas. Um voluntário da platéia é chamado a participar da seguinte brincadeira: ele recebe dez bolas verdes e dez bolas vermelhas e as distribui nas duas caixas, sem que o apresentador veja, e de modo que em cada caixa haja pelo menos uma bola. Em seguida, o apresentador escolhe uma das caixas e retira uma bola. Se a bola for VERDE, o voluntário ganha um carro. Se for VERMELHA, ele ganha uma banana. A máxima probabilidade que o voluntário tem de ganhar um

carro é igual a nm , em que m e n são inteiros positivos primos entre si. Determine o

valor de m + n. 05. Determine o maior inteiro n menor que 10000 tal que 2n + n seja divisível por 5. PROBLEMAS – NÍVEL 3 – PARTE B (Cada problema vale 10 pontos) PROBLEMA 1 Determine a quantidade de números n = a1a2a3a4a5a6, de seis algarismos distintos, que podemos formar utilizando os algarismos 1, 2, 3, 4, 5, 6, 7, 8, 9 de modo que as seguintes condições sejam satisfeitas simultaneamente: i) a1 + a6 = a2 + a5 = a3 + a4; ii) n é divisível por 9. PROBLEMA 2 Encontre todos os inteiros a > 0 e b > 0 tais que

ba 51134 +=⋅ PROBLEMA 3 Para cada inteiro positivo n, seja { }nxxRxAn =⋅∈= + ; , em que +R é o conjunto dos reais positivos e x é o maior inteiro menor ou igual a x. Determine a quantidade de elementos do conjunto

A1 ∪ A2 ∪ A3 ∪ ... ∪ A2009. PROBLEMA 4 No triângulo ABC, temos ∠A = 120° e 12=BC cm. A circunferência inscrita em ABC tangencia os lados AB e AC, respectivamente, nos pontos D e E. Sejam K e L os pontos onde a reta DE intersecta a circunferência de diâmetro BC. Determine a distância entre os pontos médios dos segmentos BC e KL.

Page 82: Eureka 2010

Sociedade Brasileira de Matemática

EUREKA! N°32, 2010

20

SOLUÇÕES NÍVEL 1 – SEGUNDA FASE – PARTE A

Problema 01 02 03 04 05 06 Resposta 40 55 65 10 392 252 01. Para fazer um novo andar num castelo já construído, precisamos de três cartas para cada andar anterior mais duas para o topo. Assim, a partir do castelo de 3 andares, para fazer o de 4 andares, precisamos de mais 3 3 2 11× + = cartas, num total de 15 + 11 = 26 cartas. Portanto, para fazer o castelo de 5 andares, precisamos de 26 4 3 2 40+ × + = cartas. Solução alternativa: Para acrescentarmos um quarto andar a um castelo de 3 andares, precisamos de 3 cartas para separar a base dos demais andares e 4 pares de cartas para a base, totalizando 3 + 2.4 = 11 cartas a mais. Veja a figura a seguir:

Analogamente, para acrescentarmos um quinto andar a um castelo de 4 andares, precisamos de 4 cartas para separar a base dos demais andares e 5 pares de cartas para a base, totalizando 4 + 2.5 = 14 cartas a mais. Assim, para montar um castelo de 5 andares, precisamos de 15 + 11 + 14 = 40 cartas. Observação: De fato, o acréscimo de um n-ésimo andar necessita de 1n − cartas para apoiar a base anterior, e n pares de cartas para a nova base. Portanto, são acrescentadas 1 2 3 1n n n− + ⋅ = − cartas por andar. 02. Seja x a quantidade de meninas. Assim, a quantidade de meninos é 15x + e a quantidade total de alunos será 2 15x + . Fazendo a proporção, temos:

42 15 11

xx

=+

Resolvendo a equação, obtemos 20x = .

Page 83: Eureka 2010

Sociedade Brasileira de Matemática

EUREKA! N°32, 2010

21

03. Se cada aluno compareceu exatamente três dias, o número total de alunos do

curso é 271 296 325 380 168 1440 480

3 3+ + + +

= = . A menor frequência foi de

168 alunos, num total de 480 – 168 = 312 faltas. Portanto, o percentual de faltas

nesse dia foi 312 0,65 65%480

= = .

04. Na direção da medida 88 cm, Mariazinha irá usar 9 folhas e na direção da medida 95 cm, irá usar 10 folhas. Mariazinha começa colando as folhas sem sobreposição da esquerda para a direita e de cima para baixo (como na figura) e ao chegar às bordas direita e inferior, desloca, respectivamente, 2 cm à esquerda e 5 cm para cima (as regiões em cinza representam as sobreposições de 2 folhas). A região retangular preta é a intersecção dessas duas faixas de sobreposição, logo é coberta por 4 folhas. Sua área é de 10 cm2.

05. No número existem 502 algarismos 2 e 502 algarismos 9. Para retirar a menor quantidade possível de algarismos, devemos tentar deixar a maior quantidade possível de algarismos 2. Porém, a soma de todos os algarismos 2 é 1004. Ainda falta 1004 para completar a soma 2008. Como 1004 9 111 5= × + devemos deixar pelo menos 111 algarismos 9. Porém, é impossível deixar exatamente 111 algarismos 9. Se deixarmos 112 algarismos 9, devemos deixar 500 algarismos 2. Portanto, deve-se retirar no mínimo 2 390 392+ = algarismos.

06. Como todos os membros de uma família devem possuir pelo menos um algarismo comum, a maior quantidade de membros de uma família cujos elementos têm três algarismos é igual ao número de elementos de qualquer conjunto formado

Page 84: Eureka 2010

Sociedade Brasileira de Matemática

EUREKA! N°32, 2010

22

por todos os números de três algarismos que possuem um determinado algarismo em sua representação decimal. O algarismo das centenas não pode ser zero. Vamos contar então todos os números que têm um determinado algarismo a, não nulo, pois há mais deles. Há 9 9 81× = números em que a aparece uma única vez, como algarismo das centenas. Há 8 9 72× = números em que a aparece uma única vez, como algarismo das dezenas (lembre-se que o das centenas não pode ser 0) e há 72 números em que o a aparece uma única vez, como algarismo das unidades. Há 9 números com a na centena e na dezena, menos na unidade, 9 números com a na centena e na unidade, menos na dezena e 8 números com a na dezena e na unidade, menos na centena e um único número formado inteiramente de a. A quantidade total de números em que figura o algarismo não nulo a é 81 + 72 + 72 + 9 + 9 + 8 + 1 = 252. Solução alternativa: Para simplificar o raciocínio, vamos contar quantos números de três algarismos não contêm um algarismo a, não nulo, fixado. Assim, nessa situação, existem 8 escolhas para o algarismo das centenas (não pode ser 0 ou a), 9 escolhas para o algarismo das dezenas (não pode ser a), e 9 escolhas para os algarismos das unidades (não pode ser a). Logo, pelo Princípio Fundamental da Contagem, há 8.9.9 = 648 números que não possuem o algarismo a. Assim, como existem 900 números de 3 algarismos, há 900 – 648 = 252 números que possuem o algarismo a ( 0≠a ). Essa é a maior quantidade de membros que uma família pode ter. Observação: Podemos verificar que a família formada por todos os números de três algarismos que possuem o zero tem 900 9 9 9 171− ⋅ ⋅ = membros. SOLUÇÕES NÍVEL 1 – SEGUNDA FASE – PARTE B PROBLEMA 1

Page 85: Eureka 2010

Sociedade Brasileira de Matemática

EUREKA! N°32, 2010

23

a) O perímetro da primeira figura é 8 6 6 10 6 36+ + + + = e da segunda figura é 10 8 6 8 8 40+ + + + = . Portanto a diferença é 40 36 4− = . b) A figura de maior perímetro é obtida quando fazemos coincidir os dois menores lados de cada um dos triângulos. Isso é mostrado na figura ao lado cujo perímetro é 10 10 10 8 6 44+ + + + = (há outras com o mesmo perímetro). PROBLEMA 2 Seja A o número de três dígitos e 10B x y= + o número de dois dígitos. Portanto, ao trocar a ordem dos dígitos de B , obtemos o número 10y x+ . Montando a equação segundo as condições do problema, temos:

(10 ) (10 ) 9 ( ) 2034A x y A y x A x y+ − + = − = Com isso,

( ) 226 2 113A x y− = = ⋅ Daí, se ,x y são consecutivos, 226A = , caso contrário 113A = . PROBLEMA 3 a) Sim, é possível. Por exemplo (há outros), podem existir quatro jogadores com pontuação 2 e outros quatro com pontuação 1. Fazendo A, B, C, D o primeiro grupo e E, F, G, H o segundo grupo, temos:

1ª Rodada A vence E B vence F C vence G D vence H

2ª Rodada

A empata com B E empata com F C empata com D G empata com H

3ª Rodada

A empata com F B empata com E C empata com H D empata com G

Page 86: Eureka 2010

Sociedade Brasileira de Matemática

EUREKA! N°32, 2010

24

b) Após três rodadas, um jogador pode acumular no máximo 3 pontos. Como as

pontuações são múltiplos inteiros de 12

, os possíveis valores de pontuação após a

terceira rodada são: 1 3 50, ,1, , 2, ,32 2 2

(7 resultados possíveis)

Como existem 8 jogadores e apenas 7 possibilidades, dois jogadores terão pontuações iguais. SOLUÇÕES NÍVEL 2 – SEGUNDA FASE – PARTE A

Problema 01 02 03 04 05 Resposta 06 25 252 14 1704 01. Inicialmente temos 4,5 litros de água e 4,5 litros de álcool. Colocados x litros de

água, para termos 30% de álcool na mistura, basta que 5,4)9(10030

=+ x , então

.6=x 02. É fácil ver que ( ) ( ) ( )( )ab bc cd da b a c b c a a c b d .+ + + = + + + = + +

Suponha sem perda de generalidade que 1a .= . Com isso, { } { } { }1 2 1 3a,c , , ,= ou

{ }1 4, e conseqüentemente { } { } { }3 4 2 4b,d , , ,= ou { }2 3, , respectivamente. Assim os possíveis valores do produto são 21, 24 e 25 e o máximo é 25. 03. O algarismo das centenas não pode ser zero. Vamos contar então todos os números que têm um determinado algarismo x, não nulo, pois há mais deles. Há 9 9 81× = números em que x aparece uma única vez, como algarismo das centenas. Há 8 9 72× = números em que x aparece uma única vez, como algarismo das dezenas (lembre-se que o das centenas não pode ser 0) e há 72 números em que o x aparece uma única vez, como algarismo das unidades. Há 9 números com x na centena e na dezena, menos na unidade, 9 números com x na centena e na unidade, menos na dezena e 8 números com x na dezena e na unidade, menos na centena e um único número formado inteiramente de x. A quantidade total de números em que figura o algarismo não nulo x é 81 + 72 + 72 + 9 + 9 + 8 + 1 = 252

Page 87: Eureka 2010

Sociedade Brasileira de Matemática

EUREKA! N°32, 2010

25

04. Seja BAn += 10 o número de dois dígitos. Se A divide n , então A divide B . Se 5>A , então AB = , pois B não pode ser 0 e AB 210 << . Listemos as possibilidades: Se 1=A então AB pode ser 11, 12, 15. Se 2=A , então AB pode ser 22, 24. Se 3=A , então AB pode ser 33, 36. Se 4=A , então AB pode ser 44, 48. Se 5=A , então AB pode ser 55. Se 6=A , então AB pode ser 66. Se 7=A , então AB pode ser 77. Se 8=A , então AB pode ser 88. Se 9=A , então AB pode ser 99. Logo, o total de números é 3 + 2 + 2 + 2 + 5 = 14. 05. Sejam K a interseção dos lados AD e FG , e L a interseção dos lados AB e EH . Por simetria, veja que KD KF= e AK KG= . Considere FK x= . Dessa forma, 48AK x= − . Usando teorema de Pitágoras no triângulo AFK , temos:

( )22 224 48x x+ = − . Que nos dá 18x = . Agora, veja que os triângulos AFK e ALE são semelhantes. Portanto,

AE ELFK AF

= .

Assim, 32EL = . Para achar a área procurada, basta subtrair a área do quadrado EFGH das áreas dos triângulos AFK e AEL . Portanto a área será 1704.

A B

CD

E

F

G

H

K

L

Page 88: Eureka 2010

Sociedade Brasileira de Matemática

EUREKA! N°32, 2010

26

SOLUÇÕES NÍVEL 2 – SEGUNDA FASE – PARTE B PROBLEMA 1:

0 1 2 3 4 5 6

0 0 15 9 3 18 12 6

1 7 1 16 10 4 19 13

2 14 8 2 17 11 5 20 A resposta é 15 + 8 + 10 + 11 + 12 + 13 = 69. PROBLEMA 2:

62552.5).()( 234 =−+=−+=−+= rssrrssrrsSSsrS

5221.2).()( 123 =−+=−+=−+= rssrrssrrsSSsrS Com isso, encontramos que 4−=+ sr e 13−=rs . Daí, .416524)( 345 =+−=−+= rsSSsrS PROBLEMA 3:

Se BP é uma mediana do triângulo então AP = CP = 6 e PN = 2. Como G é o

baricentro do triângulo então 12

PGGB

= e 12

PNNC

= , assim, pela recíproca do

teorema de Tales, GN é paralelo a BC e 90oB∠ = . Como o triângulo ABC é retângulo então AP = CP = BP = 6. Com isso, BG = 4 e GP = 2.

Page 89: Eureka 2010

Sociedade Brasileira de Matemática

EUREKA! N°32, 2010

27

PROBLEMA 4: a) Após três rodadas, um jogador pode acumular no máximo 3 pontos. Como as pontuações são múltiplos inteiros de ½ , os possíveis valores de pontuação após a terceira rodada são:

0,1/2, 1, 3/2, 2, 5/2, 3 Como existem 8 jogadores e apenas 7 possibilidades, dois jogadores terão pontuações iguais.

b) Se k é a pontuação do primeiro colocado e todas as pontuações são distintas, a soma das pontuações dos oito jogadores será no máximo:

( ) ( ) ( )1 3 5 71 2 3 8 142 2 2 2

k k k k k k k k k + − + − + − + − + − + − + − = −

Como foram disputados exatamente 4 7 28× = pontos, temos 8 14 28k − ≥

Logo, 152

k ≥ + pois as pontuações são múltiplos inteiros de 12

. Basta mostrarmos

um exemplo onde este valor é atingido.

Na tabela abaixo, marcamos na interseção da linha iA com a coluna jA o número

de pontos que iA ganhou na partida disputada contra Aj.

1A 2A 3A 4A 5A 6A 7A 8A Total

1A X 1 1 1 1 1 ½ 0 5+½

2A 0 x 1 1 1 1 1 0 5

3A 0 0 x 1 1 1 1 ½ 4+½

4A 0 0 0 X 1 1 1 1 4

5A 0 0 0 0 X 0 0 0 0

6A 0 0 0 0 1 X ½ 1 2+½

7A ½ 0 0 0 1 ½ x 1 3

8A 1 1 ½ 0 1 0 0 x 3+½

Page 90: Eureka 2010

Sociedade Brasileira de Matemática

EUREKA! N°32, 2010

28

SOLUÇÕES NÍVEL 3 – SEGUNDA FASE – PARTE A

Problema 01 02 03 04 05 Resposta 0069 0006 1339 0033 9993 01. [RESPOSTA: 0069] SOLUÇÃO:

0 1 2 3 4 5 6

0 0 15 9 3 18 12 6

1 7 1 16 10 4 19 13

2 14 8 2 17 11 5 20 A resposta é 15 + 8 + 10 + 11 + 12 + 13 = 69. 02. [RESPOSTA: 0006] SOLUÇÃO: Pelo teorema de Pitágoras, é imediato que

1425659 222 =∴=−= ACAC .

Seja r o raio do círculo inscrito, como mostrado na figura abaixo. B

A C

I 5

9

r

r r−142

r

5 – r

5 – r

r−142

Como os comprimentos das tangentes ao círculo inscrito partindo de cada vértice são iguais, ficamos com a equação

(5 – r) + )142( r− = 9, de onde obtemos 214 −=r . Novamente pelo teorema de Pitágoras, obtemos:

Page 91: Eureka 2010

Sociedade Brasileira de Matemática

EUREKA! N°32, 2010

29

636)214()214()142( 22222 =∴=++−=−+= CIrrCI . 03. [RESPOSTA: 1339] SOLUÇÃO: Fazendo x = t⋅y, a equação inicial reduz-se a

t2 + 3 ≥ c⋅(t2 + t + 4). Logo, devemos ter (c – 1)t2 + ct + (4c – 3) ≤ 0, para todo t real. Para isto, devemos ter c – 1 < 0 e o discriminante ∆ = c2 − 4⋅(c – 1)⋅(4c – 3) ≤ 0.

Da última inequação, obtemos −15c2 + 28c – 12 ≤ 0, cuja solução é32

≤c ou

56

≥c . Como c < 1, o maior valor possível de c é 2/3. Daí, 2009⋅c = 1339,333... .

04. [RESPOSTA: 0033] SOLUÇÃO: Seja P(a, b) a probabilidade de o voluntário ganhar o carro no caso em que ele tenha colocado a bolas VERDES e b bolas VERMELHAS na caixa 1. Então, necessariamente haverá (10 – a) bolas VERDES e (10 – b) bolas VERMELHAS na caixa 2. Segue que

baa

baabaP

−−−

⋅++

⋅=20

1021

21),( .

Podemos supor, sem perda de generalidade, que a + b ≤ 10, já que as caixas são idênticas. Suponha, ainda, que haja alguma bola VERMELHA na caixa 1. Vejamos o que acontece com essa probabilidade se transferirmos uma bola VERDE da caixa 2 para a caixa 1 e uma bola VERMELHA da caixa 1 para a caixa 2. Ficamos com

baa

baabaP

−−−

⋅+++

⋅=−+20

9211

21)1,1( .

Dessa forma,

020

1121),()1,1( ≥

−−−

+⋅=−−+

bababaPbaP ,

pois a + b ≤ 10.

Page 92: Eureka 2010

Sociedade Brasileira de Matemática

EUREKA! N°32, 2010

30

Assim, o voluntário sabe que, enquanto houver bola VERMELHA na caixa que contém menos bolas, a probabilidade pode ser aumentada, bastando, para isto, que ele troque uma das bolas VERMELHAS desta caixa com uma VERDE da outra. Por isso, para maximizarmos a probabilidade, basta considerarmos o caso em que a caixa 1 contém apenas bolas VERDES e a caixa 2 contém o restante das bolas. Teremos

⋅−

−=

−−=

−−

+=

−−

⋅+=

aaaaaaaP

2051

20102

21

20101

21

2010

21

21)0,(

.

Logo, a probabilidade será máxima quando a for mínimo. Como em cada caixa deve haver pelo menos uma bola, devemos ter a = 1. Neste caso, a probabilidade é:

1914

1951)0,1( =−=P .

Segue que m = 14, n = 19 e m + n = 33. 05. [RESPOSTA: 9993] SOLUÇÃO: Vamos analisar os restos das divisões de 2n e n por 5.

1 2 3 4 5 6 7 8 9 10 11 12 13 14 15 16 17 18 19 20 n 1 2 3 4 0 1 2 3 4 0 1 2 3 4 0 1 2 3 4 0 2n 2 4 3 1 2 4 3 1 2 4 3 1 2 4 3 1 2 4 3 1 2n + n 3 1 1 0 2 0 0 4 1 4 4 3 0 3 3 2 4 2 2 1

Veja que os restos das divisões de 2n por 5 formam uma seqüência de período 4, enquanto que os restos das divisões de n por 5 formam uma seqüência de período 5. Logo, os restos das divisões de 2n + n formam uma seqüência de período 20, dada pela última linha da tabela acima. Dessa forma, tomando os números de 1 a 10000 em intervalos de tamanho 20, o maior n tal que 2n + n deixa resto zero na divisão por 5 é o 13o termo do ultimo intervalo, ou seja, o número 9980 + 13 = 9993. SOLUÇÕES NÍVEL 3 – SEGUNDA FASE – PARTE B PROBLEMA 1: Seja k = a1 + a6 = a2 + a5 = a3 + a4. Temos 3k = a1 + a2 + ... + a6 é múltiplo de 9, uma vez que n é múltiplo de 9. Daí, segue que k é múltiplo de 3. Mas, como os algarismos são distintos, perceba que

Page 93: Eureka 2010

Sociedade Brasileira de Matemática

EUREKA! N°32, 2010

31

1 + 2 + ... + 6 ≤ a1 + a2 + ... + a6 ≤ 4 + 5 + ... + 9 ⇔ 21 ≤ 3k ≤ 39 ⇔ 7 ≤ k ≤ 13. Como k é múltiplo de 3, temos dois casos: k = 9 e k = 12. 1o caso: k = 9. Veja que é suficiente escolhermos a1, a2 e a3, pois a4 = 9 – a3, a5 = 9 – a2 e a6 = 9 – a1. Como os dígitos devem ser distintos, devemos escolher a1, a2 e a3 de modo que haja no máximo um dígito em cada um dos conjuntos {1, 8}, {2, 7}, {3, 6} e {4, 5}. Esta escolha pode ser feita da seguinte forma: • Escolhemos três dos quatro conjuntos: 4 maneiras; • Em cada um dos três conjuntos acima, escolhemos um dos dois dígitos: 23 = 8 maneiras; • Permutamos os dígitos escolhidos: 3! = 6 maneiras. Logo, o total de números, neste caso, é igual a 4×8×6 = 192. 2o caso: k = 12. Neste caso, os dígitos a1, a2 e a3 devem ser escolhidos do conjunto {3, 4, 5, 7, 8, 9} de modo que haja no máximo um dígito em cada um dos conjuntos {3, 9}, {4, 8} e {5, 7}. Esta escolha pode ser feita da seguinte maneira: • Em cada um dos três conjuntos acima, escolhemos um dos dois dígitos: 23 = 8 maneiras; • Permutamos os dígitos escolhidos: 3! = 6 maneiras. Logo, o total de números, neste caso, é igual a 8 × 6 = 48. O total de números é, portanto, 192 + 48 = 240. PROBLEMA 2: Analisando a equação módulo 5, obtemos ( ) ( )4 3 1 5 3 4 5a amod mod⋅ ≡ ⇔ ≡ .

Mas os valores de 3 5a mod são periódicos de período 4:

a 0 1 2 3 4 5 6 73ª mod 5 1 3 4 2 1 3 4 2 Assim, concluímos que ( )3 4 5 2 4a mod a t≡ ⇔ = + para t .∈ Agora, analisando a equação módulo 3, obtemos

( ) ( ) ( )11 5 0 3 1 1 3bb mod mod+ ≡ ⇔ − ≡ o que ocorre se, e só se, b é par. Portanto a

Page 94: Eureka 2010

Sociedade Brasileira de Matemática

EUREKA! N°32, 2010

32

e b são ambos pares, digamos a= 2c e b = 2d para dois inteiros positivos c, d. Assim,

( )2 24 3 11 5 2 3 5 11a b c d⋅ = + ⇔ ⋅ − =

( )( )2 3 5 2 3 5 11c d c d⇔ ⋅ − ⋅ + =

2 3 5 12 3 5 11

c d

c d

⋅ − =⇔

⋅ + =

3 3 2 22 25 5

c

d

a cb d

= = ⋅ =⇔ ⇔

= ⋅ ==

Assim, a única solução é: ( ) ( )2 2A,B ,= PROBLEMA 3:

Vamos fazer o gráfico da função xxxf ⋅=)( . Para cada k natural, se

1k x k≤ ≤ + , temos kx = . Logo, o gráfico de f é formado por segmentos de reta y = k⋅x, como mostra a figura ao lado: Assim, para um n fixo, a equação f(x) = n tem no máximo uma solução. Portanto, a quantidade de elementos de

A1 ∪ A2 ∪ A3 ∪ ... ∪ A2009 é igual à quantidade de inteiros n, tais que 1 ≤ n ≤ 2009, para os quais f (x) = n admite solução, isto é, os n tais que

f (k) = k2 ≤ n < k(k + 1) = k2 + k, para algum k ∈ N.

1 2 3

1

2

6

4

PROBLEMA 4 Vamos mostrar inicialmente que BL e CK são as bissetrizes dos ângulos B e C do

ABC.∆ Para isto, sejam K´ e L´ as intersecções das bissetrizes de C e B com a circunferência de diâmetro BC , como na figura. Seja ainda I o incentro de ABC∆ e β e γ as medidas de B e C , respectivamente, de modo que 60 .β + γ = °

Page 95: Eureka 2010

Sociedade Brasileira de Matemática

EUREKA! N°32, 2010

33

A E´

I F

B

D´ G K´

C

120º

γ /2 γ /2 β /2

β /2

γ /2

Sejam D e E as intersecções de K´ L com os lados AB e AC do triângulo. Para mostrar que K´ L KL,= basta mostrar que E e D são as projeções ortogonais de I aos lados AC e AB . Como BC é diâmetro, temos que BL C é reto, assim se mostrarmos que o quadrilátero IE´ L C é cíclico, provaremos que IE C é reto, e analogamente para D´. Denote por F e G os encontros das bissetrizes de C e B com os lados opostos.

Temos ( ) ( ) ( ) ( ) 302 2 2

m GIC m F IB m AFC m F BI .γ β β + γ= = − = β + − = = ° da

mesma forma, temos ( ) ( ) ( )m GE´L m BGA m GL E= − 302 2 2β γ β + γ

= + γ − = = °

pois ( ) ( ) ( )m GL E m BL K´ m BCK´= = já que ambos os ângulos subtendem o

mesmo arco BK´. Assim, ( ) ( )m GE L m GIC ,= provando que IE´L´C é cíclico.

Sendo O o ponto médio de BC, temos

( ) ( ) ( )180m KOL m LOC m KOB= ° − − 180 120= ° − β − γ = °

Assim a distância pedida é ( )

2

m LOKLO cos⋅ 60 3

2BC cos cm.= ⋅ ° =

Page 96: Eureka 2010

Sociedade Brasileira de Matemática

EUREKA! N°32, 2010

34

XXXI OLIMPÍADA BRASILEIRA DE MATEMÁTICA Problemas e soluções da Terceira Fase

TERCEIRA FASE – NÍVEL 1 PROBLEMA 1 A sequência 121, 1221, 12221, ... contém todos os números da forma 12221

2 dígitos

…n

. A

quantidade de dígitos 2 indica a posição do número na sequência. Por exemplo, o número 122222221 é o sétimo termo da sequência. a) Dentre os 2009 primeiros termos da sequência, quantos são divisíveis por 3? b) Qual é o menor número múltiplo de 1001 da sequência? PROBLEMA 2 O hexágono regular ABCDEF tem área de 12 cm2. a) Traçando segmentos a partir de um vértice, o hexágono ABCDEF foi repartido em 4 triângulos, conforme figura. Calcule as áreas desses triângulos.

b) Usando os quatro triângulos em que foi dividido o hexágono, podemos montar o retângulo PQRS, na figura. Qual é a área desse retângulo?

Page 97: Eureka 2010

Sociedade Brasileira de Matemática

EUREKA! N°32, 2010

35

PROBLEMA 3 As casas de um tabuleiro 4 × 4 devem ser numeradas de 1 a 16, como mostrado parcialmente no desenho, formando um Quadrado Mágico, ou seja, as somas dos números de cada linha, de cada coluna e de cada uma das duas diagonais são iguais.

a) Que números devem ser escritos no lugar de X e de Y? b) Apresente o Quadrado Mágico completo na sua folha de respostas. PROBLEMA 4 Carlinhos tem várias peças formadas por quatro quadradinhos de lado unitário, na forma de L:

Ele forma figuras maiores com essas peças, fazendo coincidir um ou mais lados dos quadradinhos, como no exemplo, em que foram usadas duas dessas peças, fazendo coincidir um lado unitário. Não é permitido formar buracos nas figuras.

Permitido Não permitido a) Desenhe uma figura cujo perímetro é 14. b) Descreva como formar uma figura de perímetro 2010. c) É possível formar uma figura de perímetro ímpar? Justifique sua resposta. PROBLEMA 5 Um dominó é formado por 28 peças diferentes. Cada peça tem duas metades, sendo que cada metade tem de zero a seis pontos:

Page 98: Eureka 2010

Sociedade Brasileira de Matemática

EUREKA! N°32, 2010

36

Esmeralda coloca 4 peças de dominó dentro de um estojo, respeitando as regras do jogo, isto é, peças vizinhas se tocam em metades com as mesmas quantidades de pontos. Caso seja possível guardar as quatro peças no estojo, dizemos que o conjunto de quatro peças é precioso.

Por exemplo, a figura acima mostra as maneiras de guardar o conjunto precioso

formado pelas peças , , , . a) Mostre que um conjunto precioso não pode conter duas peças duplas. A figura abaixo mostra as peças duplas.

b) Quantos conjuntos preciosos contêm uma peça dupla? c) Determine a quantidade total de conjuntos preciosos.

Page 99: Eureka 2010

Sociedade Brasileira de Matemática

EUREKA! N°32, 2010

37

TERCEIRA FASE – NÍVEL 2 PRIMEIRO DIA PROBLEMA 1 Veja o problema No. 5 do Nível 1. PROBLEMA 2 Seja A um dos pontos de interseção de dois círculos com centros X e Y. As tangentes aos círculos em A intersectam novamente os círculos em B e C. Seja P o ponto de plano tal que PXAY é um paralelogramo. Prove que P é o circuncentro do triângulo ABC. PROBLEMA 3 Prove que não existem inteiros positivos x e y tais que x3 + y3 = 22009. SEGUNDO DIA PROBLEMA 4 Resolva, em números reais, o sistema

.1

111

=

+=+=+

xyzx

zz

yy

x

PROBLEMA 5 Uma formiga caminha no plano da seguinte maneira: inicialmente, ela anda 1cm em qualquer direção. Após, em cada passo, ela muda a direção da trajetória em 60o para a esquerda ou direita e anda 1cm nessa direção. É possível que ela retorne ao ponto de onde partiu em

(a) 2008 passos? (b) 2009 passos?

1 cm

1 cm

1 cm

60°

60°

Page 100: Eureka 2010

Sociedade Brasileira de Matemática

EUREKA! N°32, 2010

38

PROBLEMA 6 Seja ABC um triângulo e O seu circuncentro. As retas AB e AC cortam o circuncírculo de OBC novamente em BB ≠1 e CC ≠1 , respectivamente, as retas BA e BC cortam o circuncírculo de OAC em AA ≠2 e CC ≠2 , respectivamente, e as retas CA e CB cortam o circuncírculo de OAB em AA ≠3 e BB ≠3 , respectivamente. Prove que as retas A2A3, B1B3 e C1C2 passam por um mesmo ponto. TERCEIRA FASE – NÍVEL 3 PRIMEIRO DIA PROBLEMA 1 Esmeralda escreve 20092 números inteiros em uma tabela com 2009 linhas e 2009 colunas, colocando um número em cada casa da tabela. Ela soma corretamente os números em cada linha e em cada coluna, obtendo 4018 resultados. Ela percebeu que os resultados são todos distintos. É possível que esses resultados sejam todos quadrados perfeitos? PROBLEMA 2 Considere um primo q da forma 2p + 1, sendo p > 0 um primo. Prove que existe um múltiplo de q cuja soma dos algarismos na base decimal é menor ou igual a 3. PROBLEMA 3 São colocadas 2009 pedras em alguns pontos (x, y) de coordenadas inteiras do plano cartesiano. Uma operação consiste em escolher um ponto (a, b) que tenha quatro ou mais pedras, retirar quatro pedras de (a, b) e colocar uma pedra em cada um dos pontos

(a, b – 1), (a, b + 1), (a – 1, b), (a + 1, b).

Mostre que, após um número finito de operações, cada ponto terá no máximo três pedras. Além disso, prove que a configuração final não depende da ordem das operações.

Page 101: Eureka 2010

Sociedade Brasileira de Matemática

EUREKA! N°32, 2010

39

SEGUNDO DIA PROBLEMA 4 Mostre que existe um inteiro positivo n0 com a seguinte propriedade: para qualquer inteiro 0n n≥ é possível particionar um cubo em n cubos menores. PROBLEMA 5 Seja ABC um triângulo e O seu circuncentro. As retas AB e AC cortam o circuncírculo de OBC novamente em BB ≠1 e CC ≠1 , respectivamente, as retas BA e BC cortam o circuncírculo de OAC em AA ≠2 e CC ≠2 , respectivamente, e as retas CA e CB cortam o circuncírculo de OAB em AA ≠3 e BB ≠3 , respectivamente. Prove que as retas A2A3, B1B3 e C1C2 passam por um mesmo ponto. PROBLEMA 6 Seja n > 3 um inteiro fixado e x1, x2, …, xn reais positivos. Encontre, em função de n, todos os possíveis valores reais de

1112

1

432

3

321

2

21

1

xxxx

xxxx

xxxx

xxxx

xxxx

nn

n

nnn

n

n +++

++++

+++

+++

++ −−−

SOLUÇÕES DA TERCEIRA FASE – NÍVEL 1 PROBLEMA 1: SOLUÇÃO DE PEDRO HENRIQUE ALENCAR COSTA (FORTALEZA – CE) (a) Um número divisível por 3 tem a soma de seus algarismos como múltiplo de 3. Assim, o primeiro termo múltiplo de 3 é 1221, pois 1 + 2 + 2 + 1 = 6, que é múltiplo de 3. O próximo é o mesmo com 3 algarismos 2 a mais. Então, para saber quantos

múltiplos de 3 escritos dessa forma existem até n, fazemos: 2 1.3

n −+ Sendo n=

2009, fica: 2009 2 20071 1 669 1 670.3 3

−+ = + = + =

(b) Vejamos inicialmente um exemplo de como multiplicar por 1001. Temos 1001 vezes 80 = 80080, pois:

Page 102: Eureka 2010

Sociedade Brasileira de Matemática

EUREKA! N°32, 2010

40

7 a lg arismos

00800080

0080080

+

O primeiro termo da sequencia que é múltiplo de 1001 possui 7 algarismos, sendo ele desta forma 1222221, que é igual a 1221 × 1001, pois:

12211221

1222221

+

É fácil verificar que os termos anteriores não são múltiplos de 1001. PROBLEMA 2: SOLUÇÃO DE ANA BEATRIZ MOTTA ARAGÃO CORTEZ (CAMPINAS – SP) a)

A B

G C F

E D

Seja G o centro do hexágono. A área GDE e GCD é igual a de AFE. Tomando a figura como desenho representativo, podemos dividir o hexágono em seis figuras de áreas iguais: AFE; AGE; GDE; GCD; AGC; ABC. Sabendo que sua área é de 12 cm2, dividimos-na por 6 (número de partes em que o hexágono foi fracionado; assim, cada fração tem 2 cm2 de área (12 cm2 : 6). Para calcularmos a área dos triângulos pedidos, é só fazer:

22cmAFE → 2 2 22cm 2cm 4cmAED AGE+ EGD→ → + → 2 2 22cm 2cm 4cmADC AGC+ GCD→ → + →

22cmABC →

Page 103: Eureka 2010

Sociedade Brasileira de Matemática

EUREKA! N°32, 2010

41

Temos então a área dos dois triângulos iguais AFE e ABC como 22cm (cada um) e a área dos outros dois triângulos iguais AED e ADC como 24cm (cada um), totalizando 212cm . Obs. Há outras formas de resolver o problema com este mesmo raciocínio. Poderíamos dividi-lo em 3 losangos, ou 12 pequenos triângulos por exemplo. b) Dividimos a figura, com um raciocínio parecido com o da letra a).

T

Q S

W U

V

P

R

Cada triângulo acima possui a mesma área. Utilizando a informação de que o triângulo em questão (SVR ou PQU) possui área de 2cm2, calculamos a área do quadrilátero multiplicando 22cm pelo número em que foi fracionada a figura, o que dá 2 22cm 8 16cm ,⋅ = que é a área do retângulo PQRS. PROBLEMA 3: SOLUÇÃO DE DIMAS MACEDO DE ALBUQUERQUE (FORTALEZA – CE) a) Veja os quadrados mágicos:

14 11 5 X

8

12 3

Y

a1 a2 a3 a4

a5 a6 a7 a8

a9 a10

a13

a11 a12

a14 a15 a16

=

Vendo-os, posso afirmar que a soma total do quadrado é 1 2 16a a a+ + ⋅⋅ ⋅ + o que equivale a 1 + 2 + ⋅⋅ ⋅ + 16 que é igual a ( )16 17 2 136.⋅ ÷ = Sabendo que em cada

Page 104: Eureka 2010

Sociedade Brasileira de Matemática

EUREKA! N°32, 2010

42

linha a soma é a mesma, a soma de cada uma delas será 136 4 34÷ = . Como em cada linha, coluna e diagonal a soma será 34 os valores de X e Y serão:

( )( )

34 14 11 5 34 30 4

34 14 8 3 34 25 9.

X

Y

= − + + = − =

= − + + = − =

b) Vamos denominar os espaços vazios do quadrado de: 1 2 3 4 5 6 7 8, , eb ,b b ,b ,b ,b b b como mostra a figura:

14 11 5 4

b1 8 b2 b3

12 b4

b6

3 b5

b7 b8 9

L1

L2

L3

L4

C1 C2 C3 C4

D2

D1

Sabendo que em cada linha, coluna ou diagonal a soma é 34, temos as seguintes equações:

3 5 22b +b = (as raízes só podem ser 15 e 6, pois alguns dos números dos outros pares já aparecem).

2 8 26b +b = (as raízes só podem ser 16 e 10, pois alguns dos números dos outros pares já aparecem).

4 7 15b +b = (as raízes só podem ser 13 e 2, pois alguns dos números dos outros pares já aparecem).

1 6 8b +b = (as raízes só podem ser 7 e 1, pois se fossem 6 e 2 não daria certo, pois o 2 já aparece em 4 7oub b ). Sendo assim, na linha 3 a única combinação qua dá certo é 4 13b = e 5 6,b = caso fossem valores diferentes a soma da linha não daria 34. Tendo descoberto esses dois valores eu posso descobrir os outros: Se 3b não é 6, só pode ser 15. Se 7b não é 13, só pode ser 2. Na linha 2 a única combinação que dá certo é 1 1b = e 2 10,b = pois caso fossem outros valores a soma não daria 34.

Page 105: Eureka 2010

Sociedade Brasileira de Matemática

EUREKA! N°32, 2010

43

Tendo descoberto esses outros dois valores posso descobrir mais outros: Se 1 1b = ,

6b só pode ser 7. Logo se 6b é 7 e 7b é 2, 8b só pode ser 16. Sabendo todos os valores desconhecidos, o quadrado mágico completo é assim:

14 11 5 4

8

12 3

9

1 10 15

13 6

7 2 16

PROBLEMA 4: SOLUÇÃO DE ISABELLA AYRES PINHEIRO DE LIMA (GOIÂNIA – GO) a)

P =14

b) Primeiro, vamos utilizar figuras de perímetro 12, nas ‘pontas’ da figura:

Esses dois lados estarão no meio da figura, e por isso, não serão contados, ou seja, o perímetro que essa figura vai ocupar na “grande” figura será de apenas 10. Como são duas desses figuras (nas ‘pontas”), já conseguimos 20 de perímetro dos 2010 que precisamos. Agora colocamos figuras de perímetro 14 no “meio”

⋅ ⋅ ⋅

Como 4 lados de cada figura estarão no meio da grande figura, cada uma delas ocupará 10, no perímetro 2010. Teremos que usar 199 destas figuras de perímetro 14, no meio; e 2 figuras de perímetro 12, nas pontas. Ao todo: 2 10 199 10 2010× + × = .

Page 106: Eureka 2010

Sociedade Brasileira de Matemática

EUREKA! N°32, 2010

44

c) Não é possível formar uma figura de perímetro ímpar, porque uma simples peça

tem perímetro par e, toda vez que adicionamos outra peça, o perímetro aumentou em 10 – 2. (número de lados usados na colagem), que é sempre par. PROBLEMA 5 Veja a solução do problema 1 do nível 2. SOLUÇÕES DA TERCEIRA FASE – NÍVEL 2 PROBLEMA 1: SOLUÇÃO DE VINÍCIUS CANTO COSTA (SALVADOR – BA) a) Supondo o contrário, isto é, que seja possível um conjunto precioso com 2 peças duplas, elas estariam intercaladas por uma peça, pois caso contrário, elas se encaixariam e isto não é possível pois não tem números em comum e isto não seria de acordo com a regra. Assim, as peças estariam arrumadas dessa forma:

X

X Y

Y ? ?

? ?

e as outras duas como iriam se encaixar com as peças duplas de X e Y, seriam da forma

X

Y

Mas isto é um absurdo, pois não existem peças iguais no jogo (c.q.d). b) Se formarmos um conjunto precioso com uma peça dupla, ele seria organizado dessa forma, seguindo as regras do jogo:

Page 107: Eureka 2010

Sociedade Brasileira de Matemática

EUREKA! N°32, 2010

45

X

X Y

Z X Z

X Y

Logo, se nós escolhermos as peças duplas e as que não têm contato com ela, nós formamos o conjunto: Apenas pegamos a peça com o número da peça dupla e um dos números da que não é dupla e a outra com o número da peça dupla e o outro número da que não é dupla e organizamos da maneira certa, que é única, como

podemos observar. Logo, a quantidade será 6

7 105,2

⋅ =

pois são 7 peças duplas

e a outra peça deve ter números diferentes entre si e da peça dupla também, logo, são 2 números para escolher em 6, já que uma não pode ser usada. c) A quantidade total de conjuntos preciosos será a quantidade que inclui uma peça dupla mais a que não tem esse tipo de peça. Já temos pelo item b) que com peça dupla é 105. Basta contar os conjuntos sem peça dupla. Esses conjuntos serão da forma

X

W Y

Z W Z

X Y

com todos os números diferentes dois a dois. Repare que para cada conjunto de 4 números de 0 a 6 temos 3 conjuntos preciosos que seriam:

W

X

W Y

Z W Z

X Y X

W Z

Y W Y

X Z X

Y Z

Y W

X Z

, , 1º. 2º. 3º.

Page 108: Eureka 2010

Sociedade Brasileira de Matemática

EUREKA! N°32, 2010

46

Logo os conjuntos preciosos sem peça dupla totalizam 7

3,4

que são as maneiras

de escolher 4 números dentre 7 vezes 3. Assim, 7 7 6 5 4 3 2 1105 3 105 3 105 7 5 3 105 105 2104 4 3 2 1 3 2 1

⋅ ⋅ ⋅ ⋅ ⋅ ⋅+ ⋅ = + ⋅ = + ⋅ ⋅ = + = ⋅ ⋅ ⋅ ⋅ ⋅ ⋅

é a

quantidade total de conjuntos preciosos. PROBLEMA 2: SOLUÇÃO DE FRANCISCO MARKAN NOBRE DE SOUZA FILHO (FORTALEZA – CE)

X X1

B

A

X Y

C

P α

α β

β

90° – β 180° – 2α

Se P é circuncentro de ABC , então ele deve ser a interseção entre as mediatrizes dos segmentos AB e AC. Como AB e AC são tangentes às circunferências,

90B AY C AX .= = ° Esses dois ângulos têm B AC em comum e portanto BAX C AY ,= que chamarei de α :

90

90

BAX C AY

BAC

X AY

= = α

= ° − α

= ° + α

Page 109: Eureka 2010

Sociedade Brasileira de Matemática

EUREKA! N°32, 2010

47

Da última igualdade, como PXAY é paralelogramo, temos 90P X A .= ° − α Por outro lado, como o triângulo BXA é isósceles ( )XA XB ,= temos

180 2AX B ,= ° − α ou seja, PX é bissetriz do ângulo AX B. Usando mais uma vez que BXA é isósceles, PX também é a mediana e altura relativa ao lado AB. Assim, PX é a mediatriz do segmento AB. Pela mesma rzão, PY é a mediatriz do segmento AC, o que conclui a prova. PROBLEMA 3: SOLUÇÃO DE LUCAS CAWAI JULIÃO PEREIRA (CAUCAIA – CE) Para provarmos o que o enunciado quer, basta analisar a equação módulo 7. Queremos descobrir, então, quais os restos que um cubo qualquer i3 deixa na divisão por 7. Conseguimos isso elevando ao cubo os possíveis restos que um número qualquer deixa por 7, que são 0, 1, 2, 3, 4, 5, e 6. Concluímos que os possíveis restos que um cubo pode deixar são 0, 1 e 6. Agora analisemos as potências de 2 módulo 7.

( )( )( )( )

1

2

3

4

2 2 mod7

2 4 mod7

2 1 mod7

2 2 mod7

Encontramos o período, então dividimos 2009 por 3. Como o resto dessa divisão é 2, logo ( )20092 4 mod7≡ . Daí encontramos um absurdo já que qualquer soma dos possíveis restos de dois cubos jamais será 4. Logo 3 3 20092x y+ = não possui solução nos inteiros. PROBLEMA 4: SOLUÇÃO DE ANDRE MACIEIRA BRAGA COSTA (BELO HORIZONTE – MG) Olhemos para a primeira equação:

1 1x yy z

+ = + → Vamos substituir o termo 1z

em termos das variáveis x e y.

Da segunda equação, temos: 11xyz xyz

= → = (substituímos na primeira equação)

1x y xyy

+ = + (multiplicamos tudo por y)

2 21xy y xy+ = + (reordenando) ( )2 1 1 0y x xy+ − − = (resolvemos pela forma de Bháskara)

Page 110: Eureka 2010

Sociedade Brasileira de Matemática

EUREKA! N°32, 2010

48

1 1y = e 21

1y .

x= −

+

Separemos em dois casos: 1) Caso y = 1. Substituímos na segunda igualdade:

1 1 1 11y z zz x z x

+ = + → + = +

11 1

11

xyzx zx z

zx

= ⋅ ⋅ =

⋅ = =

Como xyz = 1, temos 1zx

= e daí

11 z zz

+ = + (multiplicando por z. 0z ≠ )

22 1 0z z− − = (Resolvendo pela fórmula de Bháskara)

1 1z = e 212

z = − .

Da equação 11 1z ,x x

= = e 2 2x .= −

Nesse caso, temos as soluções (1, 1, 1) e 12 12

, , −

.

2) Caso ( )1 11

y xx

= − ≠ −+

Substituímos na segunda equação: 1 1

1x z

x − = +

11

xx z

− =+

1xzx+

= − ; note que, nesse caso, 1 1 1 1 111 1

x xy z ,z x x x x x

+= + = − − = − = − + = +

+ + e

a segunda igualdade também é satisfeita.

Page 111: Eureka 2010

Sociedade Brasileira de Matemática

EUREKA! N°32, 2010

49

Resposta:

( ) 1111 2 12

, , ; , , − −

e todas as triplas da forma 1 11

aa, , ,a a

+ − − + com

{ }0 1a , .∈ − −

Obs. 12 12

, , − − →

esta solução é da forma 1 11

aa, , .a a

+ − − +

PROBLEMA 5: SOLUÇÃO DE DANIEL EITI NISHIDA KAWAI (SÃO PAULO – SP)

120°

120°

60°

60°

120°

120°

120°

Temos o diagrama infinito de possíveis posições em que a formiga pode chegar.

a) Resposta: Sim. Para voltar à posição inicial em 2008 passos, basta seguir as instruções abaixo:

Page 112: Eureka 2010

Sociedade Brasileira de Matemática

EUREKA! N°32, 2010

50

Posição inicial

Posição inicial

Dê 333 voltas no hexágono (isso dará 1998 passos e depois siga o trajeto abaixo, em que são usados 10 passos e volta-se à posição inicial. No total, s formiga dará 2008 passos e voltará à posição inicial b) Resposta: Não. Pinte as posições da figura inicial de preto e branco alternadamente. A formiga começa em uma bolinha preta e toda bolinha preta está cercada de bolinhas brancas e toda bolinha branca está cercada de bolinhas pretas. Assim, quando a formiga anda um númro par de passos, ela sempre termina em uma bolinha preta e quando anda um número ímpar de passos, ela sempre terminará em uma bolinha branca. Como 2009 é ímpar, a formiga, se começar em uma bolinha preta, sempre terminará em uma bolinha branca; logo, será impossível voltar à posição inicial depois de 2009 passos. PROBLEMA 6: SOLUÇÃO DE DANIEL EITI NISHIDA KAWAI (SÃO PAULO – SP)

A

O1

B

O

C C1 M X

Page 113: Eureka 2010

Sociedade Brasileira de Matemática

EUREKA! N°32, 2010

51

Temos AO BO CO≅ e 1 1 1 1 1BO CO C O OO .≅ ≅ Além disso, 1C BO e 1C CO são ângulos inscritos do mesmo arco de circunferência

11 1 1 2

C MOC MO C BO C CO .⇒ = = Como ACO∆ é isósceles (já que AO CO≅ ),

1 1 1 1C AO C CO C AO C BO.= ⇒ = Como ABO∆ é isósceles,

1 1 1 1 1BAO ABO BAO C AO ABO C BO BAC ABC ABC= ⇒ + = + ⇒ = ⇒ ∆ é isósceles

1 ABC m .→ ∈ De maneira análoga, 2 BCA m ,∈ 3 BCA m ,∈ 1 ACB m ,∈ 3 ACB m∈ e

2 2 3AB BCC m A A m ,∈ ⇒ ⊂ 1 3 ACB B m⊂ e 1 2 ABC C m .⊂ Como AB ACm ,m e BCm se

encontram em 0, as retas 2 3 1 3A A ,B B e 1 2C C passam por um mesmo ponto. Obs. XYm é mediatriz do segmento XY . SOLUÇÕES DA TERCEIRA FASE – NÍVEL 3 PROBLEMA 1: SOLUÇÃO DE HUGO FONSECA ARAÚJO (RIO DE JANEIRO – RJ) Sim, é possível. Considerando a tabela como uma matriz ija tome 0ija = , para

1 2008i, j ,≤ ≤ e ( ) ( )2 22009 20092 1 2i , ,ia i ,a i ,= − = para 1 2008i .≤ ≤

12

32

22 42 ... d

Então já temos 4016 fileiras cujas somas são quadrados perfeitos e distintos. As duas que faltam são a última linha e última coluna. Seja 2009 2009a , d .= Queremos que

2 2 2 2

2 2 2 2

1 3 40152 4 4016

... d b,

... d c+ + + + =

+ + + + =

onde b, c são distintos e maiores que 4016. Subtraindo as equações, temos:

Page 114: Eureka 2010

Sociedade Brasileira de Matemática

EUREKA! N°32, 2010

52

2 2 1 2 4016 2008 4017c b ...− = + + + = ⋅ ( )( ) 2008 4017c b c b .⇒ − + = ⋅

Tomando 502 4017 2c = ⋅ + e 502 4017 2b ,= ⋅ − a igualdade acima é satisfeita. Para concluir, tome

( ) ( ) ( )22 2 2 2 2 2 21 3 4015 502 4017 2 1 3 4015d b ... ... .= − + + + = ⋅ − − + + +

Desse modo, ( )22 2 21 3 4015 502 4017 2... d+ + + + = ⋅ −

( )22 2 22 4 4016 502 4017 2... d .+ + + + = ⋅ + PROBLEMA 2: SOLUÇÃO DE MATHEUS SECCO TORRES DA SILVA (RIO DE JANEIRO – RJ) Vamos organizar as idéias. Se 2 5p ,q= = e 10 satisfaz. Se 2p ,> o múltiplo de q só poderá ter soma 2 ou 3, pois se tivesse soma 1, seria uma potência de 10, e como q é primo > 5, q não divide 10n ,n .+∈ Então, devemos conseguir um múltiplo com soma 2 ou 3.

• Múltiplos com soma 2: 10 1a + • Múltiplos com soma 3: 10 10 1a b+ + ( )a b≥ .

Pelo Pequeno Teorema de Fermat, ( ) ( )1

1 210 1 10 1q

q q q−

− ≡ ⇒ ≡ ou ( )1

210 1q

q−

≡ −

Se ( )1 1

2 210 1 10 1q q

q ,− −

≡ − + satisfaz as condições do problema (tem soma dos dígitos 2).

Suponha então que ( ) ( )1

210 1 10 1q

pq q−

≡ ⇒ ≡ ⇒ 10 1qord = ou p.

Se 10 1 9 3 1qord ,q q p ,= ≠ ⇒ = ⇒ = absurdo. Logo, 10qord p.= Nesse caso, vamos tentar um múltiplo com soma 3, isto é, vamos procurar inteiros positivos a e b tais que 10 10 1 0a b+ + ≡ (q).

1 210 10 10 p, ,..., são p resíduos distintos módulo 2 1q p .= + De fato, se

( )10 10x y q≡ com ( ) ( )10 1x yy x p, q ,−< ≤ ≡ 0 x y p ,< − < contradição, pois

qord 10 p= .

Se x∃ tal que ( )10x p q ,≡ tomemos ( )2 10 1 0xa b x q= = ⇒ ⋅ + ≡ e o problema

acaba. Suponha então que x∃ tal que ( )10x p q .≡

Temos então p resíduos para 1 210 10 10 p, ,..., dentre 0 1 2 3 1 1 2, , , ,..., p , p ,..., p.− + Vamos considerar a lista formada por esses p resíduos.

Page 115: Eureka 2010

Sociedade Brasileira de Matemática

EUREKA! N°32, 2010

53

0 não está na lista, pois 2 1 5q p .= + > Se y∃ tal que ( )10 1y q≡ − , teríamos ( )210 1y q≡ e

0 010 2 2 2 2 2qord p p y y kp kp k k k y k p= ⇒ ⇒ = ⇒ ⇒ ⇒ = ⇒ = ⇒

( ) ( )0010 10 10 1kk py p q= ≡ ≡ , absurdo, pois estamos supondo ( )10 1y q .≡ −

Logo, 0 e 2p não entram na lista! Considere os pares ( ) ( ) ( ) ( )1 2 1 2 2 2 3 2 3 1 1, p ; , p ; , p ;... p , p .− − − − +

Eles incluem todos os resíduos que 1 210 10 10 p, ,..., podem assumir. Temos p – 1 pares e p resíduos a escolher. Pelo Princípio da Casa dos Pombos, escolheremos dois números do mesmo par. Mas a soma de dois números do mesmo par é 2p(mod q). Logo, x, y∃ com ( )10 10 2x y p q+ ≡ ⇒

( )10 10 1 0x yA q A= + + ≡ ⇒ é múltiplo de q e tem soma dos dígitos 3. PROBLEMA 3: SOLUÇÃO DE RENAN HENRIQUE FINDER (JOINVILLE – SC) Vamos chamar “operação” o ato de tirar 4 pedras de (a, b) e colocar uma pedra em cada um dos pontos ( ) ( ) ( )1 1 1a ,b , a ,b , a,b+ − − e ( )1a,b .+ Não faremos distinção de operações no mesmo ponto que usam pedras diferentes; assim, atentaremos para quantas pedras há em cada ponto, e não quais. Provemos por indução o seguinte resultado, trivial para 4n .≤ Para qualquer n, existe ( )A n tal que, para quaisquer pedras 1 2 np , p ,..., p , não é

possível realizar mais de ( )A n operações. Suponhamos que isso valha para todo k n< para fazer o caso em que temos n pedras. É importante observar que ( )A n depende apenas de n e não da distribuição das pedras. Claramente o centro de massa das pedras é invariante. Logo, podemos fixá-lo como origem (desconsiderando a hipótese de as pedras terem coordenadas inteiras). Dada uma sequência de operações, chamaremos ( )ip t a posição de ip após t

operações , de modo que ( )0ip é a posição inicial de ip .

Note que ( ) ( )2 2 21 1 2 2x , x x x∀ ∈ − + + = +

( ) ( ) ( ) ( )2 2 2 22 2 2 2 2 21 1 1 1 4 4 4a,b , a b a b a b a b a b ⇒∀ ∈ + + + − + + + − + + + = + + Assim, se uma operação em (a, b) move para 1 2 3 4p ,p ,p ,p , com

( ) ( ) ( ) ( ) ( ) ( )1 2 31 1 1 1 1 1p t a ,b ,p t a ,b ,p t a,b+ = + + = − + = + e ( ) ( )4 1 1p t a,b ,+ = −

Page 116: Eureka 2010

Sociedade Brasileira de Matemática

EUREKA! N°32, 2010

54

( ) ( )2 2

1 11 4

n n

i ii i

p t p t .= =

⇒ = − +∑ ∑

Por indução, ( ) ( ) ( )2 2

1

44 2n

i i ii

t tt, p t t i : p t p tn n=

∀ ≥ ⇒∃ ≥ ⇒ ≥∑

Vamos escolher t grande (veremos que ( )239 1t n A n≥ − basta).

Definimos ( )ip p t .= Da invariância do centro de massa, j∃ , ( )jtp t C p, ,n

onde ( ) { }2C p,r x : x p r= ∈ − ≤ é o círculo de centro p e raio r. Isso ocorre

porque, se todas as pedras estivessem em tC p, ,n

que é convexo, seu centro de

massa também estaria, o que significa ( )0 0 t t, C p, p ,n n

∈ ⇔ ≤

absurdo.

Agora vemos as regiões ( )( )( )( )( )( )

( )( )

1

2

3

3 1

6 1

9 1

3 1n

R C p, A n

R C p, A n

R C p, A n

R C p, nA n .

= −

= −

= −

= −

Uma das n + 1 regiões 1 2 1 3 2 1n nR ,R \ R ,R \ R ,...R \ R − e 2n\ R não contém nenhuma

pedra. Como ( ) ( )239 1 3 1tt n A n nA n ,n

≥ − ⇔ ≥ − teremos jtp C p,n

e

2n j n n

tR C p, p R \ Rn

⊂ ⇒ ∉ ⇒

não está vazia. 1R não está vazia porque

1p R .∈ Logo, 1k , k n∃ ≤ ≤ tal que 1k kR \ R − não tem pedras. Assim temos até n – 1 pedras em 1kR − e até n – 1 pedras em 2

kR .− A distância entre uma pedra de 1kR − e uma fora de kR é sempre pelo menos

( )3 1A n − (vide definição). As pedras em 1kR − e as fora de kR se moverão independentemente até que duas delas ocupem a mesma posição. Para que isso

Page 117: Eureka 2010

Sociedade Brasileira de Matemática

EUREKA! N°32, 2010

55

ocorra, pela hipótese de indução, as pedras fora de kR não realizarão mais de ( )1A n − movimentos, bem como as de dentro de 1kR − . Portanto, depois de

( )2 1A n − rodadas, cada pedra se deslocará no máximo ( )1A n − unidades, logo uma pedra fora de kR não poderá ficar no mesmo ponto que uma pedra que estava dentro de 1kR ,− o que torna os dois conjuntos necessariamente independentes.

Assim, basta tomar ( ) ( ) ( )239 1 2 1A n n A n A n= − + − . Isso resolve a primeira parte. Para a segunda parte, comecemos lembrando que, se chegarmos à mesma configuração de duas maneiras diferentes, a igualdade

( ) ( )2 2

1 14 0

n n

i ii i

p t t p= =

= +∑ ∑

diz que o número de operações, t, é igual nas duas maneiras. Para a prova, suponhamos que na configuração inicial, os pontos com 4 pedras ou mais sejam 1 2X ,X ,...e eX . Considere também uma sequência de operações que leva o plano a um estado em que não é possível fazer mais operações. Certamente, ocorreram operações com centro em 1 2X ,X ,... e eX .

Considerando duas sequências de operações 1 2O ,O ,...,Oα e 1 2´ ´ ´O ,O ,...,Oβ que

terminam em uma configuração na qual não é possível fazer mais operações, provaremos que uma é permutação da outra via indução em { }min , ,α β o que resolve o problema. Seja X um ponto em que há mais de quatro pedras no princípio. Seja Oγ a primeira

operação em { }1 2O ,O ,...,Oα com centro em X.

Vamos provar que a sequência de operações 1 2 1 1O ,O ,O ,...,O ,O ,...Oγ γ− γ+ α leva ao

mesmo resultado que 1 2O ,O ,...,O .α Basta provar que 1 1 1O ,O ,...,O ,O ,...Oγ γ− γ+ α é uma sequência de operações válidas, já que cada operação tira o mesmo número de pedras de cada ponto e coloca o mesmo número em cada ponto, independentemente de quando foi realizada, de forma que as operações são comutativas. Pelo mesmo argumento, basta ver que 1 1O ,O ,...,Oγ γ− é uma sequência possível. Mas

começar com Oγ é claramente possível e, da minimalidade de γ , as operações

1O ,...e 1Oγ− têm centro em um ponto diferente de X. Assim, Oγ só pode ter

Page 118: Eureka 2010

Sociedade Brasileira de Matemática

EUREKA! N°32, 2010

56

aumentado o número de pedras nos centros de 1O ,...e 1Oγ− , e não diminuído, o que faz com que toda essa sequência seja possível. Em outras palavras, sem perda de generalidade, 1 1O O Oγ= ⇒ tem centro em X.

Analogamente, podemos supor que, 1'O tem centro em X. Agora, após a realização

da operação com centro em X, { }min ,α β diminui, e vemos que as seqüências

2O ,...,Oα e 2O ´,...,Oβ´ são iguais pela hipótese de indução.

PROBLEMA 4: SOLUÇÃO DE MARLEN LINCOLN DA SILVA (FORTALEZA – CE) Seja {S n= ∈ : é possível particionar um cubo em n cubos menores}. Lema: Se x, y S∈ , então 1x y S.+ − ∈ Prova: Particione o cubo inicial em x cubos menores. Escolha um desses cubos e o particione em y cubos. Daí, o cubo inicial estará particionando em x + y – 1 cubos menores. Claramente, 3n S ,∈ para n 2≥ inteiro. Assim 32 S;∈ portanto, 8 1 7x S ,x x S.∀ ∈ + − = + ∈ Para termos o resultado desejado, basta provarmos que existem 1 2 7a ,a ,...,a S ,∈ tais que ( )mod 7i ja a≡ ,

para 1 7i j≤ < ≤ (basta escolhermos { }0 1 7in max a , i= ≤ ≤ ).

De fato, se { }0 1 7ix n max a , i≥ = ≤ ≤ e ( )mod 7 7j jx a ,x a k S.≡ = + ∈

Seja ( ){ }mod 7S´ x ,x S .= ∈ De forma análoga, se x, y S´∈ então

( )1x y+ − ( )mod 7 S.∈

Claramente 1 6 0, , S ,∈ já que 3 32 3, e 37 S.∈ Logo ( )6 6 1 mod 7 4 S´+ − = ∈ e

( )0 6 1 mod 7 5 S´.+ − = ∈ Então ( )4 0 1 mod 7 3 S'+ − = ∈ e

( ) { }3 0 1 mod 7 2 0 1 2 3 4 5 6S' S´ , , , , , ,+ − = ∈ ⇒ = e o problema está acabado. PROBLEMA 5: SOLUÇÃO DE GUSTAVO LISBOA EMPINOTTI (FLORIANÓPOLIS – SC) Considere uma inversão com respeito ao circuncírculo do ABC.∆ Como o circuncírculo do AOC∆ passa pelo centro de inversão (O), seu inverso é uma reta pelos inversos de A e C. Mas A e C pertencem ao círculo de inversão, de modo que são seus próprios inversos. Ou seja, a reta AC é o inverso do circuncínculo de

AOC∆ . O inverso do ponto 2A é a interseção do inverso do circuncírculo de

AOC∆ –que é AC – com o inverso de AB –que é o circuncírculo do AOB∆ – isto

Page 119: Eureka 2010

Sociedade Brasileira de Matemática

EUREKA! N°32, 2010

57

é, é o ponto 3A . Então 2A e 3A são inversos, logo 2 3A A passa pelo centro de inversão, O. Analogamente, 1 3B B e 1 2C C passam por O, como queríamos. PROBLEMA 6: SOLUÇÃO DE RENAN HENRIQUE FINDER (JOINVILLE – SC) Sendo ( )nr a o resto da divisão do inteiro a por n (i.e., o único número r em ] ]0,n

tal que ( )a r mod n ,≡ definamos ( )na r ax x .= Claramente, a função ( )n*S : + →

dada por:

( )11 1 1

ni

ni i i i

xS x ,...xx x x= − +

=+ +∑ satisfaz ( )1

1 1 2

1n

in

i n

xS x ,...xx x ... x=

> =+ + +∑

Além disso, ( ) ( ) ( ) 12 3 1

1 2 2 1

2111 1 1

nn

n n n

n t tf t S ,t ,t ,t ,...,tt t t t t t

−−

− − −

−= = + +

+ + + + + +

É contínua e tal que ( )13nf = e ( ) ( )0 2 0 1 1

tlim f t n ,→∞

= + − + = de modo que

( )1 nS x ,...x pode assumir qualquer valor no intervalo 13n, .

Por outro lado, se n é

par, ( ) 11 1 12 2 1 2n tg ,t , ,t ,..., ,t

t t = + + +

é contínua e tal que ( )13ng ,= enquanto

( )2t

nlim g t .→∞

= Portanto, ( )1 nS x ,...,x assume todos os valores do intervalo 3 2n n, .

Se n for ímpar, definamos a função

( ) ( ) 2 1 3 11 1 1 12 2 2 2 2 1

n t nh t S ,t , ,t ,..., ,t ,t t t

− −= = + ⋅ + ⋅

+ + +. Temos ( )1

3nh = e

( ) 12

nh t −→ quando t .→ ∞ Segue disso que n pode tomar qualquer valor em

13 2n n, .−

No caso em que n é par, não é difícil resolver o problema se notarmos

que *x x x, y,z .x y z x y +< ∀ ∈

+ + +

De fato: 2

2 1 2

1 11 1 2 2 2 1 2 2 1 2 2 1

k ki i i

i ii i i i i i i i i

x x xx x x x x x x x x

= =− + − − − +

= ++ + + + + +∑ ∑

2 1 2

1 2 1 2 2 1 2

ki i

i i i i i

x x k.x x x x

= − −

< + =+ +∑

Page 120: Eureka 2010

Sociedade Brasileira de Matemática

EUREKA! N°32, 2010

58

Assim, sob a hipótese de n ser par, os valores possíveis de ( )1 nS x ,...x são os

elementos de 12n, .

Para o caso n ímpar, queremos mostrar que a imagem de S é 112

n, −

.

Adaptaremos a idéia usada anteriormente. Veja que essa idéia prova que

5 14

3 4 5 4 5 6 2 1 1 1

32

n n

n n n n n n

x x xx n...x x x x x x x x x x x x

− − − +

−+ + + + <

+ + + + + + + +

Logo, para que se garanta que ( )1 21

2nnS x ,x ,...x ,−

< é suficiente termos

3 1 2 31 2

0 1 2 1 2 3 2 3 4 1 2 3

x x x xx xx x x x x x x x x x x x

+ ++ + ≤

+ + + + + + + +

as condições 0 3x x≥ e 4 1x x≥ implicam essa desigualdade. Se supusermos por

absurdo ( )11

2nnS x ,...x ,−

≥ concluímos que 0 3 1 4x x x x≥ ⇒ > e 1 4 0 3x x x x .≤ ⇒ <

Analogamente, supondo 0 3x x ,≤

1 4 2 5

2 5 3 6

3 4 3 1 3 3 3n n n n

x x x xx x x x x x x x− − −

> ⇒ >> ⇒ >

> ⇒ >

O absurdo é que 0 3 6 9 3 0nx x x x ... x x .≥ > > > > = A suposição 1 4x x≤ pode ser tratada similarmente. Obs: Outra maneira de se fazer o caso 0 3x x≤ é definir 3n ny x ,−= o que dá 3 0y y .≤ Alem disso,

( ) ( )1 11 1

2 2n nn nS x ,...,x S y ,..., y ,− −

< ⇔ < o que já sabemos provar.

Page 121: Eureka 2010

Sociedade Brasileira de Matemática

EUREKA! N°32, 2010

59

XXXI OLIMPÍADA BRASILEIRA DE MATEMÁTICA Problemas e soluções da Primeira Fase – Nível Universitário

PROBLEMA 1 (a) Encontre o valor mínimo da função f : → dada por ( ) ( )x / ef x e x= − (aqui e = 2,71828... é a base do logaritmo natural). (b) Qual destes números é maior: eπ ou eπ ? PROBLEMA 2 Seja ζ ∈ uma raiz de 7 1x ,− com 1.ζ ≠ Existe um polinômio Mônico p de grau 2 com coeficientes inteiros cujas raízes são os números 2 4

1z = ζ + ζ + ζ e 3 5 6

2z .= ζ + ζ + ζ Calcule ( )3p . PROBLEMA 3 A rã Dõ descansa no vértice A de um triângulo equilátero ABC. A cada minuto a rã salta do vértice em que está para um vértice adjacente, com probabilidade p de o salto ser no sentido horário e 1 – p de ser no sentido anti-horário, onde ( )0 1p ,∈ é uma constante. Seja nP a probabilidade de, após n saltos, Dõ estar novamente no vértice A. (a) Prova que, qualquer que seja ( )0 1 1 3nn

p , ,lim P / .→∞

∈ =

(b) Prove que existe ( )0 1 100p , /∈ tal que, para algum 1nn ,P / .∈ = π PROBLEMA 4 Determine a quantidade de números inteiros positivos n menores ou iguais a 31! Tais que 3n n+ é divisível por 31. PROBLEMA 5 Dados os números reais a, b, c, d, considere a matriz

a b c dd a b c

A .c d a db c d a

=

Se ( ) 2 3f x a bx cx dx ,= + + + prove que

Page 122: Eureka 2010

Sociedade Brasileira de Matemática

EUREKA! N°32, 2010

60

( ) ( ) ( ) ( )1 1det A f f i f f i .= − − (Aqui i representa a unidade imaginária.) PROBLEMA 6 Considere a sequência 0 1 2a ,a ,a ,...definida por 0 10 3a ,a /= = π e, para 1n ,≥

( )( )

0 1 1 2 2 01 3 1

n n n nn

a a a a a a ... a aa .

n− −

+

π + + + +=

+

Calcule 31 2

00 2 2 4 8

kk

k

a aa aa ...∞

=

= + + + +∑ .

SOLUÇÕES NÍVEL UNIVERSITÁRIO – PRIMEIRA FASE PROBLEMA 1

a) A derivada da função f é ( ) ( )1 1x ef´ x e ,e

= ⋅ − que se anula apenas para x = e,

sendo negativa para x < e e positiva para x > e. Assim, o valor mínimo de f é f (e) = 0. b) Pelo resultado do item anterior, como eπ ≠ temos que ( ) 0f ,π > logo ( )ee ,π > π

ou seja, ( ) ee .π > π PROBLEMA 2 Um polinômio que satisfaz as condições do enunciado é

( ) ( )21 2 1 2p x x z z x z z .= − + +

72 3 4 5 6

1 211 11

z z .ζ −+ = ζ + ζ + ζ + ζ + ζ + ζ = − + = −

ζ −

4 6 7 5 7 8 7 9 10 2 3 4 5 61 2 3 2z z .= ζ + ζ + ζ + ζ + ζ + ζ + ζ + ζ + ζ = + ζ + ζ + ζ + ζ + ζ + ζ =

Logo ( ) 2 2p x x x= + + e ( )3 14p .= PROBLEMA 3 a) Sejam A, B, e C os vértices do triângulo no sentido anti-horário. Seja nQ (resp.

nR ) a probabilidade de, após n saltos, Dõ estar no vértice B (resp. C). Temos 0 0 01 0P ,Q R= = = e, para todo 0n ,≥

Page 123: Eureka 2010

Sociedade Brasileira de Matemática

EUREKA! N°32, 2010

61

(*) ( )( )( )

1

1

1

1

1

1

n n n

n n n

n n n

P p R pQ

Q p P pR

R p Q pP

+

+

+

= − +

= − + = − +

Dado n natural, seja { }n n n n n n nD max P Q , Q R , R P .= − − − Vamos provar que,

para todo n, { }1 1n nD max p, p . D .+ ≤ − Dado n, há 6 possibilidades para a ordem dos números n n nP ,Q ,R . Vamos analisar o caso n n nP Q R≤ ≤ (os outros 5 casos são análogos). Nesse caso, a maior distância Dn entre dois dos números n n nP ,Q e R é

n nR P .− De (*), obtemos:

( )( ) ( ) ( )( ) ( ){ }1 1 1 1n n n n n n n n n nP Q p R P p Q R max p R P , p R Q+ +− = − − + − ≤ − − − ,

{ }1 nmax p, p . D≤ − , pois n nR P− e n nQ R− têm sinais contrários.

( )( ) ( ) ( )( ) ( ){ }1 1 1 1n n n n n n n n n nQ R p P Q p R P max p Q P , p R P+ +− = − − + − ≤ − − −

{ }1 nmax p, p D ,≤ − ⋅ pois n nP Q− e n nR P− têm sinais contrários.

( )( ) ( ) ( )( ) ( )1 1 1 1n n n n n n n n n nR P p Q R p P Q p R Q p Q P+ +− = − − + − = − − + − ≤

{ } ( ) { } ( ) { }1 1 1n n n n n n nmax p, p R Q Q P max p, p R P max p, p D .≤ − ⋅ − + − = − ⋅ − = − ⋅

Assim, { } { }1 1 1 1 1 1 1 1n n n n n n n nD max P Q , Q R , R P max p, p D ,+ + + + + + += − − − ≤ − ⋅ para

todo 0n ,≥ donde { }( )1 0n

nD max p, p , n .≤ − ∀ ≥

Como 1n n nP Q R , n ,+ + = ∀ ∈

( ) ( ) { }( )21 2 1 03 3 3 3 3

nn n n nn n n nn n

P Q P RP Q R DP P max p, p , n .− + −+ +

− = − = ≤ ≤ ⋅ − ∀ ≥

Como { }0 1 1max p, p ,< − < segue imediatamente que 1 03nn

lim P ,→∞

− = e que

13nn

lim P .→∞

=

b) Para p = 0 teríamos 1nP = quando n é múltiplo de 3 e 0nP = caso contrário.

Por outro lado, tomando 1 100p ,= temos 3 1 1 3kklim P .+→∞

= Em particular, existe

r ∈ tal que 3 11

rP ,+ >π

pois 1 13

.

Page 124: Eureka 2010

Sociedade Brasileira de Matemática

EUREKA! N°32, 2010

62

Considerando ( )3 1 3 1r rP P p+ += como função de p, temos que ( )3 1rP p+ é um polinômio (de grau no máximo 3r + 1) em p, e portanto depende continuamente de

p. Como ( )3 1 0 0rP + = e 3 11 1

100rP ,+ > π

existe, pelo teorema do valor

intermediário, p com 10100

p< < tal que ( )3 11

rP p .+ =π

Solução alternativa para o item a): Podemos (Como no início da solução anterior), escrever

1

1

1

0 11 0 0

1 0

n n

n n

n n

P p p PQ p p Q , n .R p p R

+

+

+

− = − ⋅ ∀ ≥ −

Os autovalores dessa matriz 3 3× são 1 e 1 2 132 2

pi .− − ± ⋅

As normas dos autovalores distintos de 1 são iguais a ( )1 3 1 1p p ,− − < donde

n n nP ,Q e R convergem a certos números, que denotaremos por x, y, z, respectivamente. Devemos então ter:

( ) ( ) ( )1 1 1x p z py, y p x pz,z p y px,= − + = − + = − + donde

( ) ( )( ) ( ) ( )2 21 1 1 1x p z p p x pz p p x p p z x z= − + − + ⇒ − + = − + ⇒ = ⇒

( )1y p x px x,⇒ = − + = e logo 1 3x y z= = = (pois x + y + z = 1). PROBLEMA 4 Pelo pequeno teorema de Fermat, ( )303 1 31mod ,≡ e logo ( )( )3 31n mod é

periódico com período divisor de 30. Por outro lado, obviamente ( )( )31n mod é periódico com período 31.

Portanto, ( )( )3 31n n mod+ é periódico com período divisor de 31 30 930.⋅ = Pelo teorema chinês dos restos, para cada a com 0 29a≤ ≤ e b com 0 30b ,≤ ≤ existe um único ( )c a,b com ( )0 929c a,b≤ ≤ tal que ( ) ( )30c a,b a mod≡ e

( ) ( )31c a,b b mod .≡

Temos ( ) ( ) ( )3 3 31c a ,b ac a,b b mod .+ ≡ +

Page 125: Eureka 2010

Sociedade Brasileira de Matemática

EUREKA! N°32, 2010

63

Fixando a e fazendo b variar, 3a b+ percorre todas as 31 classes (mod 31). Assim, 3m m+ , 0 929m≤ ≤ passa 30 vezes por cada classe (mod 31). Como 930 31 30 31= ⋅ !, ( )3 0 31n n mod+ ≡ para 31!/31 = 30! inteiros positivos menores ou iguais a 31!. PROBLEMA 5 1ª. Solução Se x é uma raiz quarta da unidade, temos

( ) ( )2 3 2 2 3xf x d ax bx cx ,x f x c dx ax bx= + + + = + + + e

( )3 2 3x f x b cx dx ax ,= + + + de modo que

( )( )( )( )

( )22 2

33 3

1 1f xxf xx x

A f x .x f xx xx f xx x

= =

Assim, o vetor ( )2 31,x,x ,x é autovetor de A, com autovalor f(x), para x = 1, i, –1, –

i. deduzimos que A possui 4 autovetores independentes e, portanto, det A é o produto dos respectivos autovalores, ou seja, ( ) ( ) ( ) ( )1 1det A f f i f f i .= − − 2ª. Solução Observamos que det A é um polinômio do quarto grau nas variáveis a, b, c, d, enquanto f(1), f(i), f ( –1), f (– i) são polinômios irredutíveis distintos do primeiro grau nessas mesmas variáveis. Podemos realizar operações lineares nas linhas de A para provar que o polinômio det A é divisível por f(1), f(i), f (–1), f (–i). Isto fica mais rápido utilizando a mesma ideia da primeira Solução: se x é raiz quarta da unidade, multiplicando a segunda coluna por x, a terceira por x2 e a quarta por x3 e somando tudo isso à primeira coluna, obtemos ( ) ( ) ( ) ( )( )2 3f x ,xf x ,x f x ,x f x .

Assim, temos ( ) ( ) ( ) ( )1 1det A kf f i f f i ,= − − onde k é uma constante a ser determinada. Fazendo a = 1 e b = c = d = 0, obtemos det A = 1 e

( ) ( ) ( ) ( )1 1 1f f i f f i ,− − = logo k = 1, como queríamos demonstrar. PROBLEMA 6 Defina

( ) 20 1 2

0

n nn n

nf x a x a a x a x ... a x ...

=

= = + + + + +∑

Então

Page 126: Eureka 2010

Sociedade Brasileira de Matemática

EUREKA! N°32, 2010

64

( )( )2

0 0

nn

l n ln l

f x a a x∞

−= =

=

∑ ∑

( ) ( )0 0 0 1 1 0a a a a a a x ...= + + +

( )0 1 1 2 2 0n

n n n na a a a a a ... a a x ...− −+ + + + + + e

( ) ( ) 10

1 nn

nf´ x n a x

+=

= +∑

( )1 2 12 1 nna a x ... n a x ...+= + + + + +

Pela condição do enunciado, os coeficientes de ( )f´ x e de ( )( )2

3f xπ coincidem,

exceto pelo coeficiente constante. Temos portanto ( ) ( )( )2

3 3f´ x f x .π π

− = Logo

temos

( )221

3 1 3df dff dxdx f

π π= + ⇔ = ⇔

+∫ ∫

( ) ( ) ( )3 3

arctg f x C f x tg x Cπ π ⇔ = + ⇒ = +

Como ( ) 00 0f a ,= = concluímos que C = 0, e portanto

0

1 12 2 6 3

kk

k

a f tg .∞

=

π = = =

Page 127: Eureka 2010

Sociedade Brasileira de Matemática

EUREKA! N°32, 2010

65

XXXI OLIMPÍADA BRASILEIRA DE MATEMÁTICA Problemas e soluções da Segunda Fase – Nível Universitário

PRIMEIRO DIA PROBLEMA 1 Seja :[0,1] [0,1]f → crescente, derivável e inversível.

Se 1 1

1

0 0

( ) ( )f x dx f x dx−=∫ ∫ , prove que existem dois reais diferentes a e b,

0 1a b≤ < ≤ , tais que '( ) '( ) 1f a f b= = . Obs.: 1f − denota a inversa da função f . PROBLEMA 2 Seja N = {0,1,2,3,...}. Dados conjuntos ,A B ⊂ N , para cada inteiro positivo n

denote por r(A, B, n) o número de soluções da equação , ,a b n a A b B+ = ∈ ∈ . Prove que existe 0n ∈ tal que r(A, B, n + 1)> r(A, B, n) para todo 0n n> se e somente se \ AN e \BN são finitos. PROBLEMA 3 Dados 1 2, , ,..., nn a a a inteiros positivos, definimos 0 1 11,q q a= = e

1 1 1k k k kq a q q+ + −= + , para 1 1k n≤ ≤ − . Prove que, dado c > 1, existe K > 0 tal que, para todo M > K, existem n inteiro positivo e 1 2, ,..., na a a pertencentes a {1,2} tais que nM q c M≤ < ⋅ . SEGUNDO DIA PROBLEMA 4 Seja H o hiperboloide de equação 2 2 23 3 1 0x y z+ − − = . i) Prove que todo ponto ( , , )x y z H∈ pertence a exatamente duas retas contidas em H. ii) Prove que todas as retas contidas em H formam o mesmo ângulo com o plano de equação z = 0, e determine esse ângulo.

Page 128: Eureka 2010

Sociedade Brasileira de Matemática

EUREKA! N°32, 2010

66

PROBLEMA 5 Ache todas as funções :f →Z Z que satisfazem: i) f (f (n)) = f (n + 1), para todo n ∈Z . ii) f(2009n + 2008) = 2009.f(n) para todo n ∈Z . PROBLEMA 6 Para n inteiro positivo seja f(n) o número de produtos de inteiros maiores que 1 cujo resultado é no máximo n, isto é, f(n) é o número de k-uplas 1 2( , ,..., )ka a a

onde k é algum natural, ia ≥ 2 é inteiro para todo i e 1 2 ... ka a a n⋅ ⋅ ⋅ ≤ (contando a 0-upla vazia ( ), cujo produto dos termos é 1). Assim, por exemplo, f(1) = 1, por causa da 0-upla ( ) e f(6) = 9, por causa da 0-upla ( ), das 1-uplas (2), (3), (4), (5) e (6) e das 2-uplas (2, 2), (2, 3) e (3, 2).

Seja 1α > tal que 1

1 2m mα

=

=∑ .

a) Prove que existe uma constante K > 0 tal que ( )f n K nα≤ ⋅ para todo inteiro positivo n. b) Prove que existe uma constante c > 0 tal que ( )f n c nα≥ ⋅ para todo inteiro positivo n. PROBLEMA 1: SOLUÇÃO DE MARLON DE OLIVEIRA GOMES (FORTALEZA – CE) (I) Notemos primeiramente que (0) 0f = e (1) 1.f = De fato, f é inversível e portanto sobrejetora, logo, existem a e [ ]0,1b ∈ tais que ( ) 0f a = e ( ) 1.f b =

Se 0,a ≠ 1,b ≠

( ) [ )0, 0,f t t a< ∀ ∈

( ) ( ]1, ,1f t t b> ∀ ∈ Um absurdo. (II) Afirmação: f possui um ponto fixo em (0, 1). Isto é, ( )0,1c∃ ∈ tal que

( ) .f c c=

Prova: Suponha que seja ( ) ( ), 0,1 .f x x x> ∀ ∈

Então, ( )( ) ( )1 1 ,f f x x f x x− −= > ∀ pois f crescente 1f −⇔ crescente.

Logo, seria ( ) ( ) ( )1 , 0,1f x x f x x−> > ∀ ∈ ⇒

Page 129: Eureka 2010

Sociedade Brasileira de Matemática

EUREKA! N°32, 2010

67

( ) ( )1 1 1 1

0 0 0,f x dx xdx f x dx−> >∫ ∫ ∫ um absurdo, pois

( ) ( )1 1 1

0 0.f x dx f x dx−=∫ ∫

Se supusermos ( ) ( ), 0,1f x x x< ∀ ∈ temos um resultado análogo.

Suponha agora que existam ( )1 2 e 0,1x x ∈ tais que ( )1 1f x x> e ( )2 2.f x x<

Sendo f diferenciável, é também contínua, donde [ ] ( ) ( ): 0,1 , g g t f t t→ = − é contínua. Note que ( )1 0g x > e ( )2 0,g x < logo, pelo teorema do valor intermediário,

existe ( )1 2,c x x∈ tal que ( ) ( )0 ,g x f c c= ⇒ = o que prova o resultado.

(III) Pelo teorema do valor médio, existem ( )0,a c∈ e ( ),1b c∈

tais que ( ) ( ) ( ) ( ) ( )

( ) ( ) ( ) ( ) ( )

( ) 0 ´ 0 ´ 1

1(1) ´ 1 ´ 1

1

f cf c f f a c f a

cf c

f f c f b c f bc

− = ⋅ − ⇒ = =

− − = ⋅ − ⇒ = = −

.

PROBLEMA 2: SOLUÇÃO DE RAMON MOREIRA NUNES (FORTALEZA – CE) Para cada ,n∈ seja ( ), ,q A B n o número de pares ( ),x y tais que x y n+ = e,

além disso, x A∉ ou y B∉ . Assim, ( ), ,q A B n é o número de pares (x, y) com

x + y = n que ( ), ,r A B n não conta. Portanto, ( ) ( ), , 1 , , .q A B n n r A B n= + −

Veja que: ( ) ( ), , 1 , ,r A B n r A B n+ > ⇔

( ) ( ), , 1 , , 1r A B n r A B n⇔ + ≥ + ⇔

( ) ( )1 , , 1 , , 1n q A B n n q A B n⇔ + − + ≥ − + ⇔

( ) ( ), , 1 , ,q A B n q A B n⇔ + ≤

0n n∀ ≥ ; daí, ( ) ( )0 0, , , , , .q A B n q A B n n n≤ ∀ ≥

Como só existem finitos n menores que 0 ,n isso nos diz que ( ), ,q A B n é limitada como função de n. Agora, se (x, y) é tal que x y n+ = e , então ( , )x A x y∉ é contado por

( ), ,q A B n . Dessa forma, ( ) { }( ), , # 1,2,..., \ .q A B n n A≥

Page 130: Eureka 2010

Sociedade Brasileira de Matemática

EUREKA! N°32, 2010

68

Se \ A fosse infinito, poderíamos tomar { }( )# 1,2,..., \n A tão grande quanto

quiséssemos, mas como ( ), ,q A B n é limitada isso não pode ocorrer, logo, \ A é finito. Análogo \ B também é finito. Agora, suponha \ A e \ B finitos.

{ }{ }

1 2 1

1 2 1

\ , ..., ; ...

\ , ..., ; ...k k

m m

A a a a a a

B b b b b b

= < <

= < <

tome 0 .k mn a b= + Se 0 ,n n> tem-se que (x, y) com x + y = n então kx a> ou

my b> pois kx a≤ e 0.m k my b n x y a b n≤ ⇒ = + ≤ + = Absurdo.

Logo (x, y) não é contado por ( ), ,r A B n apenas quando \x A∈ ou \y B∈ (ambos não ocorrem simultaneamente); o primeiro caso ocorre n vezes e o segundo m vezes. Assim, ( ), , 1r A B n n k m= + − − , e logo r satisfaz a condição do enunciado. PROBLEMA 3: SOLUÇÃO DA BANCA Vamos escolher dois inteiros positivos grandes r, s e tomar , 1jm r s a= + = para

1 j r≤ ≤ e 2ja = para 1 .r j r s m+ ≤ ≤ + = Seja ( )1 2, ,..., ,k k kq q a a a= para

1 .k m≤ ≤ Temos 1 1,k k kq q q+ −= + para 1 1,k r≤ ≤ − e portanto 1,j jq F += para

0 ,j r≤ ≤ onde, para 1j ≥

( )1 11 1 5 1 5 1 52 2 25 5

j j j

j

oF

++ − + = − =

é o j-ésimo termo da sequência de Fibonacci. Assim,

( )( ) 1 51 12

j

jq o c +

= +

para j grande, onde 1 5 .2 5

c += Por outro lado, temos

1 12k k kq q q+ −= + para 1,r k m≤ ≤ − e logo 1 1,r j j r j rq u q u q+ + −= + onde ( )0j j

u≥

a sequência dada por 0 10, 1u u= = e 2 12 ,k k ku u u+ += + para 0.k ≥ Como

( ) ( ) ( ) ( )1 11 1 2 1 2 1 2 ,2 2 2 2

k k k

k

ou

+ = + − − = +

para 0,k ≥

obtemos

Page 131: Eureka 2010

Sociedade Brasileira de Matemática

EUREKA! N°32, 2010

69

( ) ( )( )( )1

1 11 2 1 2

2 2

j

r j r r

oq q q+ −

+= + + +

( ) ( )1 1 5 1 1 51 2 1 22 22 2

rjo

c + − +

= + + +

( )( ) ( )4 10 2 1 51 1 1 2 ,8 2

rj

o c + + +

= + +

para j e r grandes.

Como ( ) 1 5log 1 2 log2

++

é irracional (pois não é possível termos

( ) 1 51 2 ,2

mn +

+ =

com m e n inteiros positivos), a conclusão do problema

segue (tirando logaritmos) do seguinte fato, que é provado a seguir: Dados , 0>α β tais que α β é irracional, 0>ε e 0,r > existe 0 0x > tal que,

para qualquer 0, ,x x x∈ ≥ existem inteiros ,m n r≥ tais que

.m n x+ − <α β ε

Para provar este fato, notemos que, se ( )0n n n

p q≥

é a sequência de reduzidas da

fração contínua de ,α β podemos escolher k natural com 2 1 .kq + > β ε Teremos então (ver o artigo sobre Frações Contínuas na Revista Eureka! No. 3)

2 1 2 1 2 2 2 12 21 0 1k k k k kkq q p q p q+ + ++− < − < − < < − < <ε β α β α β ε β

e portanto 2 1 2 1 2 20 .k k k kq p q p+ +− < − < < − <ε α β α β ε Seja X o conjunto de todos os números da forma a b+α β com , 0a b ≥ inteiros. Se

2 1 2k ky a b q p+= + ≥ +α β α β pertence a X, temos 2 1ka q +≥ ou 2 ,kb p≥ e

portanto, tomando 1 2 1 2 1k kp q+ += −δ β α e 2 2 2 ,k kq p= −δ α β temos

1 20 ,< <δ δ ε e teremos 1y X+ ∈δ ou 2 .y X+ ∈δ Usando este fato

repetidamente, segue que, para todo [ ), , .z y z z X≥ + ∩ ≠ ∅ε Como, para

2 1 20 : ,k kq pk +

+=

α βα

temos 0k X∈α e 0 2 1 2 ,k kk q p+≥ +α α β portanto, para

Page 132: Eureka 2010

Sociedade Brasileira de Matemática

EUREKA! N°32, 2010

70

todo [ )0 , , .z k z z X≥ + ∩ ≠ ∅α ε Assim, tomando ( )0 0: ,x k r= + +α α β temos

que, dado 0 ,x x≥ temos ( ) 0 ,x r k− + ≥α β α e portanto

( ) ( ) ), .x r x r X − + − + + ∩ ≠ ∅ α β α β ε Assim, existem , 0a b ≥ inteiros

com ( ) ( ) ), ,a b x r x r+ ∈ − + − + +α β α β α β ε e portanto

( ) ( ) ,x r a r b x≤ + + + < +α β ε o que prova o fato acima. PROBLEMA 4: SOLUÇÃO DE GABRIEL LUIS MELLO DALALIO (S.J. DOS CAMPOS – SP) Tomando um ponto ( ), , ,x y z H∈ podemos representar uma reta que passa por esse ponto como:

( ){ }: , , , .r x ka y kb z kc k+ + + ∀ ∈ Para ( ), ,a b c que façam r estar contida em

H teremos:

( ) ( ) ( )2 2 23 3 1 0,x ka y kb z kc k+ + + − + − = ∀ ∈ ⇒ 2 2 2 2 2 2 2 2 23 6 3 3 6 3 2 1 0,x xka k a y ykb k b z zkc k c k⇒ + + + + + − − − − = ∀ ∈

Como 2 2 23 3 1 0,x y z+ − − = pois ( ), ,x y z H∈ tem-se:

2 2 2

3 3 0 (I)3 3 0 (II)

xa yb zca b c

+ − =

+ − =

(3x, 3y, –z) vetor normal

Plano

(I)

Cone

(II)

30o

60o

Ө

Como o menor ângulo que a direção (a, b, c) forma com o plano z = 0 é igual a

2 2,

carctg

a b

+ por (II) tem-se:

Page 133: Eureka 2010

Sociedade Brasileira de Matemática

EUREKA! N°32, 2010

71

( )2 2 23 60 .

3

c carctg arctg arctg

a b cθ

= = = = ° +

Assim está provado ii), qualquer reta contida em H terá ângulo igual a 60° com o plano z = 0. A solução do sistema é a interseção do plano com o cone, que pode ser apenas a origem, o que indicaria que não haveria r possível, ou pode ser igual a duas retas, indicando duas direções possíveis para (a, b, c), ou seja, duas retas r possíveis, ou ainda uma reta apenas de interseção, que é o caso de ( )3 ,3 ,x y z− ter ângulo

menor de 30° com o plano z = 0. Para que haja interseção de duas retas, o vetor normal do plano ( )3 ,3 ,x y z− deve ter um ângulo formado com o plano z = 0 menor que o ângulo do cone, que é de 30° . Esse ângulo é dado por

2 2 2

1 3039 9 3

z zarctg arctg arctg

x y z

< = = ° +

(pois ( )2 2 2 29 9 3 1 3 ).x y z z+ = + >

Então, como 2 2

30 ,9 9

zarctg

x y

< ° +

fica provado o item i).

PROBLEMA 5: SOLUÇÃO DE JOSIAS ELIAS DOS SANTOS ROCHA (MURIBECA - SE) Se f é injetiva, teremos ( ) 1,f n n= + pois ( ) ( )( ) 1f f n f n= + e além disso a

função ( ) 1f n n= + satisfaz (ii) ( )(2009 2008) 2009 .f n f n+ = Suponhamos

então que ( )1 2( )f n f n= com ( ) ( ) ( ) ( )2 21 2 1 2 1 21 1 ;n n f n f n f n f n< ⇒ = ⇒ + = +

indutivamente teremos ( ) ( )1 2f n k f n k+ = + para todo 0k ≥ , e assim f será

periódica a partir de 1n com periodo 2 1.n n−

Daí [ )( )1,f n∩ +∞ é finito, [ )( ) { }1 1, ,..., .kf n a a∩ +∞ =

Suponhamos sem perda de generalidade que { }1 : 1,...ia max a i k≥ = e que

( ) 1f m a= com m > 0 e 1m n> (f é periódica) ( ) 12009 2008 2009 ,f m a⇒ + =

mas [ ) ( ) [ )( )1 12009 2008 2009 2008 ,m n f m f n+ ∈ ∩ + ∞ ⇒ + ∈ ∩ +∞

Page 134: Eureka 2010

Sociedade Brasileira de Matemática

EUREKA! N°32, 2010

72

[ )( ) { }1 1 1 1 12009 0 0 , 0 .a a a a f n⇒ ≤ ⇒ = ⇒ = ⇒ ∩ +∞ =

Note-se ainda que ( )1 0f − = pois basta fazer 1n = − em (ii)

( ) ( ) ( )2009 1 1 1 0.f f f⇒ − = − ⇒ − = Assim ( ) ( )11 0f f n f− = = ⇒ é

periódica a partir de – 1 e ( ) 0, 1.f n n= ∀ ≥ − Seja 1,n < − com

( ) ( ) ( )( ) ( )2009 2008 2009 2009 1 2009 .f n r f n r f n f r= ⇒ + = ⇒ + = Assim,

ou 1r n= + ou f é periódica a partir de

( ) ( ) )( ) { }2009 1 2009 1 , 0 .n f n+ ⇔ ∩ + +∞ = Fazendo ( )1 2009 1m n= + e

12009 2008,k km m −= + teremos 1 21 ...,m m− > > > e além disso temos

( ) 0,kf m k= ∀ pois ( )1 0f m = e ( ) ( ) ( )1 2009 2008 2009 0k k kf m f m f m+ = + = =

se ( ) 0.kf m = Mas para todo 1n < − podemos achar im e 1im + tais que

( )1 0,i im n m f n+ ≤ < ⇒ = logo ( ) 0, .f n n= ∀ Assim temos apenas as seguintes soluções:

(1) ( ) 1, .f n n n= + ∀ ∈ (2) ( ) 0, .f n n= ∀ ∈

(3) ( ) 1, se 1.

0, se 1n n

f nn

+ ≤ −= ≥ −

PROBLEMA 6: SOLUÇÃO DA BANCA

Vamos inicialmente estender a função f para ),1[ +∞ definindo )()( xfxf = , ),1[ +∞∈∀x . Podemos agora mostrar que a função f

satisfaz a recorrência seguinte:

2( ) 1 ( ), [1, )

x

m

xf x f xm

=

= + ∀ ∈ +∞∑ . De fato, temos o vetor ( ) (correspondente a

0k = ), e, se 1≥k e ),...,,( 21 kaaa é tal que ja é inteiro, ja j ∀≥ ,2 e

xaaa k ≤...21 , então 12 a x≤ ≤ e, se 1a m= , 2... kxa am

≤ e há xfm

possíveis escolhas para 2( ,..., )ka a .

Page 135: Eureka 2010

Sociedade Brasileira de Matemática

EUREKA! N°32, 2010

73

Note também que 2α < , pois a função 1

1( )m mαζ α∞

=

= ∑ é decrescente e

21

1 1 1 1 1 1 1 1 1 1 1 1 11+ + + + + + +...<1+ + + + + + +...4 9 16 25 36 49 4 4 16 16 16 16

1 1=1+ + +...= 2.2 4

m m

=

=∑

Vamos resolver o item a): Mostraremos que ( ) , 1,f x x x≤ ∀ ≥α por indução em

.x Temos ( ) ( ]1 , 1,2 .f x x x= ≤ ∀ ∈α Para 2x ≥ temos , 2,x x mm

< ∀ ≥

e portanto, por hipótese de indução,

( )2 2

11 1 1 1x x

m m m x

x xf x f xm m m

= = >

= + ≤ + = + − ∑ ∑ ∑

αα

α

(pois 2 1

1 1 1 1).m mm m

∞ ∞

= =

= − =

∑ ∑α α

Como 1

1 2 3 4 5m x

x x x x xxm x x x x x>

> + + + + ≥ + + + + + ∑

α α α α αα

α

2 2 2 2 22 2 2 2 2 2 2 2 2 2 1,3 4 5 6 7 3 4 5 6 7

≥ + + + + > + + + + >

α α α α α

para

todo 2,x ≥ segue que ( ) ,f x x< α o que prova o resultado.

Vamos agora resolver o item b). Mostraremos inicialmente que 2 3.>α De fato,

1 1 1

1 1 1 1 1 1 1 2 4 1 1 1 11 ... ... ... ...2 3 4 2 4 4 2 4 8 2 2 4 8− − −

= + + + > + + + = + + + = + + + α α α α α α α α α α α α

1

1

1 112 2 ,1 2 21

2

⋅= =

−−

α

α

α

donde 2 2 1− >α e logo 2 3.>α Vamos mostrar que, se

6k ≥ e 2 ,kx < então ( ) 3 ,3 9 2

k

k kf x cx cx≥ ⋅ ≥− ⋅

α α onde 1

20480c = o que

claramente implica o resultado. Note que essa desigualdade vale para 6k = e 62 ,x < pois nesse caso

Page 136: Eureka 2010

Sociedade Brasileira de Matemática

EUREKA! N°32, 2010

74

( )2 123 81 25 1 .

3 9 2 17 4096 4096 4096

k

k k

x xcx cx c x f x= < ⋅ ⋅ = < < = ≤− ⋅

αα α α

Vamos agora mostrar essa desigualdade por indução em k. Suponhamos que ela vale para um certo 6.k ≥

Mostraremos que ( )1

1 1

33 9 2

k

k kf x cx+

+ +≥ ⋅− ⋅

α para todo x com 12 2 .k kx +≤ < Para

um tal valor de x, temos 1 2kxm

≤ < para 2 ,m x≤ ≤ e logo

( )2 2

2 2 2 2

3 3 11 1 .3 9 2 3 9 2

k k

k

x k k

k k k km m m m

x x xf x f f c c xm m m m

= = = >

= + > ≥ ⋅ ⋅ = ⋅ ⋅ ⋅ − − ⋅ − ⋅ ∑ ∑ ∑ ∑

αα

α

Temos( ) ( ) ( ) ( ) ( )11 2

2 2

1 1 1 1 1 1 1... ... ...22 2 1 2 1 2 1k k

kk k k km mm m ++ +

> ≥

< = + + + + + + + <+ − −

∑ ∑ α α α αα α α

( ) ( ) ( )( )

11

1 1 1 1

2 2 1 1 2 2... ... ...2 3 32 2 2

k kk k

k k k k

++

+ − + −

< + + = + + < + + α α α α

1 2 3pois 2 ,2 2

− = >

αα e, como

12 2 2... 3 ,3 3 3

k k k+ + + = ⋅

temos

( )1

1 1

3 2 31 3 ,3 9 2 3 3 9 2

kk k

k k k kf x c x c x+

+ +

> ⋅ ⋅ ⋅ − > ⋅ ⋅ − ⋅ − ⋅

α α pois

1

1 1

3 2 3 3 2 31 33 9 2 3 3 9 2 3 9 2

kk k k k

k k k k k k

+

+ +

− ⋅ ⋅ − ⋅ = > − ⋅ − ⋅ − ⋅ (de fato, essa última

desigualdade pode ser escrita como 1

1 1

6 2 9 21 1 ,3 9 2 3 9 2

k k

k k k k

+

+ +

⋅ ⋅+ > +

− ⋅ − ⋅ que

equivale a ( ) ( )1 16 3 9 2 18 3 9 2 ,k k k k+ +− ⋅ > − ⋅ que por sua vez equivale a

162 2 108 2 ,k k⋅ > ⋅ que obviamente vale para todo k). Obs: α é aproximadamente igual a 1,7286472389....

Kálmar provou que ( )

( )1lim 0,3181736521...

´x

f xx→∞

−= =α αζ α

Page 137: Eureka 2010

Sociedade Brasileira de Matemática

EUREKA! N°32, 2010

75

XXXI OLIMPÍADA BRASILEIRA DE MATEMÁTICA PREMIADOS

Nível 1 (6º. e 7º. Anos) NOME CIDADE – ESTADO PRÊMIO Alexandre Mendonça Cardoso Salvador – BA Ouro Daniel de Almeida Souza Brasília – DF Ouro Pedro Henrique Alencar Costa Fortaleza – CE Ouro Ana Beatriz Motta Aragão Cortez Campinas – SP Ouro Cristhian Mafalda Leme – SP Ouro Érika Rizzo Aquino Goiânia – GO Ouro Bianca Lima Barretto Salvador – BA Prata Adriana de Sousa Figueiredo Porto Alegre – RS Prata Ricardo Ken Wang Tsuzuki São Paulo – SP Prata João Pedro Sedeu Godoi Rio de Janeiro – RJ Prata Leonardo Gomes Gonçalves Brasília – DF Prata Leonardo Gushiken Yoshitake São Paulo – SP Prata Paulo Henrique Omena de Freitas São Paulo – SP Prata Edgar Kenji Ishida São Paulo – SP Prata Rodrigo Pommot Berto Brasília – DF Prata Kiane Sassaki Menezes Rio de Janeiro – RJ Prata Dimas Macedo de Albuquerque Fortaleza – CE Prata Mauricio Najjar da Silveira São Paulo – SP Bronze Murilo Corato Zanarella Amparo – SP Bronze Victor Almeida Costa Fortaleza – CE Bronze Elcio Koodiro Yoshida São Paulo – SP Bronze Carolina Lima Guimarães Vitória – ES Bronze Bruno da Silveira Dias Florianópolis – SC Bronze Emilly Guaris Costa Maceió – AL Bronze Bruno Almeida Costa Fortaleza – CE Bronze Gabriel Averbug Zukin Rio de Janeiro – RJ Bronze Marcelo Ericsson de Carvalho São Paulo – SP Bronze Sarah Barreto Ornellas Salvador – BA Bronze Isabella Ayres Pinheiro de Lima Goiânia – GO Bronze Shadi Bavar Blumenau – SC Bronze Viviane Silva Souza Freitas Salvador – BA Bronze Matheus José Araújo Oliveira Recife – PE Bronze Beatriz Miranda Macedo Niterói – RJ Bronze Matheus Uchôa Constante Goiânia – GO Bronze Julio S. Akiyoshi São Paulo – SP Menção Honrosa Antonio Wesley de Brito Vieira Cocal de Alves – PI Menção Honrosa Vinicius Jóras Padrão Rio de Janeiro – RJ Menção Honrosa Mateus Guimarães Lima de Freitas Fortaleza – CE Menção Honrosa Vitor Dias Gomes Barrios Marin Presidente Prudente – SP Menção Honrosa Mariana Teatini Ribeiro Belo Horizonte – MG Menção Honrosa Rodrigo Silva Ferreira Salvador – BA Menção Honrosa Artur Souto Martins Fortaleza – CE Menção Honrosa Tiago Martins Nápoli Itú – SP Menção Honrosa

Page 138: Eureka 2010

Sociedade Brasileira de Matemática

EUREKA! N°32, 2010

76

Laís Monteiro Pinto Rio de Janeiro – RJ Menção Honrosa Guilherme Anitele Silva Presidente Prudente – SP Menção Honrosa Pedro Papa Paniago Belo Horizonte – MG Menção Honrosa Gabriel Yudi Hirata São Paulo – SP Menção Honrosa Iago Carvalho de Moraes Recife – PE Menção Honrosa Adam Yuuki Oyama Curitiba – PR Menção Honrosa Luíze Mello D´urso Vianna Rio de Janeiro – RJ Menção Honrosa Enrico Pascucci Loffel S.B.do Campo – SP Menção Honrosa Daniel Charles M. Gomes Mogi das Cruzes – SP Menção Honrosa Ellen Tamie Ikefuti Morishigue Bastos – SP Menção Honrosa Ana Emília Hernandes Dib S.J. do Rio Preto – SP Menção Honrosa Marcelo Liu Guo São Paulo – SP Menção Honrosa Gabriel Queiroz Moura Teresina – PI Menção Honrosa Gabriel Branco Frizzo Curitiba – PR Menção Honrosa Ana Jéssyca Mendes Belarmino Fortaleza – CE Menção Honrosa Mariana Bonfim Moraes Morant de Holanda Rio de Janeiro – RJ Menção Honrosa Ricardo Vidal Mota Peixoto Vassouras – RJ Menção Honrosa Bruno de Marchi Andrade Valinhos – SP Menção Honrosa Juliana Amoedo Amoedo Plácido Salvador – BA Menção Honrosa

Nível 2 (8º. e 9º. Anos) NOME CIDADE – ESTADO PRÊMIO André Macieira Braga Costa Belo Horizonte – MG Ouro Gabriel Ilharco Magalhães Juiz de Fora – MG Ouro Daniel Eiti Nishida Kawai Atibaia – SP Ouro Henrique Gasparini Fiuza do Nascimento Brasília – DF Ouro Marina Pessoa Mota Fortaleza – CE Ouro Fellipe Sebastiam S. P. Pereira Rio de Janeiro – RJ Ouro Liara Guinsberg São Paulo – SP Prata Lara Timbó Araújo Fortaleza – CE Prata Victor Kioshi Higa São Paulo – SP Prata Fernando Lima Saraiva Filho Eusébio – CE Prata Lucas Cawai Julião Pereira Caucaia – CE Prata Mateus Henrique Ramos de Souza Pirapora – MG Prata Henrique Vieira Gonçalves Vaz São Paulo – SP Prata Lucas Nishida Pedreira – SP Prata Pedro Víctor Falci de Rezende Juiz de Fora – MG Prata Rafael Kazuhiro Miyazaki São Paulo – SP Prata Francisco Markan Nobre de Souza Filho Fortaleza – CE Bronze Vincent Cherng Hsi Lee São Paulo – SP Bronze Vinícius Canto Costa Salvador – BA Bronze Thiago Poeiras Silva Belo Horizonte – MG Bronze Victor de Oliveira Bitaraes Betim – MG Bronze Victor Hugo Corrêa Rodrigues Rio de Janeiro – RJ Bronze Luciano Drozda Dantas Martins Fortaleza – CE Bronze Breno Leví Corrêa Campo Belo – MG Bronze

Page 139: Eureka 2010

Sociedade Brasileira de Matemática

EUREKA! N°32, 2010

77

Wilson Aparecido Sedano Filho Paulínia – SP Bronze Victor Venturi Campinas – SP Bronze Daniel Lima Santanelli Rio de Janeiro – RJ Bronze Felipe Penha Alves São Luís – MA Bronze Lucas Grimauth Evangelista São Paulo – SP Bronze Gabriel Nogueira Coelho de Togni de Souza Rio de Janeiro – RJ Bronze Ana Thais Castro de Santana Rio de Janeiro – RJ Bronze Caio Cesar do Prado Dorea Reis Nova Iguaçu – RJ Bronze Rafael Rodrigues Rocha de Melo Caucaia – CE Menção Honrosa Murilo Freitas Yonashiro Coelho São Paulo – SP Menção Honrosa Gabriel José Moreira da Costa Silva Maceió – AL Menção Honrosa Nathalia Novello Fernandes Ribeiro Rio de Janeiro – RJ Menção Honrosa Gabriel Pacianotto Gouveia São Paulo – SP Menção Honrosa Pedro Ivo Coêlho de Araújo Fortaleza – CE Menção Honrosa Elias Brito Oliveira Brasília – DF Menção Honrosa Fernando Tomimura Miyashiro São Paulo – SP Menção Honrosa Igor Augusto Marques do Carmo Juiz de Fora – MG Menção Honrosa Juliane Trianon Fraga São Paulo – SP Menção Honrosa Aimê Parente de Sousa Fortaleza – CE Menção Honrosa Tadeu Pires de Matos Belfort Neto Fortaleza – CE Menção Honrosa Yuri Zeniti Sinzato Brasília – DF Menção Honrosa Gabriel Sena Galvão Brasília – DF Menção Honrosa Filipe Mourão Leite Teresina – PI Menção Honrosa Gabriela Loiola Vilar Fortaleza – CE Menção Honrosa Marcelo Cargnelutti Rossato Santa Maria – RS Menção Honrosa Pedro Henrique Botolozo Maria Curitiba – PR Menção Honrosa Jair Gomes Soares Júnior Montes Claros – MG Menção Honrosa Maria Clara Cardoso São Paulo – SP Menção Honrosa Júlio César Prado Soares Brasília – DF Menção Honrosa Fábio Kenji Arai São Paulo – SP Menção Honrosa Julio Barros de Paula Taubaté – SP Menção Honrosa Francisco Matheus Gonçalves de Souza João Pessoa – PB Menção Honrosa Daniel Kantorowitz Bragança Paulista – SP Menção Honrosa Ivan Tadeu Ferreira Antunes Filho Lins – SP Menção Honrosa Vitor Ramos de Paula Belo Horizonte – MG Menção Honrosa Victor Santos de Andrade Teresina – PI Menção Honrosa

Nível 3 (Ensino Médio) NOME CIDADE – ESTADO PRÊMIO Renan Henrique Finder São Paulo – SP Ouro Marcelo Tadeu de Sá Oliveira Sales Salvador – BA Ouro Davi Lopes Alves de Medeiros Fortaleza – CE Ouro Matheus Secco Torres da Silva Rio de Janeiro – RJ Ouro Hugo Fonseca Araújo Rio de Janeiro – RJ Ouro Marco Antonio Lopes Pedroso Santa Isabel – SP Ouro Gustavo Lisbôa Empinotti Florianópolis – SC Prata

Page 140: Eureka 2010

Sociedade Brasileira de Matemática

EUREKA! N°32, 2010

78

Marlen Lincoln da Silva Fortaleza – CE Prata Deborah Barbosa Alves São Paulo – SP Prata Illan Feiman Halpern São Paulo – SP Prata Thiago Ribeiro Ramos Varginha – MG Prata Hanon Guy Lima Rossi São Paulo – SP Prata João Lucas Camelo Sá Fortaleza – CE Prata Carlos Henrique de Andrade Silva Fortaleza – CE Prata Custódio Moreira Brasileiro Silva Embu – SP Prata Rafael Alves da Ponte Fortaleza – CE Prata Matheus Barros de Paula Taubaté – SP Bronze Bruno Silva Mucciaccia Vitória – ES Bronze Matheus Araujo Marins São Gonçalo – RJ Bronze Guilherme da Rocha Dahrug Santo André – SP Bronze Victorio Takahashi Chu São Paulo – SP Bronze Voltaire Laplace dos Reis Manhuaçu – MG Bronze Jardiel Freitas Cunha Recife – PE Bronze Rafael Horimoto de Freitas São Paulo – SP Bronze Lucas de Freitas Smaira Guaxupé – MG Bronze Robério Soares Nunes Riberio Preto – SP Bronze Gabriel Militão Vinhas Lopes Fortaleza – CE Bronze Alvaro Lopes Pedroso Santa Isabel – SP Bronze Alan Anderson da Silva Pereira União dos Palmares – AL Bronze Maria Clara Mendes Silva Pirajuba – MG Bronze Nara Gabriela de Mesquita Peixoto Fortaleza – CE Bronze Rodrigo de Sousa Serafim da Silva Itatiba – SP Bronze Rodrigo Rolim Mendes de Alencar Fortaleza – CE Menção Honrosa João Mendes Vasconcelos Fortaleza – CE Menção Honrosa Otávio Augusto de Oliveira Mendes Pilar do Sul – SP Menção Honrosa Renan Roveri do Amaral Gurgel Jundiaí – SP Menção Honrosa Fernando Fonseca Andrade Oliveira Belo Horizonte – MG Menção Honrosa Caíque Porto Lira Fortaleza – CE Menção Honrosa Gustavo Haddad Francisco e S. Braga S.J.dos Campos – SP Menção Honrosa Gustavo Cellet Marques São Paulo – SP Menção Honrosa Wagner Rosales Chaves Jundiaí – SP Menção Honrosa Wagner Carlos Morêto Loyola Filho Vitória – ES Menção Honrosa James Jun Hong São Paulo – SP Menção Honrosa Kayo de França Gurgel Fortaleza – CE Menção Honrosa Nathana Alcântara Lima Fortaleza – CE Menção Honrosa Gabriel Lima Guimarães Vitória – ES Menção Honrosa Ivan Guilhon Mitoso Rocha Fortaleza – CE Menção Honrosa Elder Massahiro Yoshida São Paulo – SP Menção Honrosa Ruan Alves Pires Rio de Janeiro – RJ Menção Honrosa André Saraiva Nobre dos Santos Fortaleza – CE Menção Honrosa Luiz Filipe Martins Ramos Niterói – RJ Menção Honrosa Felipe Abella C. Mendonça de Souza João Pessoa – PB Menção Honrosa Eduardo Machado Capaverde Florianópolis – SC Menção Honrosa Thales Sinelli Lima São Paulo – SP Menção Honrosa Tuane Viana Pinheiro Rio de Janeiro – RJ Menção Honrosa Vinícius Cipriano Klein Viçosa – MG Menção Honrosa

Page 141: Eureka 2010

Sociedade Brasileira de Matemática

EUREKA! N°32, 2010

79

André Austregésilo Scussel Fortaleza – CE Menção Honrosa Thiago Augusto da Silva Baleixo Rio de Janeiro – RJ Menção Honrosa Marcos Massayuki Kawakami São Paulo – SP Menção Honrosa Ana Luísa de Almeida Losnak São Paulo – SP Menção Honrosa

Nível Universitário NOME CIDADE – ESTADO PRÊMIO Leonardo Ribeiro de Castro Carvalho São Paulo – SP Ouro Régis Prado Barbosa Fortaleza – CE Ouro Rafael Assato Ando Campinas – SP Ouro Rafael Daigo Hirama Campinas – SP Ouro Guilherme Rodrigues Nogueira de Souza São Paulo – SP Ouro Adenilson Arcanjo de Moura Junior Fortaleza – CE Ouro Felipe Rodrigues Nogueira de Souza São Paulo – SP Ouro Ramon Moreira Nunes Fortaleza – CE Ouro Thomás Yoiti Sasaki Hoshina Rio de Janeiro – RJ Prata Rafael Tupynambá Dutra Belo Horizonte – MG Prata Luís Fernando Schultz Xavier da Silveira Florianópolis – SC Prata Thiago Costa Leite Santos São Paulo – SP Prata Gabriel Ponce São Carlos – SP Prata Carlos Henrique Melo Souza S.J. dos Campos – SP Prata Marcelo Matheus Gauy São Paulo – SP Prata Antônio Felipe Cavalcante Carvalho Fortaleza – CE Prata Kellem Corrêa Santos Rio de Janeiro – RJ Prata Rafael Montezuma Pinheiro Cabral Fortaleza – CE Prata Ricardo Turolla Bortolotti Rio de Janeiro – RJ Bronze Alexandre Hideki Deguchi Martani São Paulo – SP Bronze Rafael Sampaio de Rezende Fortaleza – CE Bronze Maurício de Lemos Rodrigues Collares Neto Aracajú – SE Bronze Joas Elias dos Santos Rocha Muribeca – SE Bronze Gabriel Luís Mello Dalalio S.J. dos Campos – SP Bronze Carlos Coelho Lechner Rio de Janeiro – RJ Bronze Enzo Haruo Hiraoka Moriyama São Paulo – SP Bronze Paulo Sérgio de Castro Moreira Fortaleza – CE Bronze Helder Toshiro Susuki São Paulo – SP Bronze Mateus Oliveira de Figueiredo Fortaleza – CE Bronze Gabriel Caser Brito Rio de Janeiro – RJ Bronze Paulo André Carvalho de Melo S.J. dos Campos – SP Bronze Caio Ishizara Costa S.J. dos Campos – SP Bronze Willy George do Amaral Petrenko Rio de Janeiro – RJ Bronze Sidney Cerqueira Bispo dos Santos Filho S.J. dos Campos – SP Bronze Rafael Endlich Pimentel Vitória – ES Bronze Guilherme Philippe Figueiredo São Paulo – SP Bronze Bruno da Silva Santos Belford Roxo – RJ Bronze Francisco Osman Pontes Neto Fortaleza – CE Bronze Luty Rodrigues Ribeiro S.J. dos Campos – SP Bronze

Page 142: Eureka 2010

Sociedade Brasileira de Matemática

EUREKA! N°32, 2010

80

Renato Rebouças de Medeiros Fortaleza – CE Menção Honrosa José Olegário de Oliveira Neto S.J. dos Campos – SP Menção Honrosa Eduardo Fischer Encantado – RS Menção Honrosa Guilherme Lourenço Mejia S.J. dos Campos – SP Menção Honrosa Jorge Henrique Craveiro de Andrade Rio de Janeiro – RJ Menção Honrosa Edson Augusto Bezerra Lopes Fortaleza – CE Menção Honrosa Eric Campos Bastos Guedes Niterói – RJ Menção Honrosa Thiago da Silva Pinheiro São Paulo – SP Menção Honrosa Leandro Farias Maia Fortaleza – CE Menção Honrosa Pedro Paulo Albuquerque Goes Fortaleza – CE Menção Honrosa Álvaro Krüger Ramos Porto Alegre – RS Menção Honrosa Alysson Espíndola de Sá Silveira Fortaleza – CE Menção Honrosa Alfredo Roque de Oliveira Freire Filho S.J. dos Campos – SP Menção Honrosa Rafael Ghussn Cano Campinas – SP Menção Honrosa Antônio Deromir Neves da Silva Júnior Fortaleza – CE Menção Honrosa Rafael Sabino Lima Rio de Janeiro – RJ Menção Honrosa Hudson do Nascimetno Lima Fortaleza – CE Menção Honrosa Diego Andrés de Barros Lima Barbosa Rio de Janeiro – RJ Menção Honrosa Reinan Ribeiro Souza Santos Aracaju – SE Menção Honrosa Marcos Victor Pereira Vieira Fortaleza – CE Menção Honrosa Daniel Ungaretti Borges Belo Horizonte – MG Menção Honrosa

Page 143: Eureka 2010

Sociedade Brasileira de Matemática

EUREKA! N°32, 2010

81

AGENDA OLÍMPICA

XXXII OLIMPÍADA BRASILEIRA DE MATEMÁTICA

NÍVEIS 1, 2 e 3 Primeira Fase – Sábado, 12 de junho de 2010

Segunda Fase – Sábado, 18 de setembro de 2010 Terceira Fase – Sábado, 16 de outubro de 2010 (níveis 1, 2 e 3)

Domingo, 17 de outubro de 2010 (níveis 2 e 3 - segundo dia de prova).

NÍVEL UNIVERSITÁRIO Primeira Fase – Sábado, 18 de setembro de 2010

Segunda Fase – Sábado, 16 e Domingo, 17 de outubro de 2010

ASIAN PACIFIC MATH OLYMPIAD (APMO) 06 de março de 2010

XVI OLIMPÍADA DE MAIO

08 de maio de 2010

XXI OLIMPÍADA DE MATEMÁTICA DO CONE SUL 13 a 19 de junho de 2010

Águas de São Pedro, SP – Brasil

LI OLIMPÍADA INTERNACIONAL DE MATEMÁTICA 02 a 14 de julho de 2010

Astana, Cazaquistão

XVII OLIMPÍADA INTERNACIONAL DE MATEMÁTICA UNIVERSITÁRIA 24 a 30 de julho de 2010

Blagoevgrad, Bulgária

XXIV OLIMPÍADA IBEROAMERICANA DE MATEMÁTICA 17 a 27 de setembro de 2010

Paraguai

II COMPETIÇÃO IBEROAMERICANA INTERUNIVERSITÁRIA DE MATEMÁTICA 3 a 9 de outubro de 2010

Rio de Janeiro, Brasil

XIII OLIMPÍADA IBEROAMERICANA DE MATEMÁTICA UNIVERSITÁRIA

Page 144: Eureka 2010

Sociedade Brasileira de Matemática

EUREKA! N°32, 2010

82

COORDENADORES REGIONAIS

Alberto Hassen Raad (UFJF) Juiz de Fora – MG Américo López Gálvez (USP) Ribeirão Preto – SP Andreia Goldani FACOS Osório – RS Antonio Carlos Nogueira (UFU) Uberlândia – MG Benedito Tadeu Vasconcelos Freire (UFRN) Natal – RN Carmen Vieira Mathias (UNIFRA) Santa María – RS Claus Haetinger (UNIVATES) Lajeado – RS Cláudio de Lima Vidal (UNESP) S.J. do Rio Preto – SP Denice Fontana Nisxota Menegais (UNIPAMPA) Bagé – RS Disney Douglas Lima de Oliveira (UFAM) Manaus – AM Edson Roberto Abe (Colégio Objetivo de Campinas) Campinas – SP Edney Aparecido Santulo Jr. (UEM) Maringá – PR Élio Mega (Grupo Educacional Etapa) São Paulo – SP Eudes Antonio da Costa (Univ. Federal do Tocantins) Arraias – TO Fábio Brochero Martínez (UFMG) Belo Horizonte – MG Florêncio Ferreira Guimarães Filho (UFES) Vitória – ES Francinildo Nobre Ferreira (UFSJ) São João del Rei – MG Genildo Alves Marinho (Centro Educacional Leonardo Da Vinci) Taguatingua – DF Graziela de Souza Sombrio (UNOCHAPECÓ) Chapecó – SC Gilson Tumelero (UTFPR) Pato Branco – PR Ivanilde Fernandes Saad (UC. Dom Bosco) Campo Grande – MS João Benício de Melo Neto (UFPI) Teresina – PI João Francisco Melo Libonati (Grupo Educacional Ideal) Belém – PA Jose de Arimatéia Fernandes (UFPB) Campina Grande – PB José Luiz Rosas Pinho (UFSC) Florianópolis – SC José Vieira Alves (UFPB) Campina Grande – PB José William Costa (Instituto Pueri Domus) Santo André – SP Krerley Oliveira (UFAL) Maceió – AL Licio Hernandes Bezerra (UFSC) Florianópolis – SC Luciano G. Monteiro de Castro (Sistema Elite de Ensino) Rio de Janeiro – RJ Luzinalva Miranda de Amorim (UFBA) Salvador – BA Marcelo Rufino de Oliveira (Grupo Educacional Ideal) Belém – PA Marcelo Mendes (Colégio Farias Brito, Pré-vestibular) Fortaleza – CE Newman Simões (Cursinho CLQ Objetivo) Piracicaba – SP Nivaldo Costa Muniz (UFMA) São Luis – MA Nivaldo de Góes Grulha Jr. (USP – São Carlos) São Carlos – SP Osnel Broche Cristo (UFLA) Lavras – MG Uberlândio Batista Severo (UFPB)) João Pessoa – PB Raul Cintra de Negreiros Ribeiro (Colégio Anglo) Atibaia – SP Ronaldo Alves Garcia (UFGO) Goiânia – GO Rogério da Silva Ignácio (Col. Aplic. da UFPE) Recife – PE Reginaldo de Lima Pereira (Escola Técnica Federal de Roraima) Boa Vista – RR Reinaldo Gen Ichiro Arakaki (UNIFESP) SJ dos Campos – SP Ricardo Amorim (Centro Educacional Logos) Nova Iguaçu – RJ Sérgio Cláudio Ramos (IM-UFRGS) Porto Alegre – RS Seme Gebara Neto (UFMG) Belo Horizonte – MG Tadeu Ferreira Gomes (UEBA) Juazeiro – BA Tomás Menéndez Rodrigues (U. Federal de Rondônia) Porto Velho – RO Valdenberg Araújo da Silva (U. Federal de Sergipe) São Cristovão – SE Vânia Cristina Silva Rodrigues (U. Metodista de SP) S.B. do Campo – SP Wagner Pereira Lopes (CEFET – GO) Jataí – GO